Binder 1

You might also like

Download as pdf or txt
Download as pdf or txt
You are on page 1of 168

1

CHAPTER
Life Processes

School Level

Introduction
In the world outside, where you study, play, and perform arts, there is a network of processes that maintains everything.

R
You eat and rest at home, travel to school to get educated, learn to play guitar strings at a local arts centre and then return
home to sleep. Have you wondered why? Cars exhaust if they run continuously for long, but your heart pumps blood

A
through out your life with out ever asking for a timeout! What makes us unique; what does it cost to be alive?

M
We can find many things around us, from plants and animals to mountains and the ocean. The earth on which we live

U
is composed of various things. These “things” can be classified into two distinct types: Living and Non-living things.

K
‰ Living things: They can breathe, eat, move, grow, reproduce and have senses.
‰ Non-living things: They do not grow, move, breathe, eat or reproduce. They do not have any senses.
T
N
While the above characteristics are general observations, many bacteria do not ‘breathe’, they can make their food in sun,
some bees do not reproduce. So, then how do we define living?
A
SH

What are Life Processes?


A

A bacterium and a giant blue whale are made up of cells with membranes and structures made up of molecules. The
PR

complexity of structure can vary. Cells are the functional and structural units of all living organisms. Cell aggregates
together make tissue, tissue organizes themselves to make organs and organs make organs system and organ system finally
constitute the body of the living organism. Various organisms are unicellular, while some of them are multicellular. Viruses
R.

do not show any molecular movement in them (until they infect some cell), which is why, there is a controversy about
D

whether they are truly alive.


However, this ordered structure is bound to break due to several fundamental reasons, primarily related to the inherit
challenges and limitation of life process and nature of the physical world. For this, we need molecular movement, or
metabolism, as molecular movements are essential for wide range of biological process from basic cellular function
to complex physiological organism function. We must acquire energy and matter from surroundings to repair and
even grow or reproduce. Hence, living beings are ordered structure capable of self maintenance. Life processes are
maintenance processes required to execute properties of living. The major life processes include:
‰ acquiring matter and energy- nutrition
‰ releasing energy to fuel metabolic reactions – respiration
‰ transport food, and energy within the organism – transportation
‰ throw waste materials generated during metabolism – excretion
It is essential to understand here how transportation has evolved. Unicellular organisms need small-distance transport
using diffusion or simple cytoplasmic streaming. Diffusion works for short distances. Multicellular organisms could
store food and perform complex tasks due to specialization. However, this meant that transport of substances between
specialized organs became challenging. Nature has solved this over a billion years, giving us the variety we see around us.

Metabolism and Types


Metabolism is the sum total of all the biochemical reactions involved in maintaining the living condition of the cells
in an organism. All living organisms require energy for different essential processes and for producing new organic
substances.
The metabolic processes help in growth, reproduction and in maintaining the structures of living organisms. The organisms
respond to the surrounding environment due to metabolic activities. All the chemical reactions occurring in the living
organisms from digestion to transportation of the substances from cell to cell requires energy.

What is Anabolism?
Anabolism is a series of biochemical reactions that synthesize molecules from smaller components. These are endergonic

R
reactions, since energy is consumed to form bonds. They require energy to progress. The hydrolysis of ATP (Adenosine

A
triphosphate) powers several anabolic reactions.

M
Anabolic hormones cause anabolic processes to occur. Insulin, which facilitates glucose absorption, and anabolic steroids,
which enhance muscular growth, are examples of anabolic hormones.

U
Examples of anabolism are –
(i) Fatty acid and glycerol react to produce a fatty acid. K
T
(ii) Amino acids join together to prepare dipeptides.
N
(iii) Simple sugars combine to synthesize water and disaccharides.
A

(iv) Water and carbon dioxide react to produce glucose and oxygen for the process of photosynthesis.
SH

What is Catabolism?
It is also a series of biochemical reactions that breaks down complex molecules into simpler ones. Additionally, this
A

process is spontaneous and thermodynamically favourable. Thus, human body cells use this process to generate energy
PR

for anabolism.
Catabolism is exergonic. It works through hydrolysis and oxidation, releasing heat in the process. Many hormones operate
as catabolism-controlling signals. Adrenaline, glucagon, cortisol are all catabolic hormones.
R.

Cells often store various complex molecules and raw materials. Catabolism breaks down these to create new products.
D

For instance, the catabolism of polysaccharides, nucleic acids, and protein generates monosaccharides, nucleotides, and
amino acids, respectively.

Examples of catabolism are –


(i) Oxygen and glucose react during cellular respiration to yield water and carbon dioxide.
(ii) Glycolysis
(iii) Digestion of food

Nutrition
Nutrition is defined as a process by which living organisms procure nutrients (food) or synthesize them and change them
into simple absorbable forms by a series of biochemical processes. There are two basic modes of nutrition: Autotrophic
and Heterotrophic.

2 Class-X BIOLOGY PW
Nutrients
Nutrients are the substances required by the body for its growth, repair, work and maintenance of the body. Distinct types
of nutrients are carbohydrates, fats, proteins, vitamins and minerals. Our daily energy requirement may vary according
to our occupation, age, sex, etc.

Types of Nutrition
The general requirement for energy and materials is similar in all organisms, but it is fulfilled in distinct ways.

Nutrition

Autotrophic Heterotrophic
nutrition nutrition

1. Autotrophic Nutrition

R
(Auto means self; trophic means food)

A
In autotrophic nutrition, green plants take in raw materials such as carbon dioxide and water from the outside and convert

M
them into stored energy (carbohydrates) in the presence of sunlight and chlorophyll.
(a) Photoautotrophs: Green plants synthesize food through the process of photosynthesis, using simple raw materials like

U
water and CO2 in the presence of sunlight. Chloroplast is the site of photosynthesis. Chlorophyll is the photosynthetic
pigment which traps solar energy and is present in the chlorophyll.
6CO + 12H 2 O + Light energy 
K → C6 H12 O6 + 6O 2 + 6H 2 O
T
2 Chlorophyll
Carbon dioxide Water
N

Since, autotrophic plants are able to produce food for others, therefore, they are also known as producers. Because the
A

autotrophic nutrition is the characteristic of plants, therefore it is also called holophytic nutrition.
SH

(b) Chemoautotrophs: Some non-green bacteria like sulphur bacteria uses chemical energy to manufacture their food.
This energy is derived from chemical reactions occurring in the bacteria. This process is called chemosynthesis.
Chemosynthesis bacteria do not require light as the source of energy.
A

2H2S + O2 → 2H2O + S + 126 KCal


PR

2S + 2H2 + O2 → 2H2SO4 + 141.8 KCal


R.

2. Heterotrophic Nutrition
(Hetero means different; trophic means food)
D

‰ It is a mode of nutrition in which the organism derives its nutrition by taking ready made food, from dead, or living plants
and animals. The survival of heterotrophs directly or indirectly depends on autotrophs.
As heterotrophs depends on other organism for their food, therefore, they are called consumers.
The heterotrophic mode of nutrition is categorised into three types based on method of acquiring food.
(i) Saprophytic nutrition: It is a type of nutrition in which organism releases some enzymes to digest dead organic
food and derive nourishment from dead and decaying matter like excreta, fallen leaves, broken twigs etc. Such
organisms are called saprophytes. e.g., Fungi, Yeast and Mushroom.
(ii) Parasitic nutrition: In this mode of nutrition, some organisms live outside and inside the body of other organism
(called host) and drive their nourishment from the host without filling it. The mode of nutrition in these organism
are called Parasitic nutrition and such animals are called Parasites. e.g., Plasmodium (Malarial Parasite), Ascaris
(Roundworm) are parasites in human body. Cuscuta (Amarbel) is a plant parasite.

Life Processes 3
(iii) Holozoic nutrition: It is a mode of nutrition in which the animal takes in complex solid food or whole material
and break down (digestion) of the food takes place in the body.
(a) Depending upon the food habits, Holozoic animals are classified as:
Herbivore (Cow), Carnivore (Lion) and Omnivore (Rat).
(b) There are five major steps in Holozoic nutrition. The steps for obtaining nutrition follow certain steps in a
sequence. It begins with the ingestion of food through the mouth. Thereafter the food enters the process of
digestion in which the complex food items are broken down into simpler forms by the action of various enzymes
in the alimentary canal. Once the food is digested it gets absorbed into the bloodstream via the villi in the small
intestine and this process is called absorption. And blood transports the digested food to all parts of the body
for the release of energy. which is known as assimilation. Post the digestion process the food that does not get
digested is eliminated by the body as water and that process is known as egestion.

Nutrition in Plants: Photosynthesis

R
‰ The process by which green plant manufacture their food (carbohydrates) from simple inorganic raw materials
(present in the surrounding) in the presence of sunlight and chlorophyll is called photosynthesis. Oxygen gas is

A
released during the process, carbohydrates are utilized for providing energy to the plants.

M
‰ The carbohydrates which are not used up immediately are stored in the form of starch in plants.

U
This process takes place in the leaves of plants.
The overall equation of photosynthesis is
K
T
6CO 2 + 12H 2 O + Light energy 
Chlorophyll
→ C6 H12 O6 + 6O 2 + 6H 2 O
Carbon dioxide Water
N

Within a leaf, photosynthesis occurs particularly in specialized cell called mesophyll cells. These cells contain chloroplast.
A
SH

Raw Materials Required in Photosynthesis


‰ Carbon dioxide: The carbon dioxide is acquired from the plants via the atmospheric air. In terrestrial plants, the CO2
A

enters into the cells of leaves with the help of tiny pores called stomata which are present on the surface of leaves.
PR

‰ Chlorophyll: Chlorophyll is a green colour pigment essential for photosynthesis. It is present in the chloroplast.
Chlorophyll is responsible for absorbing light energy. Photosynthetic pigments are present in all the green parts of
the plant that help in the absorption of energy from sunlight and makes it
R.

available for photosynthesis process to occur.


D

Photosynthetic pigments absorb only portions of visible light. You might


remember from your physics class that the visible light, if passed through a prism,
will show the seven colours. Chlorophyll absorbs red and blue wavelengths and
reflects the green wavelength, that is why chlorophyll appears green.
‰ Sunlight: Sunlight is a natural source of light for photosynthesis.
‰ Water: Plants need water for the process of photosynthesis. It is absorbed
from the roots of the plant from the soil by the process of osmosis.
Some mineral salts like nitrogen, phosphorus, iron, and magnesium required
by the plants are also transported to different parts of the plant along with
the water. Nitrogen is an essential element used for the synthesis of DNA Fig. 1: Green plants make their own food by
and proteins. photosynthesis

4 Class-X BIOLOGY PW
Activity-1
Aim: To understand that carbon dioxide is an
important component of photosynthesis.
Bell jar
Method: Take two potted plants which have been
destarched by keeping them in the dark for around Potted plant

three days. Place them on glass slabs. Place a


watch glass comprising potassium hydroxide
solution on one slab. Place an inverted bell jar KOH
crystals in
over each potted plant. Seal the edges of bell jars petri dish
Bell jar with Bell jar without
by vaseline. Place the two sets in the presence of KOH KOH
Fig. 2: Experimental Set-up (A) with Potassium Hydroxide
sunlight. After two hours pluck one leaf from each
(B) without Potassium Hydroxide
pot and test these leaves for starch.

R
Observation: The leaf of set B turns blue-black because there is no potassium hydroxide solution, whereas the leaf

A
of set A with potassium hydroxide solution remains pale-coloured.

M
Conclusion: The leaf of set B, which appear blue-black, has synthesized starch via photosynthesis. Its bell jar
comprises carbon dioxide, as there is no potassium hydroxide. The leaf of set A has not produced starch, as

U
photosynthesis did not occur. The bell jar of this set does not comprise any carbon dioxide as the same is absorbed

K
by potassium hydroxide solution. Thus, carbon dioxide is essential for photosynthesis.
T
N
A
SH

Activity-2
A

Aim: To understand that chlorophyll is essential for photosynthesis.


PR

Method: Destarch a potted Croton or Money Plant with variegated leaves (comprising both green and non-green
parts) by keeping it in complete darkness for 2-3 days. Expose the destarched potted plant to sunlight for six hours.
Then, pluck a variegated leaf and place a sheet of paper over it. Draw the outline of green and non-green areas. The
R.

green areas contain chlorophyll. The non-green areas are pale in colour and devoid of chlorophyll. Place the leaf in
D

boiling water for 5-10 minutes. Dip the leaf in spirit or alcohol kept at 50°– 60° C with the help of a water bath.
After 30 – 45 minutes, the leaf will be decolourised completely. Take out the decolourised leaf, dip in hot water to
soften the same. Spread the leaf in petri dish. Pour dilute iodine solution over the leaf. After 4-5 minutes, rinse the
excess iodine and observe.
Observation: The leaf has two kinds of patches, bluish-black and yellowish. The bluish-black areas are the ones
which consist of starch. The bluish-black colour is because of the reaction of iodine with starch. The yellow areas are
without starch. Compare the bluish-black and pale areas with green and non-green areas sketched on paper. Bluish
black areas are the ones which were green previously, whereas non-green areas remain pale-coloured.
Conclusion: Only chlorophyll containing areas secrete starch which is a product of photosynthesis. Therefore,
chlorophyll is very important for the process of photosynthesis to occur.

Life Processes 5
Activity-3
Aim: To understand that sunlight is essential for photosynthesis.
Method: Fold a circular piece of black paper and cut a design in its centre. Place black paper or tin foil, with the
cut design in its centre on the surface of a leaf of destarched potted plant through cellotape clips or strips.
Place the plant in sunlight for 2-3 hours. Take the black paper or Ganong’s light screen. Pluck the leaf. Test this leaf
for the presence of starch
Observation: Covered part of the leaf remains yellow, whereas parts of the leaf which received light turn bluish-
black. Bluish black colour shows the presence of starch.
Conclusion: Only that leaf shows photosynthesis or a positive starch test, which is exposed to light. As the covered
part did not receive sunlight, thus it did not perform photosynthesis which is evident from the absence of starch.
Therefore, sunlight is necessary for photosynthesis.

Site of Photosynthesis

R
‰ The leaves contain specialised cells called mesophyll cells which

A
contain chloroplasts (the pigment containing organelle). These

M
are the actual sites for photosynthesis.
‰ The photosynthesis pigment, chlorophyll, is the principal pigment

U
involved in photosynthesis. It is a large magnesium containing

K
molecule and absorbs violet, blue and red region light of the visible
spectrum and reflects green light and thus, appear green in colour.
T
‰ The role of the pigments is to absorb light energy, thereby con-
N

verting it to chemical energy. These pigments are located on the


A

thylakoid membrances of chloroplast.


SH

Stomata
Stomata are tiny pores or microscopic apertures present on the
A

Fig. 3: Cross Section of Leaf


epidermis of leaves guarded by two bean or kidney-shaped guard cells.
PR

Role of Stomata
‰ A large amount of gaseous exchange occurs in the leaves
R.

through stomata. The exchange of gases also takes place


across the surface of root, leaves and stem.
D

‰ Transpiration occurs through the stomata. A large amount


of water is lost in the form of vapours through stomata.

Closing and opening of stomata


‰ The opening and closing of stomata depends on turgor (a) (b)

pressure, caused by the osmotic flow of water in the guard Fig. 4: (A) Open and (B) Closed Stomatal Pore
cells. When guard cells are turgid, they expand resulting
in the opening of stomata. When the guard cells loose water, they become flaccid leading to stomatal closure.
‰ The stomata normally opens when the light strikes the leaf and are closed during the night but in desert plants,
they open at night time to check excessive loss of water.
In aquatic plants, CO2 is obtained from the water, where it remains present in dissolved form. Such plants absorb
carbon dioxide in solution form through diffusion all over their surface from the surrounding water.

6 Class-X BIOLOGY PW
Mechanism of Photosynthesis
Light reaction
(a) During light reaction, the photosynthetic pigments absorb light energy in the form of photons.
(b) Chlorophyll molecule gets excited and emits electrons.
(c) The emitted electrons travel through the electron transport chain present in the chloroplast. This process is mediated
by a number of electron acceptors.
(d) ATP is synthesized from ADP and inorganic phosphate, which is used as a source of energy during dark reaction.
(e) In light reactions, photolysis (breakdown of water in the presence of light) of water also takes place:
H2O → 2H+ + O2 + 2e–
(f) The released hydrogen ions reduce the NADP molecule into NADPH, which is also used up during dark reaction.

Dark reaction
(a) The NADPH molecules, as well as the ATP produced during the light reaction, are utilized in the stroma of the
chloroplast for synthesis of carbohydrate from carbon dioxide. This process is called dark reaction.

R
(b) In the process, carbon dioxide enters into a cycle of reactions, starting from ribulose bisphosphate.

A
(c) At the end of the cycle, carbohydrate is synthesis and ribulose bisphosphate is regenerated.

M
U
K
T
N
A
SH
A
PR

Fig. 5: Mechanism of Photosynthesis

Difference between Light and Dark reactions


R.

Features Light reaction Dark reaction


Requirement of light Required Not required
D

Takes place inside Grana part of chloroplast Stroma region of chloroplast.


ATP and NADPH2 are produced by ATP and NADPH2 formed during
ATP and NADPH2 the conversion of light energy into light reaction are used for the fixation
chemical energy. of CO2 into carbohydrates.
Sugar formation No sugar formation takes place. Sugar formation takes place.
Release of oxygen Oxygen is released. No oxygen in released.

Significance of Photosynthesis
‰ Photosynthesis is an anabolic process (simple compounds are used to make complex compounds- building up reaction)
which provides food to the autotrophs. These autotrophs sustain all most major food chains. e.g., Plants are eaten by
herbivores, herbivores by carnivores and so on.
‰ It maintains CO2 and O2 concentration in the atmosphere.

Life Processes 7
How Do Organisms Obtain Their Nutrition?
Nutrition in Amoeba
(a) Ingestion: When Amoeba comes near food particles, it makes a
pseudopodia, (false feet). It Nucleus Food particle engulfs the prey by
food cup. This process is phagocytosis (if the amoeba is ingesting a food
particle) and pinocytosis (if it is ingesting liquid droplets). At the end of
this the food remains in a food vacuole.
(b) Digestion: The food vacuole act as a temporary stomach when a
lysosome fuses with it releasing digestive enzymes. Inside the food
vacuole, complex substances are broken down into simpler substances.
(c) Absorption: The digested food gets absorbed and diffuses into the
cytoplasm and then assimilated.
(d) Assimilation: Assimilation is the utilization of digested food in various Fig. 6: Nutrition in Amoeba
anabolic and catabolic reaction conversion (resynthesis) of absorbed

R
food into complex molecules for various purpose such as production of energy growth, repair and storage.

A
(e) Egestion: The remaining undigested food is passed to the cell’s surface and expelled out.

M
Nutrition in Paramecium
In Paramecium, which is also a unicellular organism, the cell has a definite shape and food is taken in at a specific spot.

U
Food is moved to this spot by the movement of cilia which cover the entire surface of the cell.

Nutrition in Human Beings


K
T
N
The human digestive system consists of an alimentary canal and the associated digestive glands.
A

The digestive tract or alimentary canal extends from mouth to anus. It is a muscular-coiled tubular structure. The food
passes through the different structures of the alimentary tract and is
SH

broken down into simpler absorbable forms. These various structures


of digestive tract in an order are:
A

Mouth → Oesophagus → Stomach → Small Intestine → Large


PR

Intestine → Rectum → Anus

Mouth
R.

‰ Mouth: Mouth is a transverse aperture bounded by movable lips.


It helps in taking the food into the buccal (mouth) cavity.
D

 Buccal cavity: In buccal cavity, three pairs of salivary glands


are present and on the floor of the cavity, a tongue is present.
The roof of the mouth is formed by the palate, which separates
the air channel from the food channel. The cavity is supported
by upper and lower jaws. Each tooth is embedded in a socket
of jaw bone. This type of attachment is called thecodont.
(i) Teeth: They are hard structures that are specially
developed for efficient mastication, i.e., mechanically
breaking the ingested food into smaller pieces. Permanent
teeth in each jaw includes 4 incisors (used for biting),
2 canines (us molars (used for grinding).
(ii) Tongue: It is a muscular, sensory organ that bears taste Fig. 7: Human Digestive System
buds with saliva and swallowing the food.

8 Class-X BIOLOGY PW
Pharynx
Buccal cavity opens into pharynx. It is the common passage for food and air. The oesophagus and the trachea (wind
pipe) open into pharynx. A cartilaginour flap called eipgloth's prevents the entry of food grottia (opening of windpipe)
during swallowing.

Oesophagus
Oesophagus is a long narrow muscular tube which serves to carry Area of
the food from pharynx to the stomach. It exhibits contraction and contraction

Food bolus
expansion movement of walls. i.e., peristaltic movement, so that
the partially digested food is pushed forward in the tract. It runs Area of
behind the trachea. relaxation

Stomach
It is a J-shaped muscular sac present on the left side of the upper

R
abdomen, beneath the diaphragm. Partially digested food reaches

A
into the stomach from the buccal cavity through the pharynx and Fig. 8: Peristaltic Movement
oesophagus.

M
‰ Mechanical Digestion in Stomach: The food is churned in the stomach for around three hours. In this phase, the food

U
breaks down to a semi-solid paste.

K
‰ Chemical Digestion in Stomach: There are glands called gastric glands in the stomach's walls which secrete the
following:
T
 Hydrochloric acid: This acid is secreted by the stomach during digestion. It destroys harmful microorganisms that
N
comes with the food particles. It activates the enzyme pepsin.
A

 Digestive enzymes: Stomach secretes pepsin in the inactive form. HCl activates this enzyme which then digests
proteins.
SH

 Mucous: Mucous is an aqueous secretion that is produced by the mucous membranes. It helps in protecting the
stomach lining and gastric pits from the acid.
A

Thus, this vital organ stores food temporarily and gives proper time to digest the food. The movement of food into the
PR

stomach and exit from the stomach is controlled by circular muscles called sphincters. They contract to close the opening
and relax to open them.
R.

Small Intestine
D

Small intestine is considered as the longest part of the alimentary canal (6.8m long and about 2.5 cm wide). The partially
digested and churned food from the stomach passes to the small intestine.
It consists of the following parts.
‰ Doudenum: It is C-shaped region into which the bile and pancreatic ducts open.
‰ Jejunum: It is the part next to the duodenum. It is a short region of small intestine.
‰ Ileum: It is the longest part of the small intestine.
‰ The small intestine is the main region for the absorption of digested food. Internally, the inner lining of the small
intestine is provided with long finger-like projections called villi, which increases the surface area and hence, enhance
the absorption capacity. At the base of villi, intestinal glands are present. These glands secreate digestive enzymes for
digestion of proteins, carbohydrates and fats. Small intestine completes digestion of proteins, cabohydrates and nucleic
acids. It also absorbs nutrients into blood and lymph.

Life Processes 9
Large Intestine
Large intestine is wider than small intesetine. It is about 1.5 to 1.8m long. It has four parts : caecum, colon, rectum and
appendix.
‰ The external opening of rectum is called anus which is kept closed by a ring of muscles called the anal sphincter.
‰ It opens only during defaecation.

Digestive Gland
There are many digestive glands associated with the digestive tract which are as follows:
‰ Salivary glands: Three pairs of salivary glands; parotid (cheek), submaxillary or submandibular (lower jaw) and
sublingual (below the tongue) are present in the mouth. These glands secrete saliva which performs two functions:
 Lubricates food and helps in swallowing.
 Saliva Contains an enzyme salivary amylase (previously known as ptyalin) which acts on starch.
‰ Pancreas: Lobed gland situated between stomach and
duodenum and secretes pancreatic juice which contains

R
Hydrophobic Hydrophilic
side side
three enzymes - lipase, trypsin and amylase which act on

A
fats, proteins and starch respectively. This gland also secretes

M
hormones - insulin and glucagon which regulate glucose
metabolism in the body.

U
‰ Liver: It is the largest gland of the body. It is dark-brown
and divided into two main lobes-right and left lobes. On the
undersurface of liver, gall bladder is present. Bile juice is
K Fig. 9: Emulsification of Lipids
T
made in the liver and is stored temporarily in the gall bladder. Bile is released into small intestine when food enters
N

in it. It contains no enzymes but contains bile salts (sodium glycocholate and sodium taurocholate) which help in
A

emulsification of fats
‰ Gastric glands: Gastric glands are present in the wall of stomach and secrete gastric juice.
SH

‰ Intestinal glands: These lie in the wall of small intestine and secrete intestinal juice or succus entericus.
A

Process of Nutrition in Human Beings


PR

‰ Ingestion: In human beings, food is ingested through the mouth and this process is known as ingestion.
‰ Digestion: Digestion of food begins in mouth.
R.

 The food ingested is chewed by teeth and broken down into smaller particles, so that large surface area is provided
for the action of enzymes. This food is mixed with saliva which is secreted by salivary glands which moistens and
D

lubricates the food and helps in swallowing. Also, the enzyme salivary amylase (ptyalin) acts on starch present in
the food and breaks it into maltose, a disaccharide, Now, the food is called bolus and is passed along the oesophagus
by peristalsis.
 As the food reaches the stomach, it is mixed with the gastric juice secreted by the gastric glands. The contents of the
stomach are churned by the action of muscles of the stomach. The gastric juice contains hydrochloric acid, mucus
and an enzyme pepsin. A small amount of gastric lipase is also present in the gastric juice.
™ Mucus protects the inner lining of the stomach from the action of HCl under normal condition.
™ HCl present in gastric juice kills the bacteria swallowed along with the food and makes the medium acidic for
activation of pepsin.
™ Pepsin gets activated in acidic medium and acts upon the proteins to convert them into peptones (an intermediary
product).
™ Gastric lipase partially breaks down lipids.

10 Class-X BIOLOGY PW
 In stomach, the food is churned by the muscular activity of the stomach and changes into chyme. The chyme enters
the duodenum which receives two juices: bile from the liver and pancreatic juice from the pancreas. Pancreatic juice
contains a number of enzymes which act in alkaline medium.
™ Trypsin: converts remaining proteins into peptones and the peptones into peptides and amino acids.
™ Amylase: converts the undigested starch into maltose (continues the process that begun in mouth).
 From the duodenum, the food is slowly moved down to ileum where more digestive changes come into action. The
intestinal juice called succus entericus secreted by intestinal glands contains the following enzymes which act in
alkaline medium:
™ Trypsin: converts peptones into amino acids,
™ Maltase: converts maltose (a disaccharide) into glucose (simple sugar),
™ Lactase: converts lactose into glucose and galactose;
™ Sucrase: converts sucrose into glucose and fructose.
™ Lipase: converts fats into fatty acids and glycerol,
™ Dipeptidase: converts dipeptides into amino acids.

R
 Thus, the digestion started in mouth is completed in the small intestine. The digested food consists of soluble

A
products- monosaccharides, amino acids, fatty acids and glycerol. These can be absorbed into the blood stream.
‰ Absorption: Absorption is the process by which the products of digestion are taken into the blood stream. Most of the

M
absorption of food material takes place in the small intestine. The absorption surface is greatly increased by the presence

U
of millions of finger-like projections called villi present in the small intestine.
‰ Assimilation: The absorbed food travels via blood to all the parts of the human body where they are fed to cells as they
K
diffuse out to the tissue space. Cells use respective molecules for their needs like glucose will be used to make ATP.
Amino acids will be used to make proteins. Broken fat components will be used to make/repair cell membranes.
T
The digested food which is not utilized by the body is immediately stored in the liver in the form of a carbohydrate
N

known as ‘glycogen’. When the body requires energy, then, the stored glycogen can be used.
A

‰ Egestion: The last area of the large intestine referred as ‘rectum’ stores this undigested food for a while. After this, this
SH

undigested food is egested or passed out from the body through anus as ‘stool’. The process of expelling the faeces is
known as defecation or egestion. The exit of faeces is regulated by the anal sphincter.
A

Ingestion Uptake of food


Mastication of food
PR

Swallowing/ Deglutition
Digestion Catabolic process
Break complex molecules to simpler ones
R.

� Proteins → Amino acid


� Carbohydrates → Glucose
D

� Lipids → Fatty acid + Glycerol


� Nucleic acid → Nucleoside + Nucleotide
Absorption Break monomer reaches blood vessels

Assimilation Absorbed food get utilized by body for growth and development

Egestion Unabsorbed food is removed from body


Fig. 10: Steps of Digestion process

Dental Caries
‰ Dental caries or tooth decay begins when bacteria acting on sugars produce acids food paritcles. The acids acts upon
the tooth and cause softening or demineralisation of enamel and dentine. The bacterial colony grows inside and forms a
dental plaque. It finally leads to inflammation and necrosis of pulp called pulpitis.
‰ Regular brushing the teeth after meals remove the plaque.

Life Processes 11
Respiration
Most living things require oxygen (or air) to get energy from food. The food material taken in during the process of
nutrition is used by cells to provide energy for various life processes. The process of releasing energy from food is called
respiration. Therefore, respiration is a biochemical process that generally involves:
(i) Intake of molecular oxygen from the environment.
(ii) Stepwise oxidation of food in the presence of oxygen.
(iii) Elimination of carbon dioxide produced during oxidation and
(iv) release of energy due to oxidation of food.

Types of Respiration
Respiration can be of two types:
(a) Aerobic Respiration: It is a process in which there is complete breakdown of food (glucose) into carbon dioxide and
water in the presence of oxygen and energy is released in the form of ATP.
Glycolysis oxygen
(1) Glucose (2) Pyruvate + energy CO2 + 6H2O + Energy [38 ATP]

R
Cytoplasm (In mito
chondria)

A
(b) Anaerobic Respiration: It is partial breakdown of food without using molecular oxygen. In this type of process,
incomplete oxidation of food takes place and in comparison to aerobic respiration, much less amount of energy is

M
produced. During this process, one molecule of glucose is converted into two molecules of pyruvic acid (pyruvate) and

U
little energy (2ATP) is produced. The term fermentation is more commonly used instead of anaerobic respiration. It
usually occurs in yeast and certain bacteria. In human beings, anaerobic respiration occurs skeletal muscles which do
K
not get sufficient oxygen and anaerobically metabolise glucose to lactic acid during vigorous exercise.
(i) Alcoholic fermentation: It is a kind of anaerobic respiration in which the microorganisms (yeast) break down
T
glucose into ethyl alcohol and carbon dioxide and energy (2 ATP) is released.
N

Glycolysis Absence of O2
(1) Glucose (2) Pyruvate (2) Ethanol + 2CO2 + Energy [2 ATP]
A

(In Cytoplasm) (In cytoplasm of


Yeast)
SH

(C6H12O6) (CH3COCOOH) (C2H5OH)


(6 carbon molecule) (3 carbon molecule) (2 Carbon molecule)
A

(ii) Anaerobic respiration in muscle cells (Lactic acid fermentation): Human muscle cells make ATP by lactic acid
fermentation, when there is scarcity of oxygen. This occurs at the time of vigorous exercise, when demand of oxygen
PR

by the muscles is greater than its supply. Therefore, the muscles switch from aerobic respiration to fermentation
resulting in the production of lactic acid. This lactic acid accumulates in the muscles causing cramps or muscle
fatigue.
R.

Glycolysis Absence of O2
(1) Glucose (In Cytoplasm)
(2) Pyruvate (In cytoplasm) (2) Lactic acid + Energy [2 ATP]
D

(C6H12O6) (CH3COCOOH) (C3H6O3)


(6 carbon molecule) (3 carbon molecule) (3 Carbon molecule)
Table-1: Differences between aerobic and anaerobic respiration

Features Anaerobic Respiration Aerobic Respiration


Location Cytoplasm only Cytoplasm and mitochondria
O2 requirement Not required O2 required
Breakdown Incomplete breakdown of glucose Complete breakdown of glucose takes
takes place place
End products CO2 and ethyl alcohol or lactic acid CO2 and H2O
Energy produced from one glucose 2 ATP 38 ATP
molecule

12 Class-X BIOLOGY PW
Absence of
oxygen Ethanol + carbon dioxide + energy
(in yeast) (2-carbon molecule)

In Pyruvate
Glucose Lack of oxygen Lactic acid + Energy
cytoplasm (3-carbon
(6-carbon (In our muscle (3-carbon molecule)
molecule)
molecule) cells)
+ energy
Presence of
oxygen
Carbon dioxide + Water + Energy
(in
mitochondria)
Fig. 11: Break-down of glucose by various pathways

Activity-4
Aim: To demonstrate that yeast, a microorganism, can produce CO2 by fermentation of sugar.

R
Method: Take some fruit juice or sugar solution in a test tube and add some yeast into it. Put a few drops of oil to

A
reduce the contact of external air. Cover the test tube with a one holed cork. Attach a bent glass tube in the cork and

M
dip the other end of tube into another test tube having lime water (solution of calcium hydroxide). Observe these
test tubes after few hours.

U
Observation and Conclusion: The lime water turns milky. This milky colour shows that CO2 is liberated from the

K
mixture of sugar and yeast. Fermentation of sugar results in the production of CO2 and ethyl alcohol.
T
N

Respiration in Plants
A

In plants, the exchange of gases occurs from stems, leaves and roots individually. Import and export of gases in plants are
SH

very slow and take place by simple diffusion. Plants have a branched shape, thus, they have quite a large surface area as
compared to their volume. Leaves are present on each branch, absorbing gases. Being composed of mostly dead tissues,
A

their energy needs are also less. Therefore, diffusion alone can fulfill their needs and plants do not need specialized organs
PR

like humans. Respiratory roles in plants are served by:


‰ By young roots: Air is found in soil interspaces. Root hairs are in contact with them. The oxygen of the soil air diffuses
via root hair and reaches all internal cells of the young root. Carbon dioxide produced by root cells diffuses in the
R.

opposite direction.
D

‰ By Stems: In herbaceous plants, the stem consist of small openings in their epidermal cells known as stomata, the
oxygen from air enters through stomata and carbon dioxide is released alternatively. In hard and woody stems of big
plants and trees, lenticels (cracks in the bark) are present in place of stomata by which the exchange of gases occurs.

(a) (b)
Fig. 12: (a) Lenticels on the Bark of the Tree, (b) Lenticel Inner Structure

Life Processes 13
‰ By leaves: The epidermis of leaves has various aerating pores called stomata (singular stoma or stomata, Gk. stoma-
mouth). Each stomatal pore or aerating pore is enclosed in a pair of guard cells. In most of plants, the guard cells are
bean or kidney- shaped with thicker inner walls and less elastic than the outer walls.
‰ Trees of coastal swamp regions like mangroves have adapted roots
called pneumatophores to breathe since soil in swampy areas is not
aerated. Aerial roots have small opening (pores) for exchanging gases
required for respiration. The apical region of pneumatophores has
lenticels for the exchange of gases.
At the day time, when photosynthesis take place, oxygen is produced.
The leaves use some of this oxygen for respiration, and the rest of oxygen
diffuses into the air. Carbon dioxide produced through respiration is all
used up in photosynthesis via leaves. Even more carbon dioxide is taken
in from air. Thus, the net gaseous exchange in leaves during dayt i m e
is: CO2 diffuses in; O2 diffuses out.
Fig. 13: Pneumatophores in mangroves for gaseous
At night, since no photosynthesis takes place, no oxygen is produced, exchange
oxygen from air diffuses into leaves to perform respiration. And carbon

R
dioxide produced through respiration diffuses out into air. So, the net gaseous exchange in leaves at night is: CO2

A
diffuses out; O2 diffuses in.
‰ By Lenticels: Lenticel consists of cells with large intercellular spaces. They allow the exchange of gases between the

M
outer atmosphere and the internal tissue of the stem. These are found in most woody trees.

U
Respiration in Animals
K
T
Mechanisms of breathing vary among different group of animals depending mainly on their habitats and levels of
N
organisation.
Unicellular organisms use their plasma membrane to allow the diffusion of gases in the cytoplasm. In simplest animal
A

phyla, gaseous exchange takes place from their general body surface. Members of phylum Plathyhelminthes to Nematodes,
SH

are mostly anaerobic and parasites in Phylum Annelida, cutaneous (through the skin) respiration takes place and then
from Phylum Arthropoda to Mammals, various respiratory organs were developed like tracheal tubes, gills and lungs.
A

Table-2: Types of respiration


PR

Types of respiration Definition Example


1. Branchial respiration Respiration through gills Fishes
R.

2. Cell surface respiration Respiration through general body surface Amoeba, Paramecium
3. Pulmonary respiration Respiration through lungs Amphibians, reptiles, birds
D

4. Tracheal respiration Respiration through tracheal tubes Insects


5. Cutaneous respiration Respiration through cuticle moist (skin) Frogs, earthworms
Some important features of the respiratory organs of animals are as follows:
‰ Large surface area to get enough amount of oxygen.
‰ Thin walls for easy and simple diffusion and exchange of gases.
‰ Rich blood supply for transport of respiratory gases.
‰ Respiratory surface must allow transfer of respiratory gases.
Respiration in Fish: In fish, exchange of gases take place through gills so the respiration is considered as branchial. Gills
are present on both the sides of its head, surrounded by gill covers. During breathing, fish takes in water through its mouth
and pass it over the gills, the oxygen present in water is taken up by gills and water is removed out through gill slits.

14 Class-X BIOLOGY PW
This oxygen is now absorbed through blood and carried to all Gill capillaries
other parts of the body and at the same time, carbon dioxide is
released into the blood and comese back to the gills and is
expelled out into the surrounding water. Same type of respiratory Artery
Key
pattern is followed in other aquatic organisms such as prawns. Oxygen-rich blood
Heart: Oxygen-poor blood
Respiration in Amoeba: In unicellular organisms like Amoeba Atrium (A)
and in some lower multicellular animals like cnidarians and Ventricle (V)
sponges, respiration or exchange of gases takes place through Vein
general body surface as these cells are in direct contact with an
aquatic region, so the oxygen dissolved in water diffuses into the Body capillaries
cell and brings about oxidation of food, at the same time carbon
dioxide released is expelled out of the cell by the same process. Fig.14:
Fig. 7:Single
Singlecirculation:
circulation:Fish
Fish

Respiration in Earthworm: In organisms like earthworm and leech, exchange of gases takes place through their skin
as their skin is moist and thin. It is rich in blood supply, so the oxygen is absorbed via moist skin of earthworm and is
transported to all the cells of body via blood. The carbon dioxide from body cells diffuses into the blood and expelled
out from skin.

R
A
Respiration in Human Beings

M
In humans, lungs are the primary organs of the respiratory system, which aids in the exchange of gases.

U
The respiratory system in humans comprises of nostrils, nasal cavity, trachea, pharynx, bronchi, bronchioles and alveoli
in the lungs. K
T
‰ External nares or nostrils: They are pair of slits at the lower end of nose. It opens into the nasal cavity and is meant
N
for inhalation of air from the outside surroundings.
A

‰ Nostrils opens into nasal cavity.


‰ Nasal cavity: It is separated from oral cavity with the hard and bony palate. This opens into pharynx (commonly called
SH

throat).
Epiglottis
‰ Pharynx: It leads into short tubular chamber called larynx by a
A

Larynx
slit like apeture called glottis. Glottis always remains open but Trachea
PR

get closed during swallowing by a leaf like cartilaginous structure


epiglottis at in anterior margin to prevent entry of food in the Bronchus
trachea. Pleural
R.

Cut end
of rib heart membranes
‰ Larynx: Two vocal cards are present in larynx. Vibration in vocal Alveoli
D

Lung Pleural fluid


card results in production of sound by expelled air. Larynx is Bronchiole
Diaphragm
an enlarged part of the trachea called the ‘voice box’. Larynx is
followed by thin walled tube called trachea or wind pipe. It extends Fig.
Fig.3:
15:Diagrammatic
Diagrammaticview
view of human respiratory
respiratorysystem
system
downwards through the neck. (sectional
(sectionalview
view of the left lung
lung is
is also
also shown)
shown)

‰ Trachea: The trachea or wind pipe is a thin walled tube that extends downwards. It is 10-12 cm long tube. The wall of
treachea has incomplete cartilaginous rings (c-shaped) which prevent the trachea from collapsing.
‰ The part from external nostrils upto the terminal bronchioles constitute the conducting part whereas the alveoli and their
ducts from respiratory or exchange part of respiratory system. Conducting part transports the atmospheric air to alveoli,
clears it from foreign particles, humidifies and brings the air to body temperature.
‰ Each individual has a pair of lungs. The lungs lie in the thoracic cavity. The thoracic cavity is separated from the
abdominal cavity by a muscular partition called the Diaphragm.

Life Processes 15
‰ Each lung is enclosed in two membranes, the pleura. Within the lungs, the major bronchi further divide into secondary
bronchi which sub-divide into smalled tertiary bronchi and finally into still smaller bronchioles.
‰ Each bronchioles divides into alveolar ducts which enter the alveolar sacs. The alveolar sac are also called alveoli.

‰ Alveoli are the structural and functional unit of lungs as these are actual site of gaseous exchange.

‰ Alveoli have very thin walls composed of simple moise non-ciliated squamous epithelium. It is closely surrounded by a
network of blood capillaries. Due to very intimate contact of blood capillaries with alveoli, the exchange of gases takes
place here.

Passage of Air through the Respiratory System


External nostrils → Nasal cavity → Pharynx → Larynx → Trachea → Bronchi → Bronchioles → Alveolar sacs →
Alveoli.

Smoking is injurious to health. Lung cancer is one of the common causes of death in the world. The upper part of

R
respiratory tract is provided with small hair like projections called cilia. Cilia helps to remove germs, dust and other

A
harmful particles from inhaled air.

M
U
Breathing Mechanism in Humans
K
The breathing mechanism mainly involves the movements of ribcage and diaphragm. This happens as follows:
T
Mechanism of Breathing
N
A

Inhalation Exhalation
SH

(i) Inhalation: When air is breathed in, the muscles and diaphragm attached to the ribs contract, thus, ribs move outward
A

and upwards and diaphragm moves downward. This causes expansion of chest cavity and it results in increase in volume
of chest cavity. Thus, the air pressure decreases and air from outside rushes into the lungs and alveolar sacs get filled with
PR

air comprising oxygen. The oxygen present in air diffuses into the blood and CO2 from blood diffuse out into alveolar sac.
(ii) Exhalation: When air is breathed out, the diaphragm and muscles attached to ribs relax. Thus, ribs move inward and
R.

downwards and diaphragm move upward (forms dome shaped structure) which brings out contraction in chest cavity, its
D

volume get decreased. This leads to an increase in intra-pulmonary pressure to slighly above the atmospheric pressure
causing the expulsion of air from the lungs, i.e., expiration.

Fig.
Fig.4:16:
Mechanism
Mechanismofofbreathing showing: Inspiration
breathingshowing: Inspiration Fig.
Fig.5: Mechanism
17: Mechanismof
ofbreathing
breathing showing: Expiration
showing: Inspiration

16 Class-X BIOLOGY PW
Breathing takes place involuntarily. The breathing rate, however, is regulated by the brain’s respiratory centre. A
moderate breathing rate in a normal adult man is around 15 to 20 times per minute. Residual volume (RV) is the
amount of air remaining in the lungs at the end of a maximum expiration.

Transportation of Gases
Transport of Gases: Blood is the medium of transport for O2 and CO2.
‰ Transport of oxygen in the blood: Oxygen is carried by the blood from the lungs to the heart and from the heart to the
various parts of the body. Oxygen is transported in the following ways:
(i) As Oxyhaemoglobin: 97% of the oxygen is carried by the haemoglobin of the erythrocytes in the form of
oxyhaemoglobin.
(ii) As dissolved gas: 3% of the oxygen gets dissolved in the blood plasma and is carried to the different parts of the
body.
‰ Transport of CO2 in the blood: After cellular respiration, carbon dioxide, water and energy are produced. Carbon
dioxide can be transported in three different ways:
(i) Transport of CO2 as

R
Hemoglobing Oxygen molecule
bicarbonate form: 70 % of

A
molecule
carbon dioxide is converted into

M
bicarbonate ions and transported
via plasma. Carbon dioxide also

U
diffuses into the RBCs, and Hemoglobin binds to oxygen Red blood cells transport When red blood cells
combines with water, forming
carbonic acid (H2CO3), which
as red blood cells travel
through the lungs.
K
circulating oxygen to organs
where they need them.
Fig. 18: Oxygen translocation
reach their target,
hemoglobin releases oxygen.
T
dissociate into bicarbonate ions
N
(HCO3–) and hydrogen ions(H+). This reaction occurs in the presence of enzyme carbonic anhydrase.
(ii) Transport of CO2 as Carbaminohaemoglobin: Carbon dioxide reacts with amine radicals (NH2) of haemoglobin
A

to form unstable compound carbaminohaemoglobin (HbCO2). About 20-25% of CO2 is carried by haemoglobin as
SH

carbaminohaemoglobin.
(iii) Transport of CO2 in dissolved form: CO2 has high solubility, so 7% of carbon dioxide is dissolved in blood plasma
A

and transported to the lungs.


PR

Air moves in and out


blood low in oxygen, of alveolus
high in CO2 blood low in CO2
high is Oxygen
R.

Capillary wall
Alveolus wall
D

CO2 diffuses from Oxygen diffuses


blood to be exhaled into blood

Red blood Cells Oxygen is tansported


around body by red
blood cells

Gases dissolve in
moist mucus lining
Fig. 19: Levels of gaseous exchange in humans

Life Processes 17
Gaseous Exchange between the Alveoli and Tissues
The alveoli is lined by very thin blood vessels referred to as
Air
capillaries which carries blood in them. So, the oxygen of air
diffuses out from the alveoli walls into the blood. The oxygen Basement
Alveolar wall
is carried through blood to all the parts of the body. This substance
(one-celled thick)
oxygen is carried by a red coloured, iron containing respiratory Endothelium of
pigment known as haemoglobin present in red blood cells. As Alveolar cavity
blood capillary
the blood passes through the tissues of the body, the oxygen
present in it, diffuses into the cells. This take place due to its
higher concentration in the blood. This oxygen reacts with the Blood
capillary Red blood
glucose present in the cells to generate energy. Carbon dioxide cell
gas is produced as a waste product during respiration in the Fig. 20: A diagram of a section an alveolus with a
Fig. 8: A diagram of a section of an alveolus
cells of the body tissues. This carbon dioxide diffuses into the pulmonary capillary
with a pulmonary capillary
blood (due to its higher concentration in body tissues). Blood carries the carbon dioxide back to the lungs where it diffuses
into the alveoli. When we breathe out air, the diaphragm and the muscles attached to the ribs relax and as a result, the

R
chest cavity contracts and becomes smaller. This contraction of the chest pushes carbon dioxide out from the alveoli of

A
the lungs into the trachea, nostrils and then out of the body into air. By this, gaseous exchange completes in the human

M
respiratory system.

U
Respiration and Breathing

K
Breathing is simply intake of fresh air (oxygen from atmosphere) and removal of foul air (CO2 produced by the cells) from body.
Table-3: Dissimilarities between respiration and breathing
T
Respiration Breathing
N

It is the process of breaking down of glucose to generate energy, It involves the process of inhaling oxygen and exhaling
A

which is then utilized by cells to perform other activities. carbon dioxide.


SH

Respiration is an involuntary biochemical process. Breathing is an involuntary as well as a voluntary physical


process. (For example, breathing during sleep is involuntary.
A

Voluntary breathing is mainly seen when we sing, swim,


PR

speak or for relaxation techniques).


Respiration occurs at the cellular level. Breathing takes place through respiratory organs, including
the nose, lungs, etc.
R.

Various enzymes are used in this process. No enzyme is used in this process.
D

Energy is produced and released in the form of ATP. Energy is not produced.
As it take place inside cells, it is called the intracellular As it take place outside the cells, it is called the extracellular process.
process. (takes place between the organism and the external environment).

Difference between Combustion and Respiration


Table-4: Dissimilarities between Combustion and respiration

Combustion Respiration
It is not a cellular process. It is a cellular process.
It is an uncontrolled physio-chemical process. It is biochemical process under biological controlled.
Energy is released in a single step of reaction. Energy is released in stages as chemical bonds are broken down step-wise.
It is a non-enzymatic process. Different enzymes are required, one for each step.

18 Class-X BIOLOGY PW
Activity-5
Aim: To show that carbon dioxide is produced during respiration.
Method: Take two test tubes (a) and (b). Pour some freshly prepared Pichkari
Pichkari

lime water in each tube. Now, blow air in tube (a) through the lime
water with the help of a glass tube and observe. Pass air through lime
Rubber
Rubber Tube
Tube
water in test tube (b) with the help of a syringe and observe. tube
tube

Observation and Conclusion: The lime water in tube (a) turns milky
Test tube
Test tube containing
containing
showing that we exhale carbon dioxide. On the other hand, the lime Lime
Lime lime water
lime water
water
water
water in tube (b) takes a long time to turn milky. The lime water is
a solution of calcium hydroxide, it combines with CO2 to form a (a) (b)

Fig. 21: (a) Air being passed into lime water


white precipitate of calcium carbonate. This also shows that expired
with a pichkari/ syringe, (b) air being exhaled
air contains more CO2 concentration than the atmospheric air (0.04%) into lime water

R
A
Transportation

M
In unicellular organisms, a single cell carries out all the life processes.

U
As explained previously, unicellular living organisms can transport substances using diffusion. But multicellular organisms
K
need effective mechanisms for long-distance transport as diffusion is a slow process.
T
To sustain life, living organisms, either plant or animal require water, oxygen and food in every cell of its different tissue.
N
The process of transportation is basically defined as transportation of material from one region to another, mainly from
the region of their availability to the region of their usage, storage or elimination.
A

In animals, the transport of digested food, oxygen, hormones, and excretory products to particular organs and tissues
SH

is done by a well-defined organ system referred to as the circulatory system. The circulatory system consists of blood,
heart and blood vessels. Also, in higher plants, transport of minerals with water and food materials is performed by a
A

well-defined vascular system having conducting tissues, xylem and phloem respectively.
PR

Transportation
R.

Transportation in animals Transportation in plants


also known as circulation occur in two ways
consists of two systems
D

1. B
 lood circulatory 1. Ascent of sap
system
2. Lymphatic system 2. Translocation

Transportation in Plants
The transport of minerals, hormones,food, water and other important substances from one region to another region inside
the body of an organisms is referred as transportation.
Plants take in CO2, photosynthesize and store energy in the leaves. Besides CO2, plants require the other raw materials
for growth. These raw materials such as phosphorous, nitrogen and other minerals are acquired from the soil via the roots.
These raw materials are required to be transported to each and every region of the plant, mainly to the leaves.

Life Processes 19
If the distance between roots (soil-containing organ) and leaves (chlorophyll-containing organ) is small, then these raw
materials can easily reach to the leaves through the process of diffusion. If the distance between leaves and roots is large,
then, proper system of transportation is required. In plants, transportation is done with the help of a specialised vascular
system which is present in the root, stem and leaves.
Vascular system is composed of two types of vascular tissues: Xylem and phloem.
In plants the transport of materials can be divided into two parts:
(1) Transport of minerals and water by xylem tissue.
(2) Transport of food and other substances by phloem tissue.

(I) Transportation of Water and Minerals in Plants (Ascent of Sap) /Dixon-Jolly Theory
Plants need water and minerals to make food and performing other Nucleus
Nucleus
vital functions. The water and minerals are absorbed via land plants Cytoplasm
Cytoplasm Vacuole
Vacuole

from the soil by the roots where it is available in the form of the Cellwallwall
Cell

soil solution.
‰ Root hairs are the main organs for the absorption of water and

R
minerals. Root
Root hair
hair

PLANT
‰ They are directly attached with the film of water in between the
PLANT

A
soil particles.

M
‰ Water passes into the root hairs by passive transport (osmosis). Water and
Water andMinerals
Minerals

At the roots, cells are in direct contact with soil and take up ions

U
by active process. This creates a difference in concentration
of these ions between the root and the soil. Water, therefore,
moves into the root from the soil. K
T
‰ Hence, there is a steady movement of water into the root. ROOTS
ROOTS
N
Soil particles
Soil particles
Water now has to be transported upward to the other regions of the
A

Fig. 22: Fine root hair strands and water transport


plant. This upward transport of water is called the ascent of sap. It
is called the sap as it consists of various dissolved minerals.
SH

Ascent of sap is performed by xylem tissue which comprises of:


(i) Xylem fibres
A

(ii) Xylem vessels


PR

(iii) Xylem parenchyma


(iv) Xylem tracheids
R.

In flowering plants, xylem vessels (wide, dead, tubular cells) and tracheids (slender, dead, spindle shaped cells) conduct
water and minerals in the upward direction whereas, in non-flowering plants, vessels are not present, tracheids are the
D

only conducting cells. Xylem fibers have supportive roles whereas xylem parenchyma allows horizontal conduction of
water. In xylem tissue, vessels and tracheid of the roots, leaves and stems are interlinked to form a continuous system of
water-conducting channels, just like pipes, reaching all parts of the plants. Water lost by
transpiration
Forces responsible for the Ascent of Sap
Root pressure
Suction
Root pressure is a pressure in which water push is in upward direction is pressure Capillarity
with the help of system of roots. This pressure is developed in the xylem
due to the absorption of water in roots. While it can explain the transport
of water to the leaves and other parts of short plants like herbaceous Water absorbed
plants but in tall trees, the root pressure is not enough to transport the by root hairs
water up to the leaves. In these trees, transpiration pull help in t h e
transportation of water. Fig. 23: Ascent of sap

20 Class-X BIOLOGY PW
Transpiration
‰ The loss of water in the form of water vapours from the aerial parts of plant is known as transpiration.
‰ Transpiration mainly occurs through stomata (about 80% to 90%) but it may also occur through cuticle (9%) and
lenticels (1%).
‰ More transpiration occurs from the lower surface of dorsiventral leaf as compared to upper surface. This is because of
the fact that more stomatal openings are present on the lower surface.

Functions of Transpiration
‰ It helps in absorption and upward movement of water.
‰ It helps in temperature regulation. The plants are protected from the burning due to transpiration. Evaporation of water
through stomata produces cooling effect.

Activity-6
Aim: To demonstrate that the transpiration occurs from aerial parts of the plants.

R
Apparatus: A small potted plant of the same size and the amount of soil as that of a potted plant, a stick of the

A
height of the plant, and plastic sheets.

M
Procedure: Cover the soil of two pots with plastic sheets. Place a stick in the middle area of the pot which does
not have the plant. Cover both the pots with plastic sheets and place them in bright sunlight for around half an hour.

U
Observation: The internal surface of the plastic sheet belonging to the set having potted plant turns wet. There is
no such change in the second set.
K
Conclusion: The wetness of the plastic sheet must have occurred due to the condensation of water vapours. Water
T
vapours can come only from aerial shoot of the potted plant. The soil does not lose any water in evaporation as it is
N

clear from parts the second set. Loss of water from aerial of the plant in vapour form is called transpiration.
A

(2) Transportation of Food and Other Substances in Plants


SH

Amino acids, sugars and other substances are translocated from site of synthesis to the site of storage via the phloem.
Transport of food from the leaves to the distinct parts of the plant is called translocation. Translocation may be in
A

downward or upward direction according to the need of the plant. Translocation of food
PR

Sieve tube
occurs in the phloem tissue. element

Phloem tissue contains four components: Source


cell
R.

(i) Phloem parenchyma


(ii) Sieve tubes / sieve elements
D

(iii) Phloem fibres


(iv) Companion cells
Companion
Seive tubes are chief conducting elements placed like pipes. cell

Xylem
Mechanism of translocation
‰ Translocation of food in the phloem occurs with the consumption of energy in the form of
ATP. Sugar (sucrose) formed in leaves are loaded into the sieve tubes of phloem by using
ATP energy. Sink cell

‰ This increases the solute concentration inside the sieve tubes.


‰ Now, water enters the sieve tubes through the process of osmosis. This creates
pressure which drives the sap (cytoplasm laden with sugars) across to the next sieve
tube. Fig. 24: Translocation

Life Processes 21
‰ This keeps on until the sap reaches the sink (storage site/ site of need)
‰ Energy is consumed to unload the sap.

Thus, the material is translocated from higher osmotic pressure to the lower osmotic pressure areas according to the need
of plant.For example, in the spring season, sugar stored in the stem or root tissue would be transported to the buds, which
require energy for their growth.

Transportation in Animals
In animals, the process of transportation of the absorbed food, water, gases and waste products from one region to the
another in known as circulation. Body fluids, like blood, go around the entire body acquiring and distributing substances
according to their respective gradients. It is performed by various types of circulatory system, which in humans is
classified into two system namely:
Lymphatic System Blood Circulatory system
(i) Lymph (i) Blood

R
A
(ii) Lymph vessels (ii) Blood vessels
(iii) Lymph nodes (iii) Heart

M
U
Circulatory System

K
When multicellular organisms need a circulatory system to transport oxygen and nutrients throughout their body, this
process is known as Circulation. Blood, nutrients, gases, and other molecules are transported throughout the body by the
T
circulatory system.
N

There are two types of circulatory systems namely:


A

(i) The open circulatory system: The system is primarily found in invertebrates. Here, the blood flows freely through
SH

cavities and there are no vessels to conduct the blood. The organs and tissues are directly bathed in blood. As a result,
there is no differentiation between blood and interstitial fluid.
(ii) Closed circulatory system: This is the system that is found in vertebrates and a few invertebrates like earthworms. This
A

system in the presence of vessels conducts blood throughout the body. The heart, which pumps blood into the vessels so
PR

that it can reach the tissues and organs, is part of a closed circulatory system. Humans have a closed circulatory system
that circulates blood throughout the body and a lymphatic system that circulates lymph.
R.

Human Circulatory System


D

Human circulatory system consists of three chief components: Blood (the fluid), the blood vessels (the pipes) and the
heart (the pump).

Blood
Blood is a fluid connective tissue that transports oxygen and nutrients to body cells and tissues.

Composition of blood
The major constituents of blood are:
1. Plasma
2. Red blood cells (RBCs) / Erythrocytes
3. Platelets / Thrombocytes
4. White blood cells (WBCs) / Leucocytes

22 Class-X BIOLOGY PW
Plasma is a viscous fluid containing around 55% of the blood. Blood clotting factors are also present in the plasma in
inactive form. Plasma without clotting factors is called serum. 90-92% of plasma is water. Essential types of proteins such
as globulins, albumins, and fibrinogen are present in the plasma. (Albumin and globulins retain water in blood plasma.)
Plasma also consists of small amount of minerals like Na+, Ca2+, Mg2+, HCO3–, Cl–
RBCs (Red Blood Cells) WBCs (White Blood Cells) Platelets
Has haemoglobin that carries Prevents leakage of blood by
Chief Function Provides immunity to the body
oxygen blood clotting.
Cell fragments product from
Cellular
Enucleated when mature Nucleated megakaryotes (special cells in
characteristics
the bone marrow).
Functions of Blood
Blood serves the following functions in the body:
(i) Hormones (chemical messengers) are carried by blood from the endocrine glands to the target organ.
(ii) Blood allows transport of O2 from the respiratory organs to the tissues and CO2 from the tissues to the respiratory organs.

R
(iii) Blood allows the transfer of heat from the deeper tissues to the surface of the body where it can be lost.

A
(iv) Blood transports the digested food from the alimentary canal to the body cells.

M
(v) The clotting factors present in the blood reduce the loss of blood from the area of injury by the formation of clot.

U
(vi) Blood allows the transport of excretory matter to the kidneys or other excretory wound organs.

K
(vii) Protect the body against pathogens because WBCs kill the bacteria and other germs which causes diseases.
Blood can also be classified according to certain characteristics of cells and plasma. In accidents involving blood loss,
T
patients need to be matched for the blood type before they can be transfused with the blood of a donor. Some people
N
can receive blood from any of the other blood types (Universal acceptor) whereas some can donate blood to other blood
A

types. (Universal donor)


SH

Blood Groups
In 1900, Karl Landsteiner discovered the blood groups.
A

Four types of blood groups found in human being are: A, B, AB and O. The blood group of an individual is inherited
from parents. According to the presence or absence of specific antigens in their red blood cells, and the presence or
PR

absence of specific antibodies in their plasma, the blood of all the human beings is categorized into four groups. These
are shown as below:
R.

Blood Group Antibodies Antigens (Present on


(Present in Plasma) Red blood Cells)
D

A anti-B A
B anti-A B
AB None A&B
O anti-A, B None
When we mix the blood of two people having different blood groups say A and B, then one blood comprises of antigens
A and antibody ‘b’ and another person has blood with antigen B and antibody ‘a’, now antibody ‘a’ and ‘b’ will coagulate
and blood will clot and person will eventually die. The clotting of blood is referred as agglutination.
Blood Transfusion
Blood is very important to keep a person alive. In some accidental conditions when a person loses excess of blood, it is
essential to give him blood from external source. The transfusion of blood from one individual to another individual is
known as blood transfusion.

Life Processes 23
The healthy person who donates blood is called donor and the sick person who receives blood is called recipient. The
blood transfusion should be done only if the type of blood of the two individuals matches. Blood transfusion can be done
safely by using the following table.
Blood Group Antibodies Antigens (Present on Red Donor's group
(Present in Plasma) blood Cells)
B anti-A B B, O
A anti-B A A, O
O anti-A, B nil O
AB nil A, B AB, A, B, O

Blood Vessels
There are three types of blood vessels in human circulatory system which carry blood.

1. Arteries

R
Arteries carry blood away from the heart and supply it to various organs.
These arteries consist of elastic thick muscular walls and are deeply

A
seated because the blood emerges from the heart under high pressure.

M
They carry oxygenated blood except pulmonary arteries which only
carry deoxygenated blood to lungs for purification. Valves are absent

U
as pressure from heart is enough to push blood to the arteries. Arteries
branch and narrow down into smaller arterioles.
K
T
Fig. 25: Blood vessels structure
2. Capillaries
N

Arterioles narrow down into smaller and the fine blood vessels, called as capillaries. Their walls are very thin and made
A

up of a single layer of cells. They are closely placed with the cells of the tissue in an organ. The material exchange
between the blood and surrounding cells occurs across this thin wall.
SH

3. Veins
A

Capillaries unite to form venules (smaller veins) which


PR

further unite to form veins. They collect blood from the


tissue of the organs and return it back to the heart through
two big veins namely the superior and inferior vena cava.
R.

Veins are thin-walled and are superficially present. They


carry deoxygenated blood except pulmonary veins which
D

carry oxygenated blood from the lungs to the heart. They


consist of valves which ensure that the blood flows only in
one particular direction as veins have low blood pressure.

Structure of Heart
Size: The size of heart is equal to the size of one’s clenched
fist.
Shape: Heart is triangular (conical) in shape and composed
of cardiac muscles.
Location: Heart is present in between lungs in a space called
as mediastinum. Fig. 26: Cross section of veins and artery

24 Class-X BIOLOGY PW
Protection
(i) Heart is covered from all sides by ribcage and vertebral column on dorsal side. They provide protection to the
heart.
(ii) Heart is enclosed in a double layered thin, transparent sac like structure known as pericardium.
 Heart is divided into two halves and each half consists of two chambers, upper chamber (Atria; sing: atrium)
and lower chambers (Ventricles; sing: ventricle). Hence, we have four chambered heart.
 Right and left halves are separated by a septum.
 Blood vessels:
™ Superior and inferior venacava supply
deoxygenated blood to the right atrium. Aorta
™ Pulmonary artery take deoxygenated blood
from right ventricle of the heart to the lungs for Vena cava Pulmonary artery
oxygenation/ purification. Sino-atrial node Pulmonary veins
Right atrium Left atrium
™ Pulmonary vein brings the oxygenated blood from
Atrio-ventricular
the lungs to the left atrium of the heart.

R
node Bundle of His
™ Left ventricle has the thickest muscular wall and
Chordae tendinae

A
aorta distributes oxygenated blood from the left
Right ventricle Left ventricle
ventricle of the heart to the body.

M
Interventricular
 Valves in the heart: septum

U
™ Between the right atrium and right ventricle have Apex
valve: the tricuspid valve (with 3 cusps)
™ Between the left atrium and left ventricle: bicuspid K Fig. 13:
Fig. 27: Section
Sectionofofaahuman
humanheart
heart
T
valve (with 2 cusps), also called as mitral valve
N
™ Two semi-lunar valves guard the opening of the pulmonary artery and aorta, which leave the right and left
ventricle, respectively.
A

 Valves in the heart ensure that the blood does not flow backwards when the atria or ventricles contract.
SH

 Human heart is myogenic, i.e., it can initiate its own contractions and relaxations, although brain and hormones can
regulate it.
A

Working of Human Heart


PR

1. It starts with the contraction of the two atria. The


ventricles at this time are relaxing and are empty.
Therefore, the blood from the atria passes into
R.

ventricles easily. oxygenated blood


D

2. Venacava, largest veins of the body brings deoxygenated deoxygenated blood to upper body
blood (blood deficient in oxygen as tissue has consumed from upper body
it) to the right atrium.
3. When the right atrium contracts, the deoxygenated
deoxygenated
blood is pushed into the right ventricle. blood to lungs
4. When the right ventricle contracts, blood is pushed oxygenated blood
into the pulmonary artery which delivers it to lungs for from lungs
oxygenation. (Pulmonary artery is an exception which oxygenated blood
from body
carries deoxygenated blood despite being an arterty).
5. Oxygenated blood returns to the left atrium via
pulmonary vein. deoxygenated blood
oxygenated blood
6. Left atrium contracts to push the oxygenated blood from lower body
to lower body
into the left ventricle. Fig. 28: Blood flow in Human beings

Life Processes 25
7. Left ventricle contracts to push oxygenated blood into the aorta which delivers it to the whole body.
 The separation of oxygenated and deoxygenated blood between the left and right chambers of the heart respectively
prevents their mixing. This allows a highly efficient supply of oxygen to the body. This is useful in animals with high
energy needs which constantly use energy to maintain their body temperature. They have a four-chambered heart.
For example, birds and mammals.
 Some animals have three-chambered hearts and they can tolerate some mixing of oxygenated and deoxygenated
blood. e.g., amphibians and many reptiles.
 Some animals have two-chambered hearts. e.g., fishes have only two chambers and the blood is pumped to the gills,
for oxygenated there, and passes directly to the rest of the body. Thus, blood goes only once through the heart in
the fish during one cycle of passage through the body.

Double circulation in mammals


Lung
In double circulation, the blood is transported twice through the heart to capillaries
supply blood to the body.
(i) Systemic circulation

R
Oxygenated blood from lungs → heart → a o r t a → systemic arteries

A
A A Key
→ body parts → systemic veins → heart V V Oxygen-rich blood

M
Right Left Oxygen-poor blood
(ii) Pulmonary circulation

U
Deoxygenated blood from the body → heart → pulmonary arteries Systemic
→ lungs → pulmonary veins → heart.
K Systemic circuit
capillaries
T
Blood Pressure Fig. 9:
Fig. 29:Double
Doublecirculation:
Circulation: Mammal
Mammal
N
Blood Pressure is the force that blood exerts against the wall of a vessel. This pressure is much higher in arteries as
A

compared to veins. The pressure of blood in the artery during ventricular systole (contraction) is referred as systolic
pressure and pressure in artery during ventricular diastole (relaxation) is defined as diastolic pressure. The normal
SH

systolic pressure is around 120 mm of Hg and diastolic pressure is around 80 mm of Hg.


Blood pressure is measured with the help of an instrument called a sphygmomanometer. The rise in the blood pressure
A

more than 140 mm Hg (systolic) and 90 mm Hg (diastolic), is called as high blood pressure. High blood pressure is
PR

also referred to as hypertension and is caused due to the constriction of arterioles, which results in increased resistance
to blood flow. It can cause rupture of an artery and can lead to internal bleeding.
R.

Heart Beats
Heartbeat is defined as the one complete cycle of contraction and relaxation of heart. An electrocardiogram (ECG)
D

provides information about heartbeats. Normal heartbeat is around 68-72 per minute.

Lymph
‰ When blood flows into thin capillaries, some amount of plasma filters out of thin capillaries. This fluid is called interstitial
fluid or tissue fluid or lymph. As it bathes the cells and lies outside the cells, lymph is also called extracellular fluid.
‰ It is colourless and contains lymphocytes. Unlike blood, it does not contain red blood corpuscles, platelets and plasma
proteins.
‰ Lymph flows only in one direction, that is from tissues to heart.
‰ From the intercellular space, lymph drains into lymphatic capillaries, which join to form large lymph vessels that finally
open into larger veins.
‰ Lymph vessels along with lymph nodes and lymph glands constitute lymphatic system.

26 Class-X BIOLOGY PW
‰ The functions of lymph are:
 It transports fatty acids and glycerol from small intestine to blood.
 Lymphocytes present in it destroy harmful pathogens.
 It drains excess tissue fluid from intercellular sapces back into the blood. It carries lymphocytes and antibodies from
lymph nodes to the blood.
 Lymph nodes localise the infection and prevent it from spreading to other body parts.

Constituents of Lymph
‰ Water
‰ Soilds: The solid content found in the lymph is as follows:
 Carbohydrates
 Fats
 Proteins: For example, albumin, globulin and fibrinogen.
‰ Other constituents: Creatinine, urea, inorganic phosphorus, chlorides, enzymes, calcium and antibodies.

R
A
Importance of Lymph

M
(i) It transports oxygen and nutrition to those parts of the body where blood cannot reach.
(ii) It drains excess tissue fluids.

U
(iii) Lymph returns proteins to the blood from the tissue spaces.
(iv) Fats from the intestine are also absorbed via the lymph. K
T
(v) The monocytes and lymphocytes of lymph act as defensive cells of the body.
N
(vi) The lymph also destroy bacteria from tissues.
A

Proteins and some fluid


leaks from blood capillaries
SH

in interstitial fluid and enters


lymphatic vessels.
A
PR

Lymphatic vessels Inside lymphatic vessels


return lymph to the blood it is termed as lymph which
capillaries. flows throughout the body.
R.

Defensive cells (WBC) Lymphatic vessels drains


in lymph nodes get activated into lymph nodes where
D

and destroy the pathogens foreign substances and


and forign materials. pathogens are trapped.

Fig. 30: Relationship between Circulatory system and Lymphatic system

S.No. Lymph Blood


1. It is a colourless fluid. It is a red coloured fluid.
2. Form when interstitial fluid get collected through Produced in Bone marrow and flows circularly in body
lymph capillaries. Moves in body in single direction. by single or double circulation
3. Composed of calcium, blood proteins, small amount of RBC, WBC, Plasma, platelets are major component.
lymphocytes and some water.
4. It transports WBC within lymph nodes, fights against Regulate body temperature, maintain pH and perform
infectious bacteria. clotting mechanism to prevent loss of blood.
5. Carries less oxygen and proteins. Carries large proportions of oxygen and proteins.

Life Processes 27
Excretion
Carbon dioxide
Excretion is the removal of metabolic wastes from the body. Animals
It is eliminated
accumulate ammonia, urea and uric acid, CO2 water and ions like through lungs
Na+, K+, Cl–, phosphate, sulphate, etc either by metobolic activities or
by other means like ingestion. These substances have to be removed Water
totally or partially. Ammonia, urea and uric acid are the major forms Bile pigments Extra water is
of nitrogen wastes excreted by animals. Some of these removed from the
pigments are body in the form
excreted in urine Substances to of sweat
Excretion in Animals be eliminated
In unicellular organisms, waste products are discharged directly from the body
through the surface of the cell. Wastes from the body must be Nitrogenous
accumulated at one place and then released out. Multicellular waste
Excess salts
organisms use more complex excretory methods. For example, higher It is excreted out
It is given
plants eliminate gases through the stomata on the surface of leaves. out through
through
kidney
In animals, particular excretory organs are present. kidney

R
Excretory Organs/Structures in Animals

A
Insects e.g.,

M
Animals Flatworm Earthworm Amoeba Hydra All chordates
cockroach

U
Excretory Protonephridia Nephridia Cellular Body surface Kidneys Malpighian
structures

Excretory Products
(flame cells) surface
K tubules
T
N
The waste products of organisms body can be divided into two types:
‰ Organic Waste Product: It is mainly carbon dioxide which is a by product of respiration. In aquatic animals, it is
A

diffused out in the water through general surface of the body or through respiratory surface while in terrestrial animals,
SH

it is diffused out in the air.


‰ Nitrogenous Waste Products: These substances contain nitrogen in them. The aquatic animals excrete ammonia while
A

terrestrial animals mostly excrete urea or uric acid. Out of the three nitrogenous waste products, ammonia is the most
toxic, followed by urea. Uric acid is least toxic.
PR

Excretion in Human Beings


R.

Human Excretory System


D

Human excretory system comprises of:


‰ a pair of kidneys: primary excretory organ that makes the urine.
‰ a pair of ureter: collects urine from the kidneys.
‰ a urinary bladder: stores urine temporarily.
‰ a urethra: discharges urine out of the body.

Structure of Kidney
Kidney is the main excretory organ of our body. Resting on top of each kidney is an adrenal gland (also called the
suprarenal gland).
Shape – Bean shaped
Colour – Dark red

28 Class-X BIOLOGY PW
Size – 10-12 cm length, 5-7 cm width, 2-3 cm thickness.
Weight – 125-170 gms
Position – Located laterally on either sides of the vertebral column. The right kidney is slightly lower than the left due
to the presence of liver on the right side.

External structure Adrenal gland


Inferior
Each kidney is lined and covered by a tough, fibrous capsule of connective tissues. This vena cave Renal artery
Renal Vein
covering is known as renal capsule. Lateral surfaces of kidneys, are convex o n t h e Pelvis
Medulla Kidney
o u t e r s i d e whereas concave on the inner side. The depression on the inner region
is called hilum. Cortex
Dorsal aorta
Ureter
Internal Structure
The internal morphology of kidneys can be categorized into two parts. Its outer part is Urinary bladder
called cortex and inner part is called medulla. Urethra

Fig.Fig.
31:4:Human
Human urinary system

R
Structure of Nephron urinary system

A
The functional unit of the kidney is the nephron. Each kidney of an individual is formed of about one million nephrons.

M
It is differentiated into four regions having different anatomical features and different physiological roles.
The four regions of the nephron are:

U
(i) Bowman’s capsule: It is a large double-walled cup and lies in the renal cortex. The fine branches of the renal artery

K
(the artery that supplies the kidney) form a capillary network in this cup. This tuft of capillaries is called glomerulus
and the outer wall is continous with the rest of the nephron. The walls of glomerulus and and bowman’s capsule form
T
fine membrane through which blood will be filtered. The Bowman's capsule and glomerulus together constitutes the
N
renal corpuscle or malpighian body.
A

(ii) PCT: Proximal convoluted tubule originates from the back of the Bowman’s capsule and it is highly convoluted. It
lies in the renal cortex. The wall consists of a single layer of cuboidal cells bearing a lot of microvilli on the surface.
SH

i.e., brush bordered cuboidal epithelium.


Proximal Convoluted Tubule
A

It is near to the It is coiled It is a fine tube


PR

Bowman's capsule

(iii) Loop of Henle: A U-shaped segment of the nephron Afferent


Efferent
R.

extends from the PCT and joins to the DCT. It is located artcriole
artcriole
Glomerulus
in the renal medulla. It consists of two straight parallel Bowman’s
D

Proximal convoluted
limbs, a descending limb which is a continuation of capsule
tubule
Distal
the PCT and enters into the renal medulla and an convoluted
ascending limb which re-enters the renal cortex and Descending limb tubule
joins the DCT. Henle’s of loop of Henle
loop Ascending limb
(iv) DCT: It is highly twisted like the PCT. It lies in the of loop of Henle
renal cortex. The terminal short area of the DCT is Vasa recta Collecting duct

called the collecting tubule. It opens into the collecting Fig. 6: AAdiagrammatic
Fig. 32: representationofofaanephron
diagrammatic representation nephronshowing
showing
blood
duct. The collecting ducts receive the collecting tubule blood vessels,
vessels,duct
ductand
andtubule
tubule
of several ducts.
Distal Convoluted Tubule
It is away from the It is coiled It is a fine tube
Bowman's

Life Processes 29
Ureter
The collecting ducts open into the ureter. It transports urine to urinary bladder. Each ureter starts from interior part of kidney.

Urinary bladder
Each ureter opens into the urinary bladder. The urinary bladder is a muscular sac. The wall of urinary bladder is flexible.
It collects and stores urine temporarily. The urine is eliminated through urethra by contraction of muscles.

Urethra
It is a tubular and muscular structure which extends from the urinary bladder to the outside. It takes the urine to the outside.

Urine
It is mostly water. It contains 2.5% urea. It also has some ions and creatinine. The colour of urine is pale yellow due to
the urochrome pigment which is formed in the liver and intestine due to the reduction of haemoglobin.

Physiology of Excretion

R
The formation of urine involves the following processes: Glomerular filtration or ultrafiltration of the blood plasma by

A
the glomeruli. Reabsorption and secretion, that takes place in different parts of the nephron.

M
Mechanism of Urine Formation

U
The mechanism of urine formation involves the following steps:
K
1. Glomerular Filtration: Glomerular filtration occurs in the glomerulus where blood is filtered. This process occurs
across the three layers- the epithelium of Bowman’s capsule, the endothelium of glomerular blood vessels, and a
T
membrane between these two layers.
N

Blood is filtered in such a way that all the constituents of the plasma reach the Bowman’s capsule, except proteins.
A

Therefore, this process is known as ultrafiltration.


SH

2. Reabsorption: Around 99 per cent of the filtrate obtained is reabsorbed by the renal tubules. This is known as
reabsorption. This is achieved by active and passive transport.
A

3. Secretion: The next step in urine formation is tubular secretion. Here, tubular cells secrete substances like hydrogen
ions, potassium ions, etc into the filtrate. Through this process, the ionic, acid-base and the balance of other body fluids
PR

are maintained. The secreted ions combine with the filtrate and form urine. The urine passes out of the nephron tubule
into a collecting duct.
R.

4. Urine: The urine produced is 95% water and 5% nitrogenous wastes. Wastes such as urea, ammonia, and creatinine
are excreted in the urine. Apart from these, the potassium, sodium and calcium ions are also excreted.
D

Osmoregulation
Osmoregulation in Organisms
It is a process by which body regulates the osmotic pressure of fluids and electrolytic balance in organisms. In animals,
this process is brought about by osmoreceptors, which can detect any alteration in the osmotic pressure. Humans and other
warm blooded organisms have osmoreceptors in the hypothalamus. Besides the brain, osmoregulators are also present in
the kidney.

Osmoregulation in Fish
Freshwater fish and marine fish osmoregulate in quite different ways. This is because the environments which they have
varying levels of salinity, hence the process of osmoregulation is different.

30 Class-X BIOLOGY PW
‰ Osmoregulation in Marine Fish
Absorbs water through skin
 Marine water fishes are hypotonic (less salt in Actively takes up ions
their body fluids than ocean’s water) in their through gills
surrounding environment. Consequently, it
tends to lose water and absorb the salt.
Drinks little
 Marine fish drink large quantities of water water
and restrict urination. Also, another additional
energy expenditure arises as these organisms
Movement of water
actively need to excrete salt from the body Movement of ions
through the gills. (a) Excretes dilute urine

‰ Osmoregulation in Freshwater Fish Loses water through skin


 Freshwater fishes are hypertonic in their
surrounding environment, which means that
the salt concentration is higher in their blood
Drinks ample
than in their surrounding water. water
 They absorb a controlled amount of water via Direction of
the mouth and the gill membranes. Due to this Direction of ion water movement

R
movement (NA+,
Excretes ions through gills
intake of water, they secrete large quantities K+, CL–) Excretes concentrated urine
(b)
of urine, through which much salt is lost. The

A
Fig. 33: (a) Osmoregulation in a freshwater environment and
salt is replaced with the help of mitochondria-

M
(b) Osmoregulation in a saltwater environment
rich cells in the gills. These cells absorb salt
into the blood from the surrounding water.

U
Osmoregulation in Plants
K
Plants use stomata on the lower side of their leaves to maintain loss of water. Plants growing in hydrated soils compensate
T
water loss by transpiration or by absorbing more water from the soil. The plants that grow in semi-arid regions store water
in the vacuoles and have thick and fleshy cuticles to avoid water loss.
N

Osmoregulation in Human Beings


A

The kidney is the main organ accountable for osmoregulation in humans. Amino acids, water and glucose are reabsorbed
SH

by the kidneys. When the water level in the body is at peak, it releases a large amount of hypotonic urine. When the
level of water is low, it retains water and produces a low amount of hypertonic urine. This is how the kidneys regulate
A

the electrolytic balance of the body.


PR

Knowledge Hub
R.

Regulation of Urine Concentration


™ In Winters:
D

Less water used for metabolism



more blood volume

Less conc. of ADH

Less permeability of DCT

Less reabsorption of H2O into DCT and collecting duct

more urine formed

Life Processes 31
™ In Summers:
More water used for metabolism

Less blood volume

More conc. of ADH

More permeability of DCT

More reabsorption of H2O into blood capillaries

Less urine formed

Artificial Kidney / Haemodialysis

R
A
Kidneys are important organs for survival. However, several factors like
infections, injury or restricted blood flow can reduce the activity of

M
kidneys. This can cause accumulation of poisonous wastes in the body,

U
which can even lead to death. In case of kidney failure, an artificial
kidney can be used. An artificial kidney is a device to eliminate
nitrogenous waste products from the blood through dialysis. K
T
Artificial kidneys consist of a number of tubules with a semi-permeable
N
lining, suspended in a tank filled with dialysing fluid. This fluid has the
A

same osmotic pressure as blood, except that it is devoid of nitrogenous


wastes. The patient’s blood is passed through these tubes. During this
SH

passage, the waste products from the blood pass into a dialysing fluid
by diffusion. The purified blood is pumped back into the patient. This Fig. 34: Hemodialysis
A

is similar to the function of the kidney, but it is different since there is


PR

no reabsorption involved.
R.

Excretion in plants
In plants, no definite excretory organ or system is present for removing wastes. The excretory products which are easily
D

diffusible are removed via diffusion or osmosis. Plants strategically redistribute wastes between metabolic processes. In a
way, carbon dioxide is the waste product of respiration which is used by photosynthesis. Likewise, oxygen can be viewed
as a waste product of photosynthesis and is utilized by respiration.

Methods of Disposal of Plant wastes


The gaseous wastes of respiration and photosynthesis in plants (carbondioxide, water vapoour and oxygen) are removed
through the ‘stomata’ in leaves and lenticels in stems and released in the air.
‰ Plants also store their waste in leaves and stems. Various waste products are stored in the cellular vacuole. Over time,
they drop these leaves and get rid of the waste. Many of their waste products are bitter, making their lives undesirable to
herbivores. Several plant wastes have human economical value as well. Rubber harvested from the tree in the form of latex
is also a type of excretory product. Latex is a sticky, milky and white colloid substance drawn off by making incisions in the tree’s
bark and collecting the fluid in vessels by a process called “tapping”.

32 Class-X BIOLOGY PW
‰ Tea leaves, amla, betel nut and bark of trees contain tannins.
‰ Quinine and morphine are medicines derived from alkaloids stored in Cinchona bark and Opium fruits, respectively.
‰ Caffeine found in coffee seeds and nicotine in tobacco leaves are also alkaloids.
‰ Calcium oxalate crystals accumulate in some tubers like Yam (zimikand).

Summary

Different types of movement can be taken as an indication of life. The major characteristic which is used to decide
abether something is living or dead is movement. The movement in animals is fast and can be observed easily but
the movement in plants is slow and observed with difficulty. Animals can move from one region to another or they
can move their body parts. The plants can only move parts of their body such as leaves, flowers, roots and shoots.
Life processes are processes undergoing in living organisms to sustain life like respiration, nutrition, transport

R
of materials within the body and removal of waste products. Organisms need the energy to perform various

A
activities. The energy is supplied by the nutrients. Organisms need various raw materials for growth and repair.
These raw materials are provided by nutrients. Autotrophic nutrition involves the use of simple inorganic materials

M
like carbon dioxide, water and sunlight as the energy source to synthesize complex high-energy organic material

U
(carbohydrates). Heterotrophic nutrition involves the utilization of complex material prepared by other organisms
or autotrophs. Different organisms utilize different nutritional processes as it depends upon the source of carbon
K
from where the food is taken. Some organism feed on dead and decaying organic matter. Some organisms feed on
the expense of another organism and in turn causing it harm. This is called parasitic mode of nutrition. Nutrition in
T
humans involves various steps. Food is taken by mouth and is broken down by various steps along the alimentary
N

canal. The breakdown of complex food into simpler ones is known as digestion. The digested food is absorbed in the
A

small intestine and excess of water is absorbed by the large intestine. After this, the waste products are removed from
the body through the anus. Respiration may be aerobic (in the presence of oxygen) or anaerobic (in the absence
SH

of oxygen). Aerobic respiration provides more energy to the organism. Respiration involves gaseous exchange, ie.,
intake of oxygen from the atmosphere and release of carbon dioxide and cellular respiration, i.e., breakdown of
A

complex organic compounds such as glucose to provide energy in the form of ATP. Mitochondria are the region of
cellular respiration. When someone runs too fast, he may experience throbbing pain in the leg muscles. This occur
PR

due to anaerobic respiration taking place in the muscles. The deposition of lactic acid causes pain in the leg muscles.
The pain subsides after taking rest for some time. The human respiratory system involves the nasal cavities, nose,
pharynx, larynx, trachea/ windpipe, bronchioles, bronchi, and alveoli. The walls of the alveoli contain an extensive
R.

network of blood-vessels where the exchange of gases takes place. Since the amount of oxygen dissolved in water is
D

comparatively lower than that in air, the aquatic animals have to breathe rapidly to take in sufficient oxygen. Thus,
the rate of breathing in aquatic organisms is higher than the terrestrial organisms. Plants have stomata (present in
leaves) and lenticels (present in stems) which are involved in the exchange of gases. Transportation in humans is
done by the circulatory system. It is responsible for the transport of oxygen, nutrients, removal of carbon dioxide
and other excretory products. The circulatory system in humans mainly consists of blood, blood vessels and heart.
Since ventricles have to pump blood into various organs, they have thicker muscular walls than that of atria. In the
human heart, blood passes through the heart twice in one cardiac cycle. This type of circulation is called double
circulation. It ensures complete segregation of oxygenated and deoxygenated blood which is necessary for optimum
energy production in warm-blooded animals. In highly differentiated plants, transport of water, minerals, food and
other materials is a function of the vascular tissue which consists of xylem and phloem. Transpiration is the loss of
water in the form of water vapour from the aerial parts of the plant, helps in the absorption and upward movement
of water and minerals dissolved in it from roots to the leaves. The harmful metabolic wastes from the body are
removed by the process of excretion. Excretory system of humans consists of a pair of kidneys, a pair of ureters,

Life Processes 33
urinary bladder and urethra. In human beings, excretory products in the form of soluble nitrogen compounds are
removed by the nephrons. Nephron is the structural and functional unit of kidney. Any problem in the activity of
kidney leads to accumulation of poisonous wastes in the body, which can even lead to death. In such situation, an
artificial kidney can be used. Plants use a variety of techniques to get rid of waste material. Carbon dioxide, excess
water and nitrogenous compounds are the major excretory products in plants. Waste products may be stored in leaves
falls off. Other waste material may be stored in the cell-vacuoles or as resins

NCERT Corner
1. Why is diffusion insufficient to meet the oxygen 5. What are the differences between autotrophic
requirements of multicellular organisms like nutrition and heterotrophic nutrition?
humans? Ans. Differences between autotrophic and heterotrophic

R
Ans. In multicellular organisms such as humans, all nutrition are as follows:
the body cells are not in direct contact with the

A
Heterotrophic Autotrophic
external environment. Therefore, every cell of the

M
nutrition nutrition
body will not get enough oxygen as per need by
the process of diffusion as it is a slow process.

U
(i) Organisms cannot Organisms use
2. What criteria do we use to decide whether make their own simple inorganic
something is alive?
Ans. Living organisms carry out metabolism to maintain
K food from simple
inorganic matter
materials like carbon
dioxide and water
T
their ordered structures by acquiring energy and and depend on and synthesise their
N

matter from the surrounding. other organisms for food in presence of


A

3. What are outside raw materials used by an their food. sunlight.


SH

organism?
(ii) All the animals, All green plants
Ans. Energy is required by all living species, to sustain and
most bacteria and and some algae
preserve their existence.
A

fungi shows this shows this mode of


Oxygen, water, and food are examples of external raw
mode of nutrition. nutrition.
PR

materials used by organisms.


Plants obtain their raw materials from the environment 6. Where do plants get raw materials required for
in the form of carbon dioxide, water, and sunlight. the process of photosynthesis ?  (CBSE 2020)
R.

These are used to synthesize their food in the presence Ans. (i) Carbon dioxide: It is obtained from atmosphere.
D

of chlorophyll, a green color pigment. (ii) Plants absorb water from the soil through roots
Animals use basic materials in the form of food, and transport to leaves.
water, and oxygen from the environment. (iii) Sunlight: Sunlight comes from the sun.
4. What processes would you consider essential (iv) Chlorophyll: It is present in chloroplast found in
for maintaining life ? green plants.
Ans. The maintenance function of living organisms 7. What is the role of the acid in our stomach ?
must go on even when they are not doing anything Ans. Role of acid in the stomach is :
particular. The various processes essential
(i) To kill harmful bacteria that enter with food in body.
for maintaining life are nutrition, respiration,
transportation, excretion, control and coordination. (ii) To make acidic medium which is required for the
In absence of any one of these, the life become, activation of the enzyme pepsin.
difficult. (iii) It also helping digestion of proteins.

34 Class-X BIOLOGY PW
8. What is the function of digestive enzymes ? The xylem cells of roots, stems and leaves are
Ans. The food we eat is very complex in nature, interconnected to form a conducting channel that
because it comprises of complex molecules. reaches all parts of the plant body. The root cells
Digestive enzymes break down these complex take ions from the soil. This creates a difference
molecules into smaller simpler molecules, so that between the concentration of ions present in roots
they can be easily absorbed by the walls of the and soil. Therefore, there is a steady movement of
intestine. water into the xylem. An osmotic pressure builds
up, there is a movement of water and minerals
9. How is the small intestine designed to absorb from one cell to the other cell due to osmosis.
digested food ? The continuous loss of water takes place due
Ans. The small intestine has folds which increase to transpiration. Transpiration creates a suction
surface area. These folds have finger-like pressure as a result of which water is forced into
projections known as villi which further increases the xylem cells of roots. The effect of root pressure
surface area. The villi consist of many blood for transportation in plants is more important at
vessels which takes the absorbed food to each night while during daytime transpiration pull
and every cell of the body. becomes the major driving force.

R
10. What advantage over an aquatic organism 13. How is food transported in plants?

A
does a terrestrial organism have with regard Ans. The movement of food in phloem (or translocation)
to obtaining oxygen for respiration?

M
occurs by using energy. Food (sugar) made in
Ans. Aquatic organisms use oxygen dissolved in leaves is loaded into the sieve tubes of phloem

U
surrounding water. Since, air dissolved in water tissue by utilizing energy from ATP. Now, water
has fairly low concentration of oxygen, the aquatic
organisms need to have higher rate of breathing. K
enters the sieve tubes containing sugar by the
process of osmosis. Because of this, the pressure
T
Terrestrial organisms acquire oxygen from the in the phloem tissue increases. This high pressure
generated in the phloem tissue moves the food to
N
oxygen-rich atmosphere through respiratory
organs. Hence, can work less with breathing rate all parts of the plant having less pressure in their
A

than aquatic organisms. tissues. This help the phloem to transport food
according to the requirement of the plant.
SH

11. What are the different ways in which glucose is


oxidised to provide energy in various organisms? 14. How is oxygen and carbon dioxide transported
in human beings?
A

Ans. The first step of breakdown of glucose (6 carbon


molecule) occurs in the cytoplasm of cells of all Ans.
PR

the organisms. This process yields a three carbon (i) Transport of carbon dioxide: Carbon dioxide
molecule compound known as pyruvate. is more soluble in water as compared to oxygen.
Further, breakdown of pyruvate takes place in Therefore, it is mostly transported from body
R.

many distinct ways in different organisms. tissues in dissolved form in our blood plasma to
lungs.
D

Absence of oxygen
Ethanol + CO2 + Energy
(in yeast) (2-carbon (ii) Transport of oxygen: Haemoglobin present in
Molecule)
Glucose
In cytoplasm
Pyruvate
Lack of oxygen
Lactic acid the blood takes up the oxygen from the air in the
(6-carbon
Molecule)
(6-carbon
Molecule)
(in muscle cells)
(3-carbon + Energy
Molecule)
lungs. It transports the oxygen to tissues which
+ energy
In presence of oxygen
are deficient in oxygen before releasing it.
CO2 + Water + Energy
(in mitochondria)
15. How are the lungs designed in human
12. How are water and minerals transported in beings to maximise the area for exchange of
plants? gases?
Ans. The roots of a plant consists of hairs called root Ans. Within the lungs, the air passage differentiates
hair. into smaller tubes, known as bronchi which in
In plants, water and minerals are transported turn form bronchioles. The bronchioles terminates
through xylem cells from soil to the leaves. in balloon-like structures, known as alveoli. The
alveoli present in the lungs give maximum surface

Life Processes 35
area for exchange of gases. The alveoli are thin 17. Why is it necessary to separate oxygenated and
walled. They form an extensive network of blood deoxygenated blood in mammals and birds?
vessels to allow exchange of gases. Ans. Separation of oxygenated and deoxygenated
16. What are the components of the transport blood allows enough supply of oxygen to the
system in human beings? What are the body as it avoids mixing of the oxygenated
functions of these components ? and deoxygenated blood. This system is useful
Ans. The transport system (circulatory system) in in animals that have high energy requirement.
human beings mainly comprises of heart, blood Mammals and birds constantly require oxygen
and blood vessels. to obtain energy to maintain their constant body
(i) Function of blood vessels: The blood pumped temperature.
by the heart flows through the blood vessels 18. What are the components of the transport
(arteries, veins and capillaries) and also comes system in highly organised plants?
back to the heart through them.
Ans. In highly organised plants there are two conducting
(ii) Function of heart: The heart receives
tissues namely xylem and phloem.
deoxygenated blood from the body parts and

R
transport it to lungs to enrich it with oxygen. It Xylem consists of vessels, tracheids and other

A
receives purified blood from lungs and pumps it xylem tissues. The interconnected vessels and
to all the body parts. tracheids form a continuous system of water

M
conducting channels reaching all region of the
(iii) Function of blood: Blood transports oxygen,
plant. Xylem carries water and minerals.

U
digested food, carbon dioxide, hormones and
nitrogenous waste such as urea. It also protects Phloem transport soluble products of
the body from diseases and regulates the body
temperature.
K
photosynthesis from leaves to different area of
the plant body.
T
N
A
SH
A
PR
R.
D

36 Class-X BIOLOGY PW
NCERT Exercise
1. The kidneys in human beings are a part of the 6. What is the role of saliva in the digestion of food?
system for:
Ans. Saliva has salivary amylase enzyme that converts
(a) Nutrition (b) Respiration starch into sugars like maltose.
(c) Excretion (d) Transportation Starch Maltose
+ Salivary amylase
(Complex Molecule) (Simpler Molecule)
Ans. (c)
Saliva maintain also moistens the food that help
2. The xylem in plants are responsible for: in chewing and breaking down the larger pieces of
(a) Transport of water food into smaller ones.
(b) Transport of food
7. What are the necessary conditions for autotrophic
(c) Transport of amino acids
nutrition and what are its by-products?
(d) Transport of oxygen
Ans. The most essential conditions for autotrophic

R
Ans. (a) nutrition are:

A
3. The autotrophic mode of nutrition requires: (i) Sufficient sunlight.

M
(a) Carbon dioxide and water (ii) Sufficient supply of carbon dioxide.
(b) Chlorophyll (iii) Presence of chlorophyll.

U
(c) Sunlight (iv) Sufficient transport of water to green plants or cells
(d) All of the above
K of the plant.
T
Ans. (d) Carbohydrates and oxygen are the by-products of
autotrophic nutrition.
N
4. The breakdown of pyruvate to give carbon
dioxide, water and energy occurs in ________. 8. What are the differences between aerobic and
A

(a) Cytoplasm (b) Mitochondria anaerobic respiration? Name some organisms


SH

(c) Chloroplast (d) Nucleus that use the anaerobic mode of respiration.
Ans. (b) Ans. The differences between anaerobic and aerobic
A

respiration are as follows:


5. How are fats digested in our bodies ? Where
PR

does this process take place ? Anaerobic respiration Aerobic respiration


Ans. Digestion of fats occurs mainly in the small intestine. It occurs in the absence It occurs in the presence
R.

of oxygen. of oxygen.
Bile juice produced by the liver enteres in the small
intestine along with pancreatic juice. The bile salts Partial breakdown of Complete breakdown of
D

present in the bile juice emulsify the large globules food takes place. food take place.
of fats into smaller globules to provide larger surface Much less energy is
More energy is generated
area to act upon by the enzymes. produced in anaerobic
(38 ATP)
respiration. (2 ATP)
Pancreatic juice has lipase enzyme breaks down
The end products of The end products of
emulsified fats in also. Intestinal juice which consists
anaerobic respiration aerobic respiration are
of lipase enzyme that catalyzes the conversion of
fats into fatty acids and glycerol. are carbon dioxide and carbon dioxide and
pancreatic juice
ethanol (as in yeast) or water.
emulsified
Fats Emulsified Break lactic acid (as in animal
by bile salts lipase
fats muscles cells).
intestinal Some organisms which use anaerobic mode of
down of fats Fatty Acid + Glycerol
juice lipase respiration are yeast, bacteria etc.

Life Processes 37
9. How are the alveoli designed to maximise the Double circulation is very important in our body
exchange of gases ? as: The right side and the left side of the human
Ans. heart are essential to separate deoxygenated and
oxygenated blood and prevents mixing. This type of
(i) Alveoli have balloon-like structure. Thus, provides
separation of oxygenated and deoxygenated blood
maximum surface area for exchange of gases.
ensures an efficient supply of oxygen to the body.
(ii) The alveoli are thin walled and have network of This is useful for humans as they constantly require
blood vessels to allow exchange of gases between energy to regulate their body temperature.
blood and the air filled in alveoli.
12. What are the differences between the transport
10. What would be the consequences of a deficiency of materials in xylem and phloem ?
of haemoglobin in our bodies?
Ans. The differences between the transport of materials
Ans. The oxygen carrying capacity of blood reduces due in xylem and phloem are given below:
to the deficiency of haemoglobin in blood. As a
result, the production of energy by oxidation will Phloem Xylem
become slower. Therefore, an individual can fall Phloem allow the transport Xylem transports water
sick and would feel fatigue and nausea. of prepared food material and dissolved minerals

R
11. Describe double circulation in human beings. from leaves to other regions from roots to leaves and

A
Why is it necessary? of plant in dissolved form other parts of the plants
bidirectionally unidirectionally.

M
Ans. In our heart, blood is circulated twice. The
deoxygenated blood from the body is brought to 13. Compare the functioning of alveoli in the lungs

U
the right atrium through vena cava from where and nephrons in the kidneys with respect to their
it is sent to right ventricle. From right ventricle, structure and functioning.
the blood is pumped to the lungs for oxygenation
through pulmonary artery. Again, the oxygenated
Ans. K
T
blood from lungs enters the left atrium of the Nephron Alveoli
N
heart through pulmonary veins. From left atrium, A kidney has around 1 A mature lung has
A

it is carried to the left ventricle, from where this million nephrons. around 300 million
oxygenated blood is pumped to different parts of alveoli.
SH

body through the arteries. Through this, the blood


Functional unit of Functional unit of
flows from the heart two times, and thus, it is called
kidney. lungs.
A

‘double circulation’.
PR
R.
D

Quick Recall

5. _______ movements take place along the gut.


Fill in the Blanks
6. _______ from stomach digests proteins.
1. Ninety percent of the water lost by the plants during
transpiration is through _______ the of the leaf. 7. _______ emulsification of fats.

2. _______ involves the intake of complex material 8. Gaseous exchange in lungs takes place in units
prepared by other organisms. called _______.
3. Complete photosynthesis units of plants is _______. 9. The _______ prevents the entry of food into the
4. Starch changes blue in _______ solution. respiratory tract.

38 Class-X BIOLOGY PW
10. Contraction of diaphragm contributes to _______. 15. The exchange of nutrients and waste products
11. Yeast undergoes _______ fermentation. between the blood and cells occurs within the
arteries.
12. The major function of the _______ blood cells is to
transport oxygen. 16. Bowman's capsule is a part of nephron.
13. Pressure in the arteries during ventricular relaxation 17. Plant throw all their waste products.
is referred as _______ pressure.
18. Urine formation includes filtration of blood.
14. Transport through phloem is an _________ process.
19. Kidneys also perform the job of osmoregulation.
15. _______ veins pour _______ blood into heart.
20. Urine is a concentrated solution of urea.
16. The chamber of heart that pumps oxygenated blood
to the whole body is _______.
17. In humans, primary nitrogenous waste is _______.
Match the Following
18. _______ is the functional unit of the mammalian 1. Match the processes given in column-I with their
kidney. definition given in column-II and choose the correct

R
19. _______ step of urine formation ensures no nutritive option.

A
material is wasted in urine. Column-I Column-II

M
20. _______ stores urine temporarily before releasing it. P. Nutrition (i) The increase in cell size

U
and/ or number
Q. Synthesis (ii) The movement of
True and False Statements
1. Light energy is directly used in the synthesis of
K materials within the cell
or within the organism.
T
carbohydrates. R. Growth (iii) The process of obtaining
N

2. Guard cells have chloroplast. food.


A

S. Transport (iv) Combining small


3. Ball of food that descends into stomach is called
SH

molecules to create larger


chyme. more complex molecules.
4. During day, respiration does not take place, only (a) P-(iv), Q-(ii), R-(i), S-(iii)
A

photosynthesis happens.
(b) P-(iii), Q-(iv), R-(ii), S-(i)
PR

5. In humans, protein digestion starts in the mouth.


(c) P-(iii), Q-(iv), R-(i), S-(ii)
6. Villi perform digestion of lipids. (d) P-(iii), Q-(i), R-(iv), S-(ii)
R.

7. Heart muscle can perform lactic acid fermentation


2. Match the organisms given in column-I with their
during heavy exercise.
D

mode of nutrition given in column-II.


8. Aerobic respiration gives 38 ATP.
Column-I Column-II
9. Most of the gaseous exchange takes place through (Mode of Nutrition) (Organisms)
stomata.
P. Trypsin (i) Liver
10. Oxygen largely travels in blood dissolved in the Q. Amylase (ii) Gastric glands
blood plasma. R. Bile (iii) Saliva
11. Xylem transports water and minerals. S. Pepsin (iv) Pancreas
12. We have a four chambered heart which can contracts (a) P-(iv), Q-(iii), R-(i), S-(ii)
because of brain signals. (b) P-(i), Q-(iii), R-(iv), S-(ii)
13. Translocation transports sugar as glucose. (c) P-(iv), Q-(ii), R-(i), S-(iii)
14. WBCs help in preventing blood clotting. (d) P-(iv), Q-(iii), R-(ii), S-(i)

Life Processes 39
3. Match the given columns and mark the correct option. (R) Respiration (iii) The production of
Column-I Column-II new individuals of the
(P) Larynx (i) Sight of smell same kind.
(Q) Alveoli (ii) Production of sound (S) Excretion (iv) The control and
(R) Bronchi (iii) Conduction of air coordination of
chemical processes
(S) Nose (iv) Exchange of gases
within the organism.
(a) P-(ii), Q-(iv), R-(iii), S-(i)
(a) P-(iv), Q-(ii), R-(i), S-(iii)
(b) P-(i), Q-(ii), R-(iii), S-(iv)
(c) P-(iii), Q-(iv), R-(i), S-(ii) (b) P-(iv), Q-(iii), R-(ii), S-(i)
(d) P-(iii), Q-(i), R-(iv), S-(ii) (c) P-(iii), Q-(iv), R-(i), S-(ii)
4. Match column-I with column-II and choose the (d) P-(iii), Q-(i), R-(iv), S-(ii)
option.
Column-I Column-II Multiple Choice Question

R
P. Trypsin (i) Pancreas
1. Transportation in unicellular organisms usually

A
Q. Amylase (ii) Gastric glands
R. Bile (iii) Liver occurs by:

M
S. Pepsin (iv) Saliva (a) Transporting organs

U
(a) P-(i), Q-(iv), R-(ii), S-(iii) (b) DNA
(b) P-(iv), Q-(i), R-(ii), S-(iii)
(c) P-(i), Q-(iv), R-(iii), S-(ii) K(c) Diffusion
(d) They do not need any transport
T
(d) P-(i), Q-(ii), R-(iv), S-(iii)
N

5. Match column-I and column-II and choose the correct 2. If a plant is being irrigated with water containing
A

option. a radioactive isotope of oxygen, very likely the


_________ molecule will also be radioactive.
SH

Column-I Column-II
P. Xylem (i) Oxygenated blood (a) Hydrogen
A

Q. Nephron (ii) Translocation of minerals (b) Carbon dioxide


R. Arteries (iii) Clotting of blood (c) Oxygen
PR

S. Platelets (iv) Excretion (d) Carbohydrates


(a) P-(ii), Q-(i), R-(iv), S-(iii)
3. In desert plants, intake of carbon dioxide takes
R.

(b) P-(i), Q-(ii), R-(iv), S-(iii)


place:
(c) P-(ii), Q-(i), R-(iii), S-(iv)
D

(d) P-(ii), Q-(iv), R-(i), S-(iii) (a) During day


6. Match the processes given in column-I with their (b) During night
definition given in column-II. (c) During both day and night
Column-I Column-II (d) They do not intake carbon dioxide; they totally
(P) Regulation (i) The removal of depend on carbon dioxide produced from
metabolic waste from respiration.
an organism.
4. Which of the following cells are usually rich in
(Q) Reproduction (ii) The chemical process
of oxidizing organic chloroplasts?
molecules to release (a) Mesophyll cells (b) Epidermis cells
energy. (c) Root cells (d) Xylem cells

40 Class-X BIOLOGY PW
5. The contraction and expansion movement of the 13. We can assume four-chambered heart:
walls of the food pipe is called: (a) Will be suitable in cold blooded animals
(a) Pulse (b) Peristalsis (b) Will be suitable in warm blooded animals
(c) Mastication (d) Micturition (c) Will be suitable in both
(d) Will be suitable in none
6. Which of the following statements is incorrect?
14. The vein which brings oxygenated blood into the
(a) ATP cannot be produced without mitochondria. heart is:
(b) ATP can be produced in mitochondria. (a) Vena cava (b) Pulmonary artery
(c) ATP can be produced in the cytoplasm. (c) Pulmonary vein (d) Aorta
(d) ATP can be consumed in the cytoplasm. 15. Primary nitrogenous waste in humans that is thrown
7. The dough for making cakes rises due to: out is:
(a) Carbon dioxide (b) Ammonia
(a) Production of alcohol
(c) Urea (d) Amino acids
(b) Production of lactic acid
16. Absence of which of these processes would have
(c) Production of carbon dioxide
wasted a lot of glucose:

R
(d) Production of water (a) Filtration (b) Reabsorption

A
8. Bronchi most specifically, has the role of: (c) Secretion (d) None of these
(a) Exchange of gases

M
Assertion & Reason Type Questions
(b) Conduction of air

U
(c) Storage of air Direction: In the following Questions, the Assertion
and Reason have been put forward. Read the statements
(d) Oxygenation of blood
9. Breathing is inefficient in humans because:
K
carefully and choose the correct alternative from the
T
following:
(a) Oxygen cannot be isolated from air.
N
(a) Both Assertion (A) and Reason (R) are true and
(b) We live in oxygen-deficient atmosphere. Reason (R) is the correct explanation of Assertion (A).
A

(c) We have common path for inhalation and


(b) Both Assertion (A) and Reason (R) are true and
SH

exhalation.
Reason (R) is not the correct explanation of Assertion (A).
(d) We lack oxygen specific transport proteins.
(c) Assertion (A) is true but Reason (R) is false.
A

10. If plant xylems are blocked, the plant will:


(d) Assertion (A) is false but Reason (R) is true.
PR

(a) Suffer from water shortage only.


(b) Suffer from water and carbohydrate shortage. 1. Assertion (A): Raw materials needed for photosyn-
thesis are carbon dioxide, water and minerals.
(c) Suffer from water and mineral shortage.
R.

(d) Suffer from water, mineral and carbohydrates Reason (R): All nutrients provide only energy to an
organism.
D

shortage.
11. If the phloem is blocked, the plant will: 2. Assertion (A): Autotrophic nutrition occurs in green
plants.
(a) Suffer from food shortage only.
Reason (R): Green plants self-manufacture their
(b) Suffer from mineral shortage.
food from inorganic carbon source.
(c) Suffer from water and food shortage.
3. Assertion (A): Pyruvate is a six-carbon molecule
(d) Suffer from water and food shortage.
Reason (R): It is prepared in the cytoplasm as the
12. The advantage of a four-chambered heart is:
first step to cellular respiration.
(a) Increased blood-carrying capacity.
4. Assertion (A): Haemoglobin is the respiratory
(b) Increased blood storage capacity.
pigment in human beings.
(c) Increased blood production capacity.
(d) Increased blood oxygen-carrying efficiency. Reason (R): It transport oxygen in the human body

Life Processes 41
5. Assertion (A): Strenuous physical exercise may cause 1. Statement-I: Photosynthesis occurs only in
fatigue due to accumulation of CO2 in the blood. eukaryotes.
Reason (R): Lactic acid has a toxic effect on cells,
Statement-II: Prokaryotes do not have photosensitive
which causes muscle fatigue. pigments
6. Assertion (A): Pulmonary veins and Venacava both 2. Statement-I: Protein digestion cannot begin until
are veins. acidic environment is created in stomach.
Reason (R): Concentration of oxygen is higher in
Statement-II: Mucus is activated by acidic
the pulmonary vein compared to venae cavae as it
environments.
carries oxygenated blood to the systemic circuit,
unlike other veins which carry deoxygenated blood. 3. Statement-I: Plants have structures that take in
7. Assertion (A): The translocation of food and other gases distributed throughout their body.
substances takes place by xylem. Statement-II: They need huge amount of energy.
Reason (R): Translocation occurs with the help of 4. Statement-I: Fishes have higher breathing rates.
adjacent companion cells.
Statement-II: Oxygen is less available to them.
8. Assertion (A): Artificial kidney is a device used to

R
remove nitrogenous waste products from the blood 5. Statement-I: Humans have double circulation.

A
through dialysis. Statement-II: They have blood cells which are all

M
Reason (R): Reabsorption does not occur in efficient in carrying oxygen.
artificial kidney.

U
6. Statement-I: Guttation is droplet exudation,
9. Assertion (A): Excretion is the biological process whereas transpiration is the evaporation of water
of removing nitrogenous waste from the body.
Reason: The mode of excretion is the same in both
Kfrom the plant surface.
T
Statement-II: Waste products are also stored
unicellular and multicellular organisms. in some plants in the form of oil like Jasmine,
N

eucalyptus and orange.


A

Statement Type Questions 7. Statement-I: Plants do not produce wastes.


SH

Read the following statements and select the correct Statement-II: Plant lacks specialised excretory
options. structures.
A

(a) Both Statement-I and Statement-II are correct. 8. Statement-I: Donation of a kidney may be made
PR

(b) Both Statement-I and Statement-II are incorrect. without the consent of the donor and his/her
(c) Statement-I is correct & Statement-II is incorrect. family.
(d) Statement-I is incorrect & Statement-II is correct.
Statement-II: People can survive on one kidney.
R.
D

a mesophyll cell is performing photosynthesis, the


Very Short Answer Type Questions
ratio of pH of thylakoid to that of cytoplasm will
1. How desert plants perform photosynthesis if their be:
stomata remain closed during the day? (i) > 1
2. Photolysis of water is a key step in photosynthesis. (ii) < 1
It happens inside thylakoids in the chloroplast. If (iii) = 1

42 Class-X BIOLOGY PW
3. Most of the digestion and absorption of the food 3. What will happen if the gastric glands do not secrete
takes place in the: mucus?
(i) small intestine. 4. Explain the process of nutrition in Amoeba.
(ii) liver.
5. (a) W
 hy does a piece of bread taste sweet when
(iii) stomach.
chewed for some time?
(iv) large intestine.
(b) Cellulose acts as a roughage in man but serves as
4. What is the role of sphincters in digestion? a source of nutrients in cow. Justify the statement
5. Draw an analogy between how energy is stored 6. Describe the function of trachea? Why do the walls
within ATP and between two similar poles of not collapse even when there is less air in it?
magnet.
7. Why is diffusion not sufficient to meet oxygen
6. What is the function of alveoli in the lungs.
requirement of all the cells in multicellular
7. What happens to the rate of breathing during organisms?
vigorous exercise and why?
8. Name three kinds of blood vessels of the human
8. Define which one of the following statements is circulatory system and write one function of each
correct about the human circulatory system? in a tabular from.

R
(a) Blood transports only oxygen and not carbon
9. (a) What is translocation? Why is it essential for

A
dioxide.
plants?

M
(b) Human heart has five chambers.
(b) Where do the substances in plants reach as a result
(c) Valves ensure that the blood does not flow
of translocation?

U
backwards.
(d) Both oxygen-rich and oxygen-deficient blood gets 10. (a) What will happen to guard cells and the stomatal
mixed in the heart. K pore when water flows into guard cells.
T
9. Name the tissue which transports soluble products (b) How do plants transmit information from cell to
of photosynthesis in a plant. cell?
N

10. How does transport of water occur at night in the 11. Give reasons for the following:
A

absence of transpiration? (a) The two ventricles have thicker muscular walls
SH

11. How are nitrogenous wastes formed in the body? than the two atria in human heart.
(b) The capillaries have walls which are one-celled
12. Justify Plants biological wastes are sometimes riches
A

for humans. thick.


12. How do artificial kidneys work?
PR

13. Reflect on the statement, “Kidneys process 180 L of


blood to make 1.8 L of urine”
Long Answer Type Questions
R.

Short Answer Type Questions 1. Name the organ and site of photosynthesis in green
D

1. Write four sequential steps of the procedure of the plants. What are the raw materials essential for this
experiment ‘‘Preparing a temporary mount of a leaf process? How are they obtained by green plant?
peel to show stomata.’’ 2. Draw a labelled diagram of cross-section of a leaf.
2. Complete the following flow chart as per the given 3. (a) Write the function of the following in the human
instructions: alimentary canal :
Gastric Glands Present in (i) Saliva
the wall of Stomach
(ii) HCl in stomach
Secretions
(iii) Bile juice
(iv) Villi
a b c (b) Write one function each of the following enzymes:
Function Function Function
(i) Pepsin
d e f (ii) Lipase

Life Processes 43
4. In the experiment of preparing a temporary mount
of a leaf peel to observe stomata, we use two liquids
other than water. Name these two liquids and state
when and why these liquids are used.
List four precautions in proper sequence which we Wide mouthed bottle

observe while preparing a temporary mount of a


leaf peel Potted Plant

5. Draw a diagram to show the human alimentary KOH


canal and label on it the following: Gall bladder,
Stomach. Name the longest part of the alimentary 1. Which factor is being studied here?
canal.
2. What is the purpose of keeping KOH in the flask?
6. Draw a diagram of human respiratory system and
label – pharynx, trachea, lungs, diaphragm and 3. Explain the term photosynthesis.
alveolar sac on it. 4. What will you observe when the leaf A is tested for

R
7. (a) 
Write the correct sequence of steps followed starch?
during journey of oxygen rich blood from lungs

A
5. Write a well-balanced chemical equation for the
to various organs of human body.

M
process of photosynthesis
(b) What happens when the system of blood vessels

U
develop a leak? Case Study-II
8. (a) Mention any two components of blood.
(b) Plants can sustain slower transport needs. Justify. K
The diagram given here represents the relationship between
a mouse and a physiological process that occur in green
T
(c) Write the function of valves present in between
plants. Study the diagram and answer the question that
N
atria and ventricles.
follows.(2017)
A

9. (a) Define excretion.


1. Name the physiological process occurring in the
SH

(b) Name the basic filtration unit present in the kidney.


(c) Draw excretory system in human beings and label green plants that have kept the mouse alive.
the following organs of excretory system which 2. Explain the physiological process mentioned above.
A

perform following functions:


PR

3. Why did the mouse die in the bell jar B?


(i) form urine.
(ii) is a long tube which collects urine from 4. What is the significance of the process as stated in
kidney. (1) for life on earth?
R.

(iii) store urine until it is passed out. 5. Reframe the above-mentioned physiological process
D

10. (a) Name the organs that form the excretory system in the form of a chemical equation.
in human beings.
(b) Describe in brief how urine is produced in human Sunlight
body.

Case-Based Type Questions Sunlight Mouse - dead

Case Study-I
The diagram given below represents an experiment to Green plant

prove the importance of a factor in photosynthesis. Answer


Mouse - dead
the question that follow:

44 Class-X BIOLOGY PW
Case Study-III 1. Identify the system.

Look at the experiment setup A and B showing that heat is 2. Label the parts marked 2 and 4. Mention the
evolved during respiration when germinating seed are kept function of part 5.
in the thermos flask.
3. Name the structural and functional units of the part
THERMOMETER marked 1.
4. What is the fluid that accumulates in part 3?
WET COTTONWOOL
5. Which is the main nitrogenous waste present in it?

DEAD SEEDS WITH ANTISEPTIC


Case Study-V
GERMINATING SEEDS

The given figure depicts a single nephron from a


mammalian kidney. Identify the labelled parts (A-E) and
match them with the options (I-IV) given below.

R
(I) The site of ultrafiltration.

A
A B (II) Collection and concentration of urine.
(Experiment) (Control)

M
(III) The main area for the reabsorption of glucose and
1. Why germinating seeds produce heat in the thermos

U
amino acids.
flask B but not in A?
2. What will happen if we keep dry seeds in thermos
flask B? K
(IV) Mainly responsible for the maintenance of the pH
of blood.
T
N
Case Study-IV
A

The diagram given below represents an organ system in D


the human body. Study the same and answer the questions E
SH

AC
that follow:
A

1 B
PR

(a) (I)-A, (II)-E, (III)-C, (IV)-D


2
(b) (I)-A, (II)-B, (III)-C, (IV)-A
R.

(c) (I)-B, (II)-A, (III)-C, (IV)-E


3
D

4 (d) (I)-E, (II)-B, (III)-D, (IV)-A


5

Life Processes 45
Competitive Level
Life Processes?
1. How life on the Earth depends on carbon based molecules?
Life on Earth is fundamentally dependent on carbon-based molecules, and carbon is a key element in the organic
compounds that form the building blocks of living organisms. Here are several ways in which life on Earth relies on
carbon-based molecules:
Organic Molecules:

 Carbon is unique in its ability to form diverse and complex molecules due to its tetravalent nature, allowing it to bond
with up to four other atoms. This property is crucial for the formation of organic molecules, including carbohydrates,
lipids, proteins, and nucleic acids—the fundamental components of living cells.

R
Cellular Structure:

A
 The structural framework of living organisms is composed of carbon-containing molecules. Carbohydrates provide
energy and serve as structural components, lipids form cell membranes, proteins performs various functions within

M
cells, and nucleic acids (DNA and RNA) carry genetic information.

U
Energy Storage:

K
 Carbon-based molecules, such as glucose and other carbohydrates, serve as energy storage molecules in the living
organisms. Through processes like cellular respiration, organisms can break down these carbon compounds to release
T
energy for various cellular activities.
N

Genetic Information:

A

 Carbon is a fundamental element in the structure of DNA and RNA, the molecules that carry and transmit genetic
SH

information. The ability of carbon to form stable and varied structures is essential for encoding the diversity of
genetic instructions that govern life processes.
A

Biomolecules and Metabolism:


PR

 Proteins, enzymes, and other biomolecules that regulate and catalyze metabolic processes are carbon-based. These
molecules facilitate the chemical reactions necessary for life, including the synthesis and breakdown of other organic
compounds.
R.

Diversity of Life:
D

 The vast diversity of life on Earth is made possible by the versatility of carbon. Carbon can form a wide array
of compounds through various bond arrangements, leading to the incredible diversity of organic molecules and,
consequently, the diversity of living organisms.
Carbon Cycle:

 The carbon cycle, which involves the movement of carbon through the atmosphere, oceans, soil, and living organisms,
is crucial for maintaining the balance of carbon on Earth. Plants, through photosynthesis, take in carbon dioxide and
convert it into organic compounds, providing the foundation for the entire food chain.
2. Why is molecular movement absent in virus when it is outside the host cell?
Viruses are acellular, that is, they contain no cytoplasm or cellular organelles. They carry out no metabolism on their
own and must replicate using the host cell's metabolic machinery. In other words, viruses don't grow and divide. Instead,
new viral components are synthesized and assembled within the infected host cell.

46 Class-X BIOLOGY PW
Nutrition in Plants: Photosynthesis
Historical Aspects of Photosynthesis
A record of the early studies and experiments conducted which served as landmarks in the field of photosynthesis are
listed below.
‰ The first scientist to establish the fact that gaseous constituents of air and components of light, both contributes towards
the building up of the plant body and photosynthetic nourishment occurring in plants plant was Stephan Hales (1927).
‰ He is also considered to have discovered the process of photosynthesis and is called as ‘Father of Plant Physiology’.

Priestley Experiment
1. Joseph Priestley (1770) demonstrated that purification of the respired air released by animals was carried out by plants.
In a series of experiments involving bell jar, candle, mint (pudina) plant and rat, the essential role of air in the growth,
development and survival of green plants was demonstrated.
Priestley noted that the candle burning on its own in a closed space such as a bell jar, gets extinguished very quickly.
Burning of the candle results in the generation of impure air which was labelled as phlogiston. When a live mouse was

R
placed along with the candle in the closed bell jar, it resulted in the death of the mouse due to suffocation. However,

A
on placing a live mint plant in the closed bell jar containing the burning candle and live mouse. it resulted in both the
plant and mouse surviving while the burning of candle continues. The final conclusion achieved was that the candle

M
which requires air continued burning and/or the animal that breathes air remained alive due to the presence of plants.
The hypothesis proposed by Priestley was that the restoration of the phlogiston to its pure form, which was termed as

U
dephlogiston was achieved by plants. This pure air or dephlogiston was whatever breathing animals and burning candles

K
remove. Oxygen as an essential gas for survival was also established by Priestley (1774).
Conclusion: A and B Animal does not survive as candle used up the oxygen. C and D - The plant replenishes oxygen
T
by using up the carbon-dioxide and hence the animal survives.
N
A
SH
A
PR
R.
D

Fig. 35: Priestley Experiment


2. Jan Ingenhousz stated that green leaves give out dephlogisticated air (air rich in oxygen) in presence of sunlight while
in the absence of sunlight (in the dark) they give out phlogisticated air (air rich in CO2) and make the air ‘impure’.
3. Julius von Sachs (1854) discovered that the green parts in plants is where glucose is synthesized and that glucose is
usually stored as starch.
4. T.W. Engelmann worked using on Cladophora and Spirogyra. He noted that when light is split using a prism and used
to illuminate the algae, the organisms aggregate in the blue and red regions.
Thus Engelmann concluded that the red and blue light of the visible spectrum generate more photosynthetic activity than
other parts of the visible spectrum.
5. Cornelius Van Niel, carried out experiments using green and purple sulphur bacteria. He showed that hydrogen released
from suitable oxidisable compounds reduces CO2 to carbohydrates. He also put forth that water is the source of oxygen
in photosynthesis.

Life Processes 47
Factors of affecting Photosynthesis
The factors which affect the rate of photosynthesis are light, CO2, H2O and temperature.
(a) Effect of light: In the dark photosynthesis does not take place. If Intermembrane Inner
Outer membrane Stroma
light intensity is slowly increased then the rate of photosynthesis space
membrane (aqueous fluid)
increases to optimum point after which it becomes constant. Very
high light intensity slows down photosynthesis. Chlorophyll
absorbs blue and red light mostly. Usually plants show high rate
of photosynthesis in red light than in blue light. The green light
has minimum effect.
(b) Effect of CO2: The amount of CO2 available to a plant if
increased causes increase in rate of photosynthesis, but after
Granum Lumen
a point it becomes constant. This point is called as Saturation (stack of thylakoids) Thylakoid Lamella (inside of thylakoid)
point.
Fig. 36: Diagrammatic representation of an electron
(c) Effect of H2O: If the amount of water available to plant decreases, micrograph of a section of chloroplast
the rate of photosynthesis decreases becauses deficiency of water

R
causes closing of stomata which stops the gaseous exchange.

A
(d) Effect of temperature: As the temperature increases, the rate of photosynthesis increases upto a certain temperature.
At high temperature, the rate of photosynthesis decreases because enzymatic functions get adversely affected at very

M
high temperature.

U
Respiration in Plants
K
T
Compensation Point in Plant Respiration
N
Rate of photosynthesis

In the depicted graph, the compensation point is identified as the

Photosyntheic rate
A

Photosyntheic rate
juncture where the rates of cellular respiration and photosynthesis Saturation point
SH

are in equilibrium. As light intensity escalates, this equilibrium

Net

Total
is achieved. Beyond the compensation point, the rate of
photosynthesis sees a rise until reaching the saturation point.
A

Unlike photosynthesis, which is contingent on sunlight intensity,


Light intensity
PR

respiration maintains a constant rate unaffected by light levels.


Compensation point
Heterotrophic Nutrition in Plants Respiration rate
R.

Fig. 37: Compensation point in plant respiration


Certain plants fail to synthesize their own organic nutrients from
D

inorganic substances. These are thus dependent on some other external sources for their nutrition. Such plants are called
heterotrophic plants and mode of nutrition is called heterotrophic nutrition.
‰ Saprophytes:
 Those plants which grow and live on rotting dead leaves e.g., Indian pipe, Coral root and Monotropa.
‰ Parasitic Plants:
 Dodder (Cuscuta) known locally as Amarbel/Akashbel is a parasitic plant that lacks both chlorophyll and leaves. It
is a yellow colour climber that attaches itself to the host. It gives out haustoria or the suckers that get attached to the
phloem of the host and derive nutrition.
 Cuscuta does not have roots in the mature condition. It produces bunches of whitish or yellowish bell shaped flowers.
‰ Insectivorous Plants:
 There are certain plants which are autotrophic but show heterotrophic nutrition as well in order to supplement the
deficiency of a particular mineral (e.g. nitrogen ) in the soil.

48 Class-X BIOLOGY PW
 Insectivorous plants are the best examples. They feed on insects. They are generally found in nitrogen deficient
habitats and hence to compensate the loss, they use, insects as a source of nitrogen.
 For example, Pitcher plant (Nepenthes); Sundew (Drosera); Venus flytrap (Dionaea); Bladderwort (Utricularia).
‰ Symbiotic relationship: Mutually beneficial association in which organisms live together and share shelter and nutrients
is called symbiotic relationship. For example, Mycorrhiza and lichen.
 Mycorrhiza: The symbiotic association of the mycelium of a fungus with the roots of conifers and orchids.
The tree provides nutrients to the fungus and, in return, receives help from it to take up water and nutrients from the
soil. This association is very important for the tree as well as the fungus.
 Lichens: Association in which a chlorophyll-containing partner (alga) and a fungus live together. The fungus provides
shelter, water and minerals to the alga and in return, the alga provides food which it prepares by photosynthesis.

Nutrition in Human Beings


Types of Digestion

R
1. Intracellular Digestion (Intra: inside): All the five steps of nutrition occur inside the cell itself, as in Amoeba,

A
Paramecium and other unicellular organisms. Amoeba projects out pseudopodia to capture the food & forms a food

M
vacuole.
Food vacuoles are temporary structures and every time the Amoeba feeds, a new food vacuole is produced. The food

U
is digested inside the vacuole.

K
2. Extracellular digestion (Extra: outside): Digestion occurs outside the cell. Many animals carry out extracellular
digestion. They have either a cavity, a tube, or an alimentary, canal which receives the ingested food. Food is acted
T
upon by digestive enzymes and the products of digestion are absorbed back into the cells lining the cavity.
N
A

Disorders of the Digestive system


SH

‰ Vomiting: It is the ejection of stomach material through the mouth. The vomit centre regulates this reflex action in the
medulla. A feeling of nausea cause vomiting.
A

‰ The inflammation of the intestinal tract is the most common problem due to viral or bacterial infections. Also, the
infections can be caused due to parasites of the intestine such as tape worm, thread worm, round worm, pin worm, hook
PR

worm, etc.
‰ Diarrhoea: The abnormal frequency of bowel movement and increased liquidity of the faecal discharge is called
R.

diarrhoea. It diminishes the absorption of food.


D

‰ Constipation: In constipation, the faeces are retained within the rectum as the bowel movements take place irregularly.
‰ Ulcers: Ulcers develop due to infection from a bacterium Heliobacter pylori. This bacterium can survive in acidic
environments of the stomach and cause degradation of the stomach or oesophagus wall.

Respiration in Human Beings


Glycolysis
‰ Takes place in cytoplasm and is a universal pathway in all biological systems.
‰ In this, one molecule of glucose is reduced to two molecules of pirovate.
‰ 2 ATP molecules are formed from the breakdown of one glucose molecule.
‰ Two molecules of NADH2 (energy carriers) are formed.

Life Processes 49
Link Reaction
‰ Pyruvic acid (3C) formed in the process of glycolysis is converted into molecules of Acetyl-Coenzyme A (2C).

‰ This step is called gateway step or link reaction because acetyl-coA acts as a connecting link between glycolysis and
krebs cycle. Two molecules of CO2 are released during this process.

Krebs Cycle
‰ Krebs cycle is a series of chemical reactions that occurs in mitochondrial matrix‑ of all living cells that utilize oxygen
during the process of cellular respiration.
‰ Acetyl CoA molecules enters the mitochondria and the Acetyl part of Acetyl Co-A is completely oxidized through this
cyclic process.
‰ Complete oxidation of one molecule of Acetyl-CoA produces 2CO2, energy carriers ( 3NADH2, 1FADH2 ) and 1 ATP.

R
A
M
U
K
T
N
A

Fig. 38: Krebs Cycle in mitochondria


SH

Electron transport system & Oxidative Phosphorylation


A

‰ The electron transprt chain is a series of four protein complexes that couple redox reactions, creating an electrochemical
PR

gradient that leads to the creation of ATP.

Molecules of energy carriers ( NADH2 and FADH2 ) produced in glycolysis give away their energy-rich electrons through
R.

a series of carriers to oxygen.


D

‰ Oxygen accepts these electrons and H+ ions to form metabolic water.

‰ This process also creates H+ ion gradient.

‰ H+ gradient is utilized by F0-F1 particle to create ATP.

‰ Maximum ATP (34) are produced by this step.

‰ Since addition of phosphate groups (phosphorylation) happens due to energy made available by loss of electrons from
energy carriers (oxidation), this is called “Oxidative Phosphorylation”.

Respiratory Quotient
It is a ratio of volume of carbon dioxide evolved to the volume of oxygen consumed in the process of respiration.
Respiratory quotient is also known as respiratory ratio.

50 Class-X BIOLOGY PW
It can be represented as:
Volume of CO 2 evolved
RQ =
Volume of O 2 consumed

Value of the respiratory quotient depends on the type of respiratory substrate used during respiration. e.g., RQ value in
case of carbohydrates: When carbohydrates are used as substrate and are completely oxidised, the RQ will be 1, because
equal amount of CO2 and O2 are evolved and consumed.
C6H12O6 + 6O2 → 6CO2 + 6H2O + energy
RQ = 6CO2/ 6O2 = 1
For Fats, RQ = 0.7
For Proteins, RQ ∼ 0.9

Disorders Related to Respiration

R
Generally, respiratory disorders are of two types: Infectious and Chronic. Most pulmonary infections are due to bacteria

A
or viruses. Chronic diseases are persistent and long-lasting. Some of the respiratory disorders are discussed in table below.

M
Disorders Cause Symptoms Cure

U
Hypoxia Severe asthma attacks and lung Cough, fast heart rate, rapid Medical treatment.
damage due to conditions like breathing, sweating, etc.
anaemia, cyanide poisoning and
other heart problems.
K
T
Asphyxiation Strangulation, drowning, orloss of consciousness coughing, Artificial respiration is helpful
N

any other obstruction in the


dyspnea. in certain cases (CPR) or
A

respiratory tract. Cardiopulmonary resuscitations.


SH

Emphysema Cigarette smoking, air pollution,


Shortness of breath, coughing, Antibiotic therapy to
chemical fumes. fatigue remove infection and use of
bronchodilator drugs.
A

Bronchial Asthma Allergens stimulate release of Coughing, wheezing (breathing Antibiotic therapy to remove
PR

histamine from the mast cells. heavily), difficulty in breathing infection, use of bronchodilator
and excess of mucus released drugs and oxygen therapy.
by the respiratory tract that may
R.

clog bronchi and bronchioles.


D

Transportation in Plants
Osmotic Pressure (OP)
Osmotic pressure (OP) of a solution is equivalent to the pressure, which must be exerted upon it, to prevent the flow of
solvent into it, across a semipermeable membrane. It is usually measured in pascals or bars or atmospheres.

Turgor Pressure
Turgor pressure is the force within the cell that pushes the plasma membrane against the cell wall. It is also called
hydrostatic pressure, and is defined as the pressure in a fluid measured at a certain point within itself when at
equilibrium.

Life Processes 51
Wall Pressure
The pressure placed on the contents of a plant cell by the cell wall that is equivalent to and opposite to the force of the
turgor pressure is referred to as wall pressure.

DPD or Suction Pressure


Suction pressure is also called Diffusion Pressure Deficit. If some solute is dissolved in solvent, its diffusion pressure
decreases. The difference between diffusion pressure of pure solvent and solution is called diffusion pressure deficit
(DPD).

Water Potential
Water potential is the difference between the free energy of water molecules in pure water and the energy of water in any
other system e.g., water in a solution or in a plant cell or tissue. At atmospheric pressure, pure water has a water potential
of zero. The presence of solute reduces the free energy of water and thus decreases the water potential (negative value).

R
Therefore, the water potential of a solution is always less than zero. Water always moves from the area of high water

A
potential or high energy to the area of low water potential or low energy.The value or water potential can be measured
in terms of bars,pascals or atmospheres.

M
Transpiration Pull and Cohesion-Tension Theory

U
K
The main force responsible for the movement of water in the upward direction is transpiration pull. It is generated in the
leaves which pulls the water column filled in the xylem tracheids and vessels. The loss of water in the form of water
T
vapours from the aerial region of a plant is called transpiration. Transpiration mainly occurs via stomata (about 80% to
N

90%), but it may also take place through the cuticle (9%) and lenticels (1%).
A

During day time, cohesion force (attractive force between water molecules) and transpiration pull becomes the main
SH

driving force responsible for water movement in upward direction in the xylem from roots to leaves.
As transpiration happens at leaves end, the route of transport is unidirectional. Water always moves up and never
A

downwards.
PR

Transpiration is a Necessary Evil


R.

Transpiration is considered to be a necessary evil because it leads to the loss of water from plants but provides the plant
with several advantages as well.
D

‰ Creates transpirational pull for transport


‰ Supplies water for photosynthesis.
‰ Transports minerals from the soil to all parts of a plant.
‰ Cools the surface of the leaves (due to the evaporation of water).
‰ Keeps the cells turgid; hence, maintains their shape.

Guttation
This loss of water in the form of water droplets is called guttation.
When the nights are humid, the rate of transpiration reduces. The cell sap moves out of the cell via specialized pores
along the leaf margin known as hydathodes.

52 Class-X BIOLOGY PW
Comparision of Guttation and Transpiration
S.No. Guttation Transpiration
1. Occurs with the help of hydathodes. Occurs via stomata, cuticle or lenticel.
2. Xylem sap full of solutes moves out in form of water droplets. Pure water is released in form of water vapour.
3. Observed in herbaceous plants. Observed in all plants.

Comparision of Transpiration and Evaporation


Transpiration Evaporation
It is a biological process. It is a physical process
Transpiration is a slow process. Comparatively a fast process.
Occurs in living tissues. Involves non-living matter.
In the process of transpiration, water is lost from plant cells In this process, water is lost from the surface of plant parts
Transpiration makes the surface of leaves and young stems Evaporation provides dryness to the free surface.
wet and protects them from sunburn.

R
A
Transportation in Animals

M
Leukocytes
Types of WBCs

U
White blood cells can be further classified as granulocytes
and agranulocytes on the basis of the presence or absence
of granules. K
T
Granulocytes Agranulocytes
1. Agranulocytes: (Granular leukocytes) (Not granular leukocytes)
N
Granules are absent in such cells and can be classified
A

further into lymphocytes and monocytes.


 Lymphocytes are non-phagocytic in nature and
SH

produce antibodies which destroy microbes and kill


other foreign particles. Lymphocytes
A

 Several studies suggest that lymphocytes also heal


injuries. Eosinophils Basophils Neutrophils B-Lymphocytes T-Lymphocytes
PR

 Monocytes are phagocytic as they can engulf bacteria


and other damaged cells.
R.

 Monocytes are twice the size of red blood cells and Monocytes
have two lobed nuclei. Fig. 38: Types of White blood cells
D

2. Granulocytes
Granules are present in these cells and have an irregular lobed nucleus. On basis of stains they take, they can be further
divided into three types: Eosinophil, Basophils and Neutrophils.

Heart sounds
Heart produces mainly two sounds Lubb and Dubb.
S.No. Lubb Dubb
1. Sound lubb is low pitched Sound dubb is sharp and high-pitched
2. Occurs for longer duration i.e., for 0.15 seconds. Occurs for duration of 0. 1second.
3. It occurs due to closure of bicuspid valve and tricuspid It occurs due to closure of semilunar valve during
valve during contraction of ventricles. ventricular diastole.
Table 3.6 Comparison of Heart Sounds- Lubb and Dubb.

Life Processes 53
Heartbeat
‰ The heart shows regular contraction and relaxation.
‰ In a child, the heart beat varies from 90 to 100 times a minute. In an adult, heart beat varies from 70-72 times per minute.
‰ Heartbeat increases when you do some extra work, such as running, cycling or when you are excited or under stress,
This can be felt as an increase in the thumping or throbbing of the heart.
‰ Heart beat originates at the Sino-Atrial Node or S.A Node which is a modified part of the muscular wall of the right
atrium chamber.
‰ Since, sino-atrial node initiates and regularizes the heartbeat, it is also called the pacemaker. The pacemaker is influenced
by nerves, hormones, CO2 and O2 content of blood, heat, etc.
‰ The device that amplifies the sound of a person's heart beat is called a stethoscope and consists of a chest piece that
holds a sensitive diaphragm.
‰ The electrocardiograph (ECG) is a machine that records the electrical activity of the heart over a period of
time using electrodes placed on skin.

Cardiac Cycle

R
‰ One complete contraction (systole) and relaxation (diastole) of the heart is called a heartbeat. The sequence of events

A
which takes place during the completion of one heartbeat is called the cardiac cycle.

M
Steps of cardiac cycle:

U
‰ Joint diastole: During the time when the muscles of all four chambers of the heart are relaxed, the blood return to the
heart under low pressure and enters the two atria. Blood from large veins, called vena cava, pours into right atrium. This
K
blood comes from head, upper body parts and lower body parts where oxygen has been used up and the blood is free
from oxygen, i.e., deoxygenated. At the same time, the pulmonary vein from lungs pours oxygenated blood into the left
T
atrium. Thus, the oxygenated blood enters the left atrium.
N

‰ Atrial systole: As the right and left atria fill with blood, pressure in them rises so that the valves between left atrium and
A

left ventricle (bicuspid valve) and between right atrium and right ventricle (tricuspid valve) open and the atria contract.
Atrial contraction forces pumping of deoxygenated blood from right atrium into the right ventricle through tricuspid
SH

valve and oxygenated blood from left atrium into left ventricle through bicuspid valve.
‰ Ventricular systole: It involves contraction of ventricles. During contraction of ventricles, the deoxygenated blood from
A

right ventricle flows to the lungs through pulmonary artery and the oxygenated blood from left ventricle is distributed
to all the parts of the body through the largest artery, called aorta.
PR
R.

Knowledge Hub
D

Heart Attack
Heart attack is a medical condition when the supply of blood gets blocked due to blood clot or any other blockage.
Many a times cholesterol get deposited within the blood vessels, supplying blood towards heart and clog the
pathway of blood. This condition is commonly called coronary heart disease (CHD). CHD is a common cause for
heart attack to happen.
Heart failure means the state of heart when it is not pumping blood effectively enough to meet the needs of the
body. It is sometimes called congestive heart failure because congestion of the lungs is one of the main symptoms
of this disease. Heart failure is not the same as cardiac arrest (when the heart stops beating) or a heart attack
(when the heart muscle is suddenly damaged by an inadequate blood supply).

54 Class-X BIOLOGY PW
1. The internal (cellular) energy reserve in autotrophs (iii) Mucus
is called ________. (iv) Trypsin
(a) Glycogen (b) Starch
(a) (i) and (ii) (b) (i) and (iv)
(c) Protein (d) Fatty acid
(c) (ii) and (iii) (d) (i) and (iii)
2. The first step in photosynthesis is:
(a) Conversion of light energy into chemical energy. 8. When a few drops of iodine solution are added to
(b) Reduction of carbondioxide. rice water, the solution turns blue- black in colour.
(c) Absorption of light energy by chlorophyll. This indicates that rice water contains:
(d) Formation of carbohydrates. (a) Fats (b) Glycogen
3. Which of the following events in the mouth cavity (c) Starch (d) Simple proteins

R
will be affected if salivary amylase is absent in the 9. Which of the following statements are correct.

A
saliva?
(i) Saliva has salivary amylase to digest protein.

M
(a) Starch hydrolysis into sugars.
(ii) The pH of human saliva is 3.8.
(b) Proteins hydrolysis into amino acids.

U
(c) Absorption of vitamins. (iii) Fats get digested completely in the small intestine.
(d) Fats breaking down into fatty acids and glycerol.
4. From the options given, identify the event that
K
(iv) The pH of human saliva is 6.5 to 6.8 PH.
(a) (i) and (iv) (b) (i) and (ii)
T
doesn't occur in the photosynthesis. (c) (i) and (iii) (d) (iii) and (iv)
N

(a) Oxidation of carbon to carbon dioxide.


10. The given figure is a demonstration of an
A

(b) Absorption of light energy by the chlorophyll.


experiment to show that carbon dioxide is essential
SH

(c) Reduction of carbon dioxide to carbohydrates.


for photosynthesis. What is the substance “X” , kept
(d) Conversion of light energy into chemical energy.
in petri dish?
A

5. In which order do these events occur in human


Bell jar
nutrition?
PR

(a) Digestion → ingestion → absorption → assimilation


(b) Digestion → ingestion → assimilation →
R.

absorption
(c) Ingestion → digestion → absorption → assimilation
D

(d) Ingestion → digestion → assimilation →


‘X’
absorption crystals in
petri dish
6. The enzymes pepsin and trypsin are secreted
respectively by: (a) Potassium hydroxide
(a) Stomach and pancreas (b) Sodium bicarbonate
(b) Salivary gland and stomach (c) Sodium carbonate
(c) Liver and pancreas
(d) Potassium sulphate
(d) Liver and salivary gland
11. The diagram shows an experiment to investigate the
7. Which of the following is not a digestive enzyme
contained in the pancreatic juice? balance between respiration and photosynthesis.
(i) Amylase In which tube are photosynthesis and respiration
(ii) Hydrochloric acid taking place at the same time?

Life Processes 55
17. Which of the following changes can be seen in
diaphragm during inspiration?
(a) Expands (b) Contracts
(c) No effect (d) None of these
18. Read the following statements about glycolysis
(a) A will have both photosynthesis and respiration (breakdown of glucose) and select the option that
while B will only have photosynthesis shows the CORRECT statements about it.
(b) All tubes will have respiration (i) Occurs in cytoplasm.
(c) C will have photosynthesis only and D will have (ii) Pyruvic acid is formed by the breakdown of
respiration only glucose in the number of steps.
(d) A and D will have respiration only (iii) H ⊕ is released in this breakdown through the
12. In which of the following animals, respiration does oxygen supplied by forming H2O.
not require any respiratory organ? (iv) Very little energy is released.
(a) Fish (b) Frog (a) (i) and (ii) (b) (ii) and (iii)

R
(c) Cockroach (d) Earthworm (c) (i), (ii) and (iv) (d) (i) and (iv)

A
13. Consider the following statements: 19. Which of the following expression depict the

M
(I) Carbon dioxide taken in through stomata is used respiratory quotient?
in photosynthesis. (a) RQ → volume of CO2 absorbed/ volume of O2

U
(II) Oxygen taken in through stomata is used in consumed.
respiration.
(III) Water vapour taken in through stomata is used in K(b) RQ → volume of CO2 evolved/ volume of O2
consumed.
T
transpiration. (c) RQ → volume of O2 evolved/ volume of CO2
N

Select the correct statements from the following. consumed.


A

(a) (I) and (II) (b) (I) and (III) (d) RQ → volume of O2 absorbed/ volume of CO2
SH

(c) (II) and (III) (d) (I), (II) and (III) consumed.
14. Which process occurring in human body does not 20. Which of the following statement(s) is (are) true
A

involve energy from respiration? about respiration?


(a) Contraction of heart muscle. (i) During inhalation, ribs move inward and diaphragm
PR

(b) Diffusion of oxygen from the alveoli into the blood. is raised.
(c) Digestion of bread. (ii) In the alveoli, exchange of gases takes place i.e.,
R.

(d) Maintaining a constant body temperature. oxygen from alveolar air diffuses into blood and
carbon dioxide from the blood into the alveolar air.
D

15. Which of the given step is common in both aerobic


(iii) Haemoglobin has a greater affinity for carbon
respiration and anaerobic respiration?
dioxide than oxygen.
(a) Conversion of pyruvate to carbon dioxide, water
and energy. (iv) Alveoli increase surface area for exchange of gases.
(b) Conversion of glucose to pyruvate. (a) (i) and (iv) (b) (ii) and (iii)
(c) Conversion of pyruvate to lactic acid and energy. (c) (i) and (iii) (d) (ii) and (iv)
(d) Conversion of pyruvate to ethanol, carbon dioxide 21. During contraction, what prevents the backflow of
and energy. blood inside the heart?
16. Which of the following organ is the respiratory (a) Valves in the heart
organ in insects? (b) Thick muscular walls of ventricles
(a) Lung (b) Tracheal tubes (c) Thin walls of atria
(c) Skin (d) Gills (d) Thick walls of atria

56 Class-X BIOLOGY PW
22. Which of the following is correct regarding 25. Which among the following procedures is used for
capillaries? cleaning the blood of a person by separating the
(a) Thick walled blood vessels that carry blood from waste substance from it?
the hearts to all the parts of the body. (a) Kidney transplant (b) Blood transfusion
(b) Thin walled blood vessels and are extremely (c) Dialysis (d) Hydrolysis
narrow tubes which connect arteries to veins.
26. The waste materials in plants are stored in the form
(c) Thin walled blood vessels which carry blood from of:
all the parts of body to the heart.
(a) Water (b) Gums and resins
(d) Thick walled blood vessels which carry blood from
(c) Minerals (d) Carbohydrates
all the parts of body to the brain.
27. What is the correct pathway of transport of urine in
23. Agranulocytes are:
our body.
(a) Lymphocytes, Monocytes
(a) Kidney → ureter → urethra → urinary bladder
(b) Lymphocytes, Basophils
(b) Kidney → urinary bladder → urethra → ureter
(c) Eosinophils, Basophils
(c) Kidney → ureters → urinary bladder → urethra
(d) Eosinophils, Monocytes

R
(d) Urinary bladder → kidney → ureter → urethra
24. Your blood alternates between your body’s

A
28. What is the function of the glomerulus and
pulmonary circuit and systemic circuit through
Bowman's capsule of the nephron?

M
various vessels and chambers. Below is a list of
vessels and structures that are associated with your (a) Filtration of blood

U
heart. What is the correct order for the flow of (b) Re-absorption of ions from blood
blood entering from the systemic circulation?
1. Right atrium 2. Left atrium K(c) Re-absorption of hormones from blood
(d) Re-absorption of water from blood
T
3. Right ventricle 4. Left ventricle 29. Which substances out of the following in the dialysis
N
5. Venae cava 6. Aorta fluid should be at a lower concentration than in the
A

7. Pulmonary trunk 8. Pulmonary veins blood of patient?


(a) 1, 7, 3, 8, 2, 4, 6, 5 (a) Salt and urea
SH

(b) 1, 2, 7, 8, 3, 4, 6, 5 (b) Glucose and amino acid


(c) 5, 1, 3, 8, 7, 2, 4, 6 (c) Glucose and urea
A

(d) 5, 1, 3, 7, 8, 2, 4, 6 (d) Glucose and salts


PR
R.

Competitive Corner
D

1. Identify the correct statements related to dark 2. Mode of nutrition in cuscuta is: [Haryana 2018]
reaction of photosynthesis.  [2017 Kerala] (a) Saprophytic
(i) Takes place in the stroma of chloroplasts (b) Autotrophic
(ii) Formation of ATP
(c) Parasitic
(iii) Formation of glucose
(d) Insectivorous
(iv) Evolution of oxygen
(a) (i) and (ii) 3. By whom was dark reaction of photosynthesis
(b) (ii) and (iii) discovered? [Orrisa 2018]
(c) (i) and (iii) (a) Calvin (b) Hill
(d) (ii) and (iv) (c) Johnson (d) Mitchell

Life Processes 57
4. Deficiency of one of the under-mentioned vitamins III. Lymphocyte - Initiates blood clotting
causes cracking of lips in human beings: IV. EosinoPhil - Associated with allergy
 [2017 Jharkhand] (a) I and II (b) II and III
(a) Vitamin A (b) Vitamin B12 (c) I and IV (d) III and IV
(c) Vitamin K (d) Vitamin C
11. What will happen to the body of an adult human
5. Which vitamin is produced by Escherichia coli being if his spleen is removed: [Jharkhand 2017]
present in out intestine? [Orrisa 2017] (a) RBC production will be reduced.
(a) B6 (b) B7 (b) Antibodies production will be less.
(c) B9 (d) B12 (c) WBC production will be less.
6. Which one of the following is the function of the (d) Filtration of dead RBCs would not be possible.
enzymes of pancreatic Juice? [Punjab 2017] 12. Bicuspid valve is present in the human heart in
(a) Trypsin digests protein and lipase digests between which of the following? [Delhi 2017]
carbohydrates. (a) Right atrium and right ventricle
(b) Trypsin digests emulsified fats and lipase digests (b) Left atrium and left ventricle
proteins.

R
(c) Right and left atria
(c) Tryspin and lipase digest fats.

A
(d) Left atrium and systemic aorta
(d) Trypsin digests proteins and lipase digests
13. Match the column-I with those which are most

M
emulsified fats. appropriate in Column-II. [West Bengal 2018]

U
7. Identify the correct sequence for process of energy
Column-I Column-II
production from carbohydrates.
 [2018-19, Maharashtra] KP. Lymphatic System (i) Carries oxygenated
blood
T
(a) Carbohydrates → Glycolysis → Pyruvic acid → Q. Pulmonary Vein (ii) Immune response
N
AcetylCoA → Krebs cycle → CO2 + H2O + energy
R. Thrombocytes (iii) Re-join the
A

(b) Carbohydrates → Glycolysis → Pyruvic acid → tissue fluid to the


Krebs cycle → AcetylCoA → CO2 + H2O + energy circulatory system
SH

(c) Carbohydrates → Glycolysis → AcetylCoA → S. Lymphycytes (iv) Coagulation of


Pyruvic acid → Krebs cycle → CO2 + H2O + blood
A

energy (a) P-(ii), Q-(i), R-(iii), S-(iv)


PR

(d) Carbohydrates → Glycolysis → AcetylCoA → (b) P-(iii), Q-(i), R-(iv), S-(ii)


Krebs cycle → Pyruvic acid → CO2 + H2O + (c) P-(iii), Q-(i), R-(ii), S-(iv)
energy (d) P-(ii), Q-(i), R-(iv), S-(iii)
R.

8. The habitat related with presence of sunken stomata 14. Assertion: Blood in veins and lymph in lymphatic
D

in leaves is: [Rajasthan 2019] vessels do not flow reverse.


(a) Hydrophytic (b) Mesophytic Reason: It is because, the pressure is very high in
(c) Xerophytic (d) Cryophytic veins and lymphatic vessels.
9. During ventricular systole: [West Bengal 2017] [Andhra Pradesh 2020]
(a) Atrial systole occur. (a) A and R both are correct.
(b) The atrio-ventricular valves are closed. (b) A is correct and R is wrong.
(c) The pressure inside the ventricles is less than atria. (c) A is wrong and R is correct.
(d) The mitral valve is closed. (d) A and R both are wrong.
15. The excretory organs in Earthworm are known as:
10. Identity the wrongly matched pairs. [Kerala 2017]
 [Jharkhand 2017]
I. Monocyte - Actively phagocytic
(a) Malphigian cells (b) Renal cells
II. Thrombocyte - Produces antibodies
(c) Nephridia (d) Flame cells

58 Class-X BIOLOGY PW
16. ADH takes part in:  [WB 2018] (b) Air from mouth contains CO2 which is utilized
(a) Water retention in urine. in photosynthesis and hence, increase in O 2
(b) Na+ re-absorption. production.
(c) Reducing urea formation. (c) Bacteria from mouth will infect plant and hence,
reduction in O2 production.
(d) Absorption of water from urine.
(d) Water is already in contact with air and hence, air
17. Which of the following situations is likely to cause from mouth will have no effect.
muscle cramps? [NTSE 2012] 20. Which one of the following is known as energy
(a) Glucose No Oxygen Lactic acid currency of cell? [NTSE 2014]
(b) Glucose Oxygen Lactic acid (a) Adenosine diphosphate
(c) Glucose No Oxygen Alcohol + CO2 (b) Adenosine triphosphate
(d) Glucose Oxygen CO2 + H2O (c) Pyruvate
(d) Glucose
18. Select the correct schematic representation of blood
circulation in humans from the following options: 21. Movement of food in digestive tract is due to:
 [NTSE 2012]  [NTSE 2014]

R
Body (a) Concentration gradient

A
Pulmonary (b) Secretions
(a) Veins Heart Lungs

M
Vein (c) Peristalsis
Pulmonary Artery
(d) Villi

U
Pulmonary Artery 22. The phenomenon of normal breathing in a human

(b) Body
Veins
Heart Lungs Kbeing comprises: [NTSE 2015]
(a) An active inspiratory and a passive expiratory
T
Arteries
phase.
N
Pulmonary Vein

(b) A passive inspiratory and an active expiratory


A

Arteries
Pulmonary
(c) Body Heart Lungs phase.
SH

Artery
Veins (c) Both active inspiratory and expiratory phase.
Veins Veins Veins Artery (d) Both passive inspiratory and expiratory phase.
A

(d) Body Heart Lungs Heart Heart


23. Which one of the following statement is true with
PR

19. Photosynthesis in an aquatic plant was measured by


counting the number of O2 bubbles coming out of respect to photosynthesis? [NTSE 2015]
the cut end of the plant. What will happen to O2 (a) Oxygen evolved during photosynthesis comes
R.

production if you use a pipe to blow air from your


from CO2.
mouth into water in the beaker? [NTSE 2013]
D

(b) Chlorophyll a is the only photosynthetic pigment


in plants.
(c) Photosynthesis occurs in stem of some plants.
(d) Photosynthesis does not occur in red light.
24. Suppose a mutant of a photosynthetic alga has
dysfunctional mitochondria, it would affect its
ability to perform: [NTSE 2016]
(a) Glycolysis
(b) Anaerobic respiration
(a) Air from mouth contains O2 which is being added (c) Aerobic respiration
to the plant and hence, increase in O2 production. (d) Photosynthesis

Life Processes 59
25. Cow has a special stomach as compared to that of a 31. A student was performing an experiment to
lion in order to: [NTSE 2016] understand the enzyme-substrate reaction. The
(a) Absorb food in better manner. student measured the formation of coloured product
using a colorimeter. The student plotted the graph
(b) Digest cellulose present in the food.
below which shows the reaction rate versus the
(c) Assimilate food in a better way.
substrate concentration.
(d) Absorb large amount of water.
26. Pancreas is composed of: [NTSE 2016]
(a) Only exocrine cells
(b) Only endocrine cell

Reaction rate
(c) Both endocrine and exocrine cells
(d) Nephrons
27. Pancreatic juice contains more than one enzyme.
Which among the following combination is correct?
Substrate concentration

R
 [NTSE 2017]
Following interpretations were drawn by the

A
(a) Pepsin and Lipase (b) Amylase and Pepsin
student:

M
(c) Pepsin and Trypsin (d) Trypsin and Lipase
A. The higher concentration of substrate acts as an
28. A farmer made an observation in a backwater paddy

U
enzyme inhibitor.
field of coastal region that the paddy plants wilt
B. It is sigmoidal curve with sharp transition from low
during noon onwards everyday but appear normal
next morning. What would be the possible reasons K to high reactions rates over the increasing substrate
T
concentration.
for wilting? [2018]
N
(a) The rate of water absorption is less than the rate C. The curve reaches a plateau and does not further
increase with increasing substrate concentrations
A

of transpiration in the afternoon.


(b) The rate of water absorption is more than the rate due to saturation of enzyme with the substrate.
SH

of transpiration in the afternoon. Choose which of the interpretations of the graph are
(c) The changes in the rate of water absorption and correct.
A

transpiration are not associated with wilting. (a) A and B (b) A and C
PR

(d) The rate of water absorption is not related to the (c) B only (d) B and C
rate of transpiration.
32. Glucose is the prime source of energy in our body.
29. Raw banana has bitter taste, while ripe banana has
R.

However, it is stored in the form of glycogen in


sweet taste. It happens because of the conversion of:
the muscle and liver of animals and in the form
(a) Starch to sugar
D

of starch in plants. As a result, every time a cell


(b) Sucrose to fructose
requires glucose, it must hydrolyze glycogen which
(c) Amino acids to sugar
is an energy consuming process. Why does the cell
(d) Amino acids to protein
store glycogen instead of glucose in free form?
30. The gaseous by product of a process in plants is (a) Glycogen is more compact and more hydrophilic.
essential for another vital process that releases
(b) Storage of glucose in free form will consume
energy. Given below are four combinations
of processes and products. Choose the correct more ATP.
combination. (c) Glucose in the free form creates more osmotic
(a) Photosynthesis and oxygen pressure.
(b) Respiration and carbon dioxide (d) Glucose is highly reactive molecule and hence,
(c) Transpiration and water vapour storing in the free form can result in unwanted
(d) Germination and carbon dioxide reactions in the cells.

60 Class-X BIOLOGY PW
33. The figure given below is designed to show 34. An experiment conducted in the laboratory is
respiration in yeast. In one of the tubes, there is tabulated below:
yeast suspension in glucose solution. This solution Test tube - A Test tube-B Test tube - C
was boiled before yeast was added to it. Which Saliva Starch Starch
one of the following is the possible reason for the + + +
boiling of sugar solution? lodine Saliva Saliva
↓ ↓ +
incubation incubation Enzyme
Inhibitor

Incubation
+
lodine
Oil What would be the colour observed in test tube A,

R
Glucose B and C at the end of the experiment?
Bicarbonate
+ (a) A-Yellow, B-No color, C-Blue

A
indicator
Yeast (b) A-No color, B-Blue black, C-Yellow

M
suspension
(c) A-Blue black, B-Yellow, C-No color
(a) To ensure aerobic fermentation. (d) A-No color, B-Yellow, C-Blue black

U
(b) To provide the initial warmth for the yeast to 35. What would happen to earth if carbondioxide was
become active.
Kabsent from its atmosphere?
(a) The earth would be a pleasant place.
T
(c) To remove the dissolved oxygen and carbon
(b) Absence of carbon dioxide would not make any
N
dioxide from the solution. difference to earth.
A

(d) To remove dissolved carbon dioxide and trap the (c) Earth would be devoid of life
oxygen from the atmosphere. (d) Earth would have only animal life.
SH
A
PR
R.
D

Life Processes 61
Statement Type Questions
School Level 1. (d) 2. (a) 3. (a) 4. (c) 5. (a)
6. (c) 7. (b) 8. (b)
Quick Recall
Subjective Questions
Fill in the Blanks
1. Stomata 2. Heterotrophy Very Short Answer Type Questions
3. Chloroplast 4. Iodine 1. Desert plants absorb carbon dioxide during night
5. Peristalsis 6. Pepsin when stomata opens. It is stored as an acidic
intermediate. This intermediate is broken down
7. Bile 8. Alveoli

R
during the day to release carbon dioxide. Reduction
9. Epiglottis 10. Inhalation

A
of carbon dioxide is done during the day when
11. Alcoholic fermentation chemical energy is created by using light energy.

M
12. Red 13. Diastolic 2. Photolysis of water produces a surplus of H⊕ ions.

U
14. Active 15. Pulmonary veins oxygenated This will lower its pH relative to the cytoplasm;
hence, the ratio will be <1.
16. Left ventricle 17. Urea
18. Nephron 19. Reabsorption K
3. Small intestine
T
4. Sphincters are muscles that control the passage of
20. Urinary bladder
food between parts of the alimentary canal. They
N

open and close in a regulated fashion which ensures


True and False Statements
A

that food is exposed to actions of a part of the gut


1. False 2. True 3. False for the required time.
SH

4. False 5. False 6. False 5. ADP contains 2 phosphate groups which make ADP
7. False 8. True 9. True negatively charged. So, there will be repulsions
A

10. False 11. True 12. False between ADP and phosphate like between similar
PR

poles of magnets. But, energy can keep these poles


13. False 14. False 15. False
together just like energy can keep ADP and a third
16. True 17. False 18. True phosphate together as ATP.
R.

19. True 20. False 6. Alveoli is the site of exchange. Oxygen from inhaled
air diffuses across the membrane to the blood where
D

Match the Following it is carried by hemoglobin.


1. (c) 2. (d) 3. (b) 4. (a) 5. (d) 7. During vigorous exercise oxygen is consumed
6. (b) rapidly which makes our blood deficient. Brain can
detect this and increase breathing and heart rate.
Multiple Choice Questions Increased breathing brings more oxygen supply and
1. (c) 2. (c) 3. (b) 4. (a) 5. (b) the heart supplies it to the body where it is needed.
6. (a) 7. (c) 8. (b) 9. (c) 10. (a) 8. (c) Valves ensure that the blood does not flow
11. (a) 12. (d) 13. (b) 14. (c) 15. (c) backwards.
16. (b) 9. Phloem
10. At night transpiration is negligible but absorption
Assertion & Reason Type Questions of water from roots continues. This creates a root
1. (c) 2. (a) 3. (d) 4. (a) 5. (d) pressure that pushes water through the xylem and
6. (c) 7. (d) 8. (b) 9. (c) allows transport.

62 Class-X BIOLOGY PW
11. Body has proteins and nucleic acids (DNA and RNA) (b) In herbivores, the intestine harbors bacteria which
as two major classes of nitrogenous compounds. contains enzymes capable of digesting cellulose.
These compounds are broken down into ammonia. Humans lack those bacteria; hence, cellulose acts
This ammonia is converted to urea the primary as roughage in humans but as a source of nutrient
nitrogenous waste in our body. in cows.
12. Many of plants chemical compounds are useful for 6. Trachea conducts air down to the lungs. Cartilaginous
humans. They are used as medicines, perfumes etc. rings prevent the collapse of wind pipe even during.
Latex of plant is used to make rubber. Resins are 7. Diffusion is a slow process. In multicellular
used as paints. Hence, plant wastes have economic organisms not all the cells are in direct contact
importance for humans with the environment and hence distance required
13. Much part of the blood enters the filtrate, but most for exchange will be too much for diffusion to be
of it is reabsorbed by using both active and passive sufficient for the exchange to take place.
transport. This is the reason kidneys amount to 8.
around 21% of our energy need.
Blood vessel Function
Short Answer Type Questions Artery Usually carries oxygenated blood

R
1. Steps to prepare a temporary mount are: from heart to body

A
1. Take a leaf, fold it from middle, from the layers at Capillary Allows exchange at the site of

M
the edge pull one thin layer using forceps tissues
2. Place this piece in a watch glass, add water, wait Vein Usually carries deoxygenated

U
for some time and then add safranin. blood to heart so that it can be sent
3. After 2-3 min, take this piece and place it on a
glass slide and add glycerin. K to lungs for oxygenation
9. (a) Translocation is the transport of sugars
T
4. Place the coverslip and remove the extra glycerin via phloem. It is essential as products of
N
using a blotting paper. photosynthesis can travel from leaves to roots
and stem where they can be used for metabolism
A

Now you can view this under a microscope using a


and maintenance
low power lens.
SH

(b) Substances reach from source to sink. Source is


2. The various secretions and their functions of the
usually the site of production and sink is the site
gastric glands are
A

of storage/need. During autumn months, however,


(a) HCL; function (d)-active pepsinogen to form the route reverses. Storage organs serve as source
PR

pepsin, and kill microbes whereas remaining parts the sink.


(b) Mucus; function (f)- to protect damage of the wall
10. (a) When water flows into the cells it swells. The
from acid
R.

cell wall of guard cells are thin on the outside


(c) Pepsin; function (e)-digest proteins but thick on the inside. Hence these guard cells
D

3. If the gastric glands do not secrete mucus, HCl will swell more towards the outside opening up and
cause degradation of the lining of the stomach. aperture between them.
4. Amoeba uses its pseudopodia to capture food from (b) Plants also have chemicals that work as messengers.
the surroundings. The food is trapped in a food We call them hormones. These hormones travel via
vacuole. This food vacuole fuses with the lysosome phloem or sometimes simply through diffusion and
which contains hydrolytic enzymes capable of bring about changes.
digestion. Simple compounds produced diffuse out 11. (a) Right ventricle pumps blood to the lungs whereas
into the cytoplasm. The indigestible matter is stored the left one pumps to the whole body. Atria
in the residual body and is eventually thrown out pump to respective ventricles. Hence, ventricles
via exocytosis. have thicker walls to generate stronger pressure.
5. (a) Starch contained in bread is digested partially by (b) Capillaries need to allow diffusion of substance to
salivary amylase. The simple sugars produced the tissue fluids which bathe the cells. Hence they
are sweet need to be one celled thick.

Life Processes 63
12. Blood from the patient is pumped into a machine 1. We should place the peel at the centre of the slide
which has membranes. The membrane separated the and the slides should be held from the sides.
chamber of blood from the chamber of dialysing 2. Straining should be optimum
fluid. The fluid is low in urea whereas blood has 3. We should use a brush to handle the peel to avoid
a higher concentration. This allows urea to be damaging the cells.
removed. The fresh dialysing solution continuously
4. Glycerin should be used in order to prevent drying
gets in and waste is thrown out. Purified blood is
of the peel
returned to the body using pipes.
5. Coverslip needs to be placed in such a way that air
Long Answer Type Questions bubbles are avoided
1. Photosynthesis happens in leaves. The mesophyll 5.
cells in leaves are rich in chloroplasts where
photosynthesis happens.
(i) Carbon dioxide: It is obtained from atmosphere.
(ii) Plants absorb water from the soil through roots
and transport to leaves.

R
(iii) Sunlight: Sunlight comes from the sun.

A
(iv) Chlorophyll: It is present in chloroplast found in
green plants.

M
2.

U
K
T
N
A
SH
A

6. Epiglottis
Larynx
PR

Trachea
3. (a) (i)   Saliva moistens the food and also contains
salivary amylase which digests starch
R.

Bronchus
partially Cut end Pleural
heart membranes
D

of rib
(ii) HCl activates pepsin and is anti-bacterial Alveoli
Pleural fluid
(iii) Bile juice has bile salts which emulsify fat. Lung
Bronchiole
They also neutralize the acidity due to HCl. Diaphragm

(iv) Villi increases the surface area of absorption Fig. 3: Diagrammatic view of human respiratory system
and has a rich supply of blood vessels that carry 7. (a) Circulation
(sectional view of the left lung is also shown)
the absorbed food away from the gut. (i) Circulation is the orderly movement of blood
(b) (i)   Pepsin digests proteins through the blood vessels.
(ii) Lipase digests fats (ii) It is pumped up by the heart, and the flow of
blood is in a cyclic manner.
4. We use safranin to stain the cells so as to improve
their visibility under microscope. We use glycerin Steps followed during circulation are:
to prevent drying of the cells. We must take the (i) The pulmonary veins carry the oxygenated
following precaution. Avoid folding the leaf too blood from both the lungs to the left atrium.
much. The peel should be snipped to a proper size (ii) From here, the blood enters the left ventricle.

64 Class-X BIOLOGY PW
(iii) The left ventricle passes the pumped blood from (ii) Tubular reabsorption: Useful substances from
the aorta to different organs of the human body. the filtrate are reabsorbed back by capillaries
(b) When any blood vessel develops a leak, platelets surrounding the nephron
come into action. A cascade of reactions set in (iii) Secretion: Urea, extra water and salts are
which convert soluble fibrinogen to fibrin protein secreted in the tubule which open up into the
mesh. This mesh traps the cells and closes the collecting duct and then into the ureter .
leakage.
8. (a) Blood when centrifuged shows two major Case-Based Type Questions
components- Plasma (the fluid part) and Formed
elements. Formed elements consist of cells and
cell fragments in the blood. Case Study-I
(b) Plants are not mobile. They are largely made 1. The effect of carbon dioxide on photosynthesis is
up of dead cells. This reduces their requirement being studied in the given experiment.
of materials. Leaves which majorly provide for 2. KOH absorbs carbon dioxide. Thus it helps to
energy needs are also distributed on plant’s body. understand if photosynthesis can take place without
Leaves also are inlet of gasses. This reduces

R
carbon dioxide.
distance of transport for two most important

A
3. Photosynthesis is a process by which autotrophs
susbtances. Plants also cross utilise carbon dioxide
convert light energy into chemical energy, which is

M
and Oxygen between the process of photosynthesis
and respiration. This increases the level of self- later used to fuel cellular activities.

U
sufficiency of its parts. These reasons eliminate 4. If leaf A is tested for starch, we will find that the
part of leaf inside the bottle will be colourless (as
the need of a faster transport in plants.
(c) Valves control the passage of blood between the Kno starch synthesis took place in absence of carbon
T
atria and ventricles. They also prevent backflow dioxide. Whereas, the part of leaf earlier placed
outside turned blue black
N
during ventricular contraction. This forces the
blood into the pulmonary artery and aorta from 5. 6CO2 + 6H2O → C6H12O6 + 6O2
A

the Right and left ventricles respectively.


SH

9. (a) Excretion is the process that eliminates metabolic


Case Study-II
wastes from the body. 1. Photosynthesis
A

(b) Nephron 2. Photosynthesis is a physiological process by which


plant cells containing chlorophyll produce food in
PR

(c) Inferior Adrenal gland


vena cave Renal artery the form of carbohydrates by using carbon dioxide,
Pelvis Renal Vein water and light energy. Oxygen is released as a by-
R.

Kidney product.
Medulla
3. In bell jar B, the mouse suffocated and died because
D

Cortex it was deprived of oxygen.


Dorsal aorta
Ureter 4. Photosynthesis can be said to be the food giver for all.
Green plants synthesise their food by photosynthesis,
and all other organisms are dependent on plants for
Urinary bladder their food directly or indirectly.
Urethra light energy
5. 6CO 2 + 12H 2 O  → C6 H12 O6 + 6H 2 O + 6CO 2 ↑
chlorophyll

10. (a) 2 Kidneys, 2 ureters,


Fig. 4: 1 urinary
Human urinary bladder and
system
1 urethra form the excretory system in humans
Case Study-III
(b) Urine formation involves three steps : 1. Seeds in Flask B are alive and perform respiration.
(i) Glomerular filtration: Nitrogenous wastes, Respiration produces energy as ATP and heat as
glucose, water, amino acids filter from the well.
blood into Bowman.s capsule of the nephron 2. There will be no change in temperature.

Life Processes 65
Case Study-IV 7. (a) 
The process of energy production from the
carbohydrates(glucose) starts by the process
1. The given system is human excretory system
of glycolysis which results in the formation
2. part 2 is ureter, part 4 is urethra and part of two molecules of Pyruvic acid as it's a
3. Nephron is the structural and functional unit of the terminal product. The formed Pyruvic acid
kidney. gets converted to AcetylCoA by the process
4. urine of oxidative decarboxylation by complex set
5. urea of reaction mechanism carried out by pyruvate
dehydrogenase inside mitochondrial matrix.
Case Study-V The formed AcetylCoA enters in Krebs cycle to
1. (a) 
Malpighian corpuscle (A) is the site of form CO2 and H2O along with large amount of
ultrafitration. A considerable amount of water energy.
is reabsorbed in the collecting duct (E) under So, the correct answer is 'Carbohydrates →
the influence of ADH. Proximal convoluted Glycolysis → Pyruvic acid →AcetylCoA →
tubule (C) is the main site for the reabsorption Krebs cycle → CO2 + H2O + energy'.
of glucose and amino acids. In distal convoluted

R
8. (c) Xerophytes are plants which can survive in dry
tubule (D), both hydrogen ions and ammonium or desert regions, i.e. with little water. Sunken

A
ions are secreted, thus it maintains blood pH. stomata ensure less water loss, thus reduces

M
the rate of transpiration. Hence, these are
characteristically found in xerophytes.

U
Competitive Level
9. (b) During ventricular systole, The atrio-ventricular
K valves are closed.
T
Multiple Choice Questions 10. (b) 
Thrombocyte initiates blood clotting and
lymphocytes produce antibodies.
N

1. (b) 2. (a) 3. (a) 4. (a) 5. (c) 11. (d) The main function of the spleen is to filter the
A

6. (a) 7. (c) 8. (c) 9. (d) 10. (d) blood by trapping blood-borne microorganisms.
SH

11. (c) 12. (d) 13. (a) 14. (b) 15. (b) Spleen also aids the body to fight against
16. (b) 17. (b) 18. (c) 19. (b) 20. (c) infections. So, when the spleen is removed from
21. (a) 22. (b) 23. (a) 24. (d) 25. (c)
the adult human body filtration of dead RBCs
A

26. (b) 27. (c) 28. (a) 29. (a)


(red blood cells) would not be possible.
PR

12. (b) 
Bicuspid valve is present between left atrium
Competitive Corner
and left ventricle of human heart.
R.

1. (c) The dark phase does not require the activation 13. (b) P-iii, Q-i, R-iv, S-ii
of sunlight and it occurs in the stroma of the 14. (b) Assertion is true but reason is false because in
D

chloroplast and results in formation of glucose. veins, the blood flows under low pressure.
2. (c) Mode of nutrition in Cuscuta is parasitic. 15. (c) 
Nephridia are the tubular excretory structures
3. (a) 
Dark reaction or biosynthetic phase of seen in organisms like earthworms.
photosynthesis uses products of light reaction 16. (d) 
ADH takes part in absorption of water from
– ATP and NADPH to synthesise glucose. This urine.
reaction was first worked out by Melvin Calvin.
17. (a) 
During heavy exercise, there is a shortage of
4. (b) cracking of lips in human beings is caused by oxygen. In such a case, the muscles undergo
deficiency of vitamin B12 anaerobic mode of respiration. In this type of
5. (d) E.coli bacteria in the human intestine help in anaerobic respiration, due to less oxygen being
synthesizing vitamin B12. available, glucose is converted to lactic acid and
6. (d) 
Trypsin digests proteins and lipase digests energy. This lactic acid gets accumulated in the
emulsified fats. muscles and leads to muscle cramps.

66 Class-X BIOLOGY PW
18. (b) Veins carry deoxygenated blood from the 27. (d) 
The pancreatic juice contains enzymes that
different parts of the body which enter the complete the digestion of starch called pancreatic
right atrium. This blood is passes to the right amylase. It also contains pancreatic enzymes
ventricle which enters the pulmonary artery to that complete the digestion of protein called
supply the blood to the lungs. The oxygenated trypsin, chymotrypsin and carboxypeptidase.
blood is then brought back to the heart through 28. (a) Wilting is the drying of the plants due to loss of
pulmonary vein. Them, the oxygenated blood water. In the afternoon, the rate of transpiration
enters the left atrium and passes into the left is high causing plants to show wilting. During
ventricle which pumps the blood into the aorta. this time, the rate of absorption of water is
The aorta supplies the blood to the tissues of
lowered as the plants needs to spend energy
the body through arteries.
for absorption. So, the rate of water absorption
19. (b) Air from the mouth contains CO2 which is is less than the rate of transpiration in the
utilized in photosynthesis. Hence increase in O2 afternoon.
production.Air from mouth contains O2 which
29. (a) Raw banana has a bitter taste due to the presence
is being added to the plant. Hence increase in
of starch. When the banana ripens, the starch is
O2 production.
converted to sugar in the form of glucose. The

R
20. (b) Adenosine triphosphate (ATP) known as the sugar is responsible for providing a sweet taste.
energy currency of the cell. It is called so

A
So, the conversion is starch to sugar.
because it is responsible for providing the
30. (a) 

M
energy for all activities inside the cell. Photosynthesis and respiration are the two
vital processes occurring in the plants. In the
21. (c) The movement of food within the alimentary

U
given question the two processes that has been
canal is mostly caused by peristalsis. Peristalsis
discussed are photosynthesis are respiration.
is an involuntary contraction of muscles in
the digestive tract that permits food to travel K During photosynthesis, carbon dioxide is fixed
to form glucose and oxygen is released.
T
through the alimentary canal.
31. (b) 
The given graph states that increasing the
N
22. (a) The phenomenon of normal breathing in a
concentration of substrate creates saturation in
human being comprises an active inspiratory and
A

the enzyme-substrate reaction. As the higher


a passive expiratory phase. During inspiration,
SH

muscles of ribs and diaphragm contracts. Hence, concentration of substrate acts as an enzyme
it is an active process. During expiration, the inhibitor, the curve reaches a plateau and does
muscles of ribs and diaphragm relax. Hence, it not increase further.
A

is a passive process. 32. (c) 


Glycogen is insoluble, thus, storing it as
PR

23. (c) The oxygen which is evolved during the process glycogen will not disturb the osmotic pressure
of photosynthesis is liberated due to the splitting rather than glucose which is soluble in water.
of the water molecule.The chlorophyll a is And if the cell stores it as glucose, it will
R.

the essential pigment. All the other pigments disturb the osmotic pressure (hypertonic) that
trap the solar energy and transfer the energy will cause lysis of the cell.
D

to the chlorophyll a. The pigment allows the 33. (c) Boiling reduces the solubility of gasses in the
conversion of the light energy to chemical solution. This allows yeast to go for anaerobic
energy. Photosynthesis occurs in the regions of respiration.
the red and blue light. 34. (a) 
Saliva will break down starch in Test Tube
24. (c) Mitochondria is the site of cellular respiration. B and hence no colouration will be there.
25. (b) Cow is dependent on fodder which is rich in However, in Testtube C, the inhibitor will not
cellulose.To digest cellulose, special microbes allow the enzyme in saliva-amylase-to work and
are present in its stomach. starch will be detected.
26. (c) Pancreas is a heterocrine gland which secretes 35. (c) 
Earth would be devoid of life if there is no
hormones as well as digestive enzymes. carbon dioxide is present on earth.

Life Processes 67
2
CHAPTER
Control and
Coordination

School Level

Introduction
The ability to respond is fundamental to the characteristic of life. It allows organisms to detect sources of energy and matter
in its surrounding. Organisms move in response to various kinds of stimuli e.g., light, heat, nutrients etc. You salivate

R
while passing by a sweet shop. The understanding of changes in the environment produces action, be it procuring food

A
or a defensive display of feathers. Hence, a system for understanding changes in environment and producing a response
is crucial. Control and coordination in the body are mainly concerned with maintenance of state of stability between the

M
internal conditions of organisms and its external environment i.e., homeostasis.

U
Coordination in Animals
K
Animals are heterotrophs. They need to actively run, catch and kill their prey for food. Several actions in the body of
T
animal must happen simultaneously or in a sequence. As food passes through the alimentary canal, the different parts of the
N
digestive system must come into action and release their digestive juices. This is an example of coordination. Control &
A

coordination is achieved by two systems:


In multicellular organisms, it is achieved by both endocrine as well as nervous system. Both the endocrine and nervous
SH

system controls and coordinates the body’s activities but they do so in different ways.
‰ Nervous system: A system that takes in information through our senses, processes the information and triggers the
A

reactions and provides an organised network of point to point connection for a quick coordination.
PR

‰ Endocrine system: A system of chemical messengers or hormones used to communicate within the body.

Nervous System
R.

Except sponges, all multicellular animals contains a nervous system which can be simple or complex. The nervous system
D

is made up of specialised cells termed as neurons or nerve cells which can detect, receive and transmit different kinds of
stimuli. Neuron is the structural and functional unit of nervous system.
The neutral organisation is very simple in lower invertebrates. e.g., in Hydra, it is composed of a network of neurons. The
neural system is better organised in insects, where a brain is present along with a number of ganglia and neutral tissues. The
vertebrates have a more developed neural system.

Structure of Neuron
It is the functional and structural unit of nervous system. It generates and transmits nerve impulses. It is the longest cell of
the body. It usually consists of three different parts: cell body, dendrites and axon.
1. Cell body (cyton): Cell body is the upper expanded part of the neuron and consists of a nucleus with abundant
cytoplasm. In the cytoplasm, numerous cell organelles are embedded, e.g., mitochondria, endoplasmic reticulum,
golgi apparatus, etc. Nissl’s granules (clumps of rough endoplasmic reticulum) are also present in cytoplasm and their
main function is the synthesis of proteins. They are absent in axons.
Nucleus

Dendrite
Myelin sheath

Node of Ranvier
Axon Nerve
ending
Cell body
Fig. 1: Structure of a neuron
2. Dendrites: Short cytoplasmic projections of cyton that branch repeatedly and project out from the cell body are called
dendrites. These may be one to many in number. They receive the stimuli from environment or other neuron and pass
it to the cyton. They carry the nerve impulses towards the cell body.

R
3. Axon: The axons communicate with other neurons through synapse. It is a long branched fibre which transmit

A
information away from cell body to other neurons.

M
The cytoplasm of axon is called axoplasm. Axon is covered by axolemma (plasma membrane). Nissl’s granules are
absent in the axoplasm.

U
Types of Neurons on the basis of Myelination
K
Neurons may have myelinated or non-myelinated axons. Myelinated (medullated) axons have a myelin sheath. A type of
T
glial cell wraps around axon multiple times. The myelinated fibres are enveloped with schwann cells, which form a myelin
N
sheath around axons. Nerve fibres that lack myelin sheath are called non-myelinated fibres. The gaps between two adjacent
A

myelin sheaths are called “Nodes of Ranvier”. Myelinated nerve fibres are found in spinal and cranial nerves. Unmyelinated
nerve fibres are found in autonomous and somatic nervous system. A myelinated neuron conducts impulse faster as it allows
SH

impulse to jump across the gaps. This is called as the saltatory conduction.
Each terminal branch of axon ends in a bulb-like structure called as synaptic knob. Synaptic knob of one nerve fibre forms
A

a junction called synapse with the dendrites of another neuron. Synaptic knob possess synaptic vesicles containing chemicals
PR

known as neurotransmitters.

Types of Neurons on the Basis of Function


R.

Based on their functions, the neurons can be classified into three different categories as follows:
D

1. Sensory/Afferent Neurons: They receive sensory information from organs like skin or tongue and transmit it to the
brain or spinal cord i.e. sensory organ → CNS.
2. Motor/Efferent Neurons: They carry commands from the brain and spinal cord to the muscles, organs and glands
throughout the body.
3. Interneurons/Relay Neurons: As the name suggests, they connect motor neurons and sensory neurons. Much of the
processing happens here.

Knowledge Hub
Unlike body cells which reproduce by cell division, the neurons once mature are incapable of cell division because they
lack centrioles.

Control and Coordination 69


Bipolar Pseudo-unipolar Multipolar
Fig. 2: Types of neuron on the basis of number of dendrons and axons

Transmission of Nerve Impulse


A nerve impulse is transmitted from one neuron to another through junctions called as synapse. The nerve impulse is carried
between a pair of neurons through chemical substances called as neurotransmitters.

R
1. When a stimulus acts on the receptor neuron, it triggers a chemical reaction in the cell body.

A
2. This redistriubutes the ions across the plasma membrane at the beginning of axon and creates a voltage difference.

M
3. If the difference is big enough, similar voltage changes are triggered across adjacent lengths of the axon and the nerve

U
impulse travels along it.
4. When the impulse arrives at the synapse, the synaptic vesicles containing neurotransmitters in the synaptic knob fuses
K
with the plasma membrane, releasing the neurotransmitters into the synapse.
T
5. The dendrites of the next neuron have special receptor proteins that recognise this neurotransmitter and gets excited
N
and the same chain of events starts when the impulse reaches the dendrites.
Some examples of neurotransmitters are Acetylcholine (Ach) (excitatory) which stimulates impulses at the synapse and
A

GABA (Gamma-aminobutyric acid) is a inhibitory neurotransmitter which inhibits impulses at the synapse.
SH

Synapse
A

A nerve impulse is transmitted from one neuron to another through junctions called synapses. A synapse is formed by the
membranes of a pre-synaptic neuron and a post-synaptic neuron, which may or may not be separated by a gap called synaptic
PR

cleft. There are two types of synapses, namely, electrical synapses and chemical synapses.
Synaptic
Presynaptic neuron Postsynaptic neuron
R.

cleft
D

Nerve impulse Nerve impulse

Axon terminal Neurotransmitter Neurotransmitter receptor

Synapse
Fig. 3: Working of a synapse

70 Class-X BIOLOGY PW
A highly developed synapse that forms between a motor neuron and a muscle fiber is known as the neuromuscular junction.
The electrical impulses produced by motor neuron are transformed into electrical activity in the muscle fibers by these
junction.
Axon

Neuromuscular
Junction

Mitochondrion
Muscle
fibre

R
A
Capillary

M
Fig. 4: Structure of a neuromuscular junction

U
Table-1: Chemical vs electrical synapse

S. No. Chemical Synapse K Electrical Synapse


T
1. The synaptic gap is more which causes The synaptic gap is less evident and no
N
synaptic delay. synaptic delay occurs.
A

2. Synaptic vesicle containing neurotransmitters Synaptic vesicles are absent.


are present.
SH

3. Neurotransmitters carry the message. Ions carry the message.


A

4. Slower conduction of nerve impulse. Faster conduction of nerve impulse.


PR

Reflex Actions and Reflex Arc


R.

A reflex action is a sudden, involuntary response to stimulus. Reflexes which are controlled by spinal cord are called spinal
D

reflex. Reflex arc is defined as the route taken by the nerve impulse and responses in a reflex action. The information of the
action does reach the brain but till the time it does, the action has already been executed.

Components of Reflex Arc


The basic components of reflex arc includes:
(i) Receptor: The ends of a dendrite initiates a nerve impulse in a sensory neuron in response to an internal or external
stimulus.
(ii) Sensory (afferent) neuron: The nerve impulse travels from the receptor to the axon terminal branches of the sensory
neuron.
(iii) Integration centre: These are association neurons (interneurons) that process incoming sensory information from
sensory neurons and then, produce response by activating the appropriate motor neurons.

Control and Coordination 71


(iv) Motor (efferent) neuron: It transmits the impulses from the integration centre to the effector organ of the body, such
as a muscle or gland.
(v) Effector: It is the organ of the body that responds to motor nerve impulse, (e.g., muscle by contracting or relaxing).
Spinal cord Message to
(CNS) brain

Sensory neuron
Motor
neuron
Receptors = Heat/Pain Relay neuron
Receptors in skin
Effector (Muscle in arm)

R
A
M
Fig. 5: Reflex arc

U
Examples
‰ Contraction of pupil of human eye when exposed to bright light. K
T
‰ Withdrawal of hand or foot every time when a needle or hot object is touched.
N

‰ Withdrawal of the leg if a person walking barefoot happens to step on a nail.


A

‰ Coughing or sneezing when any unwanted particle enters into throat or nose.
SH

‰ Watering of mouth at the sight of tasty food.

Advantages of Reflex Action


A

‰ This protects the body by enabling the body to give quick responses to harmful stimuli.
PR

‰ In many animals, reflex arcs have evolved as efficient ways of functioning in the absence of true thought processes.
R.

Knowledge Hub
D

Reflex actions are of two types:


(i) Unconditioned reflex: Inborn and inherited. e.g., retracting your hand on touching something hot, sneezing,
coughing, yawning.
(ii) Conditioned reflex: Learned reflexes e.g., salivating at the sight of tasty food.

Human Nervous System


Human nervous system consists of two parts:
1. Central nervous system (CNS)
2. Peripheral nervous system (PNS)

72 Class-X BIOLOGY PW
Nervous System

Central Nervous System Peripheral Nervous System

D Components
Brain Spinal Cord
Spinal nerves
Receptors Nerves
R.
Forebrain Midbrain Hindbrain Cranial nerves
PR
A Pons Medulla Cerebellum
Functional classification
Cerebrum Diencephalon
SH oblongata

Epithalamus Thalamus
A
Hypothalamus
N Afferent (To CNS) Efferent (From CNS)

Pineal gland Pituitary gland


T
K
U Somatic Autonomic
M
Voluntary messages
A Involuntary messages
R
Sympathetic Parasympathetic
(Run & hide, to support emergency response) (Rest & digest)
Bring back to normal state

Control and Coordination


Fig. 6: Classification of nervous system

73
Central Nervous System (CNS)
The central nervous system includes the brain and spinal cord.

Protection of the Brain


Just inside the skull, the brain is further protected by meninges. Meninges are formed from three membranes. A fluid
called CSF (cerebrospinal fluid) flows between meninges which give protection to the brain. Brain also has cavities called
ventricles filled with CSF. Ventricles continue in the spinal cord as central canal.

Skull
Meninges
and the CSF

Ventricles
of the brain

R
Central canal of
the spinal cord

A
Cerebrospinal
CSF Spinal

M
fluid
cord

U
K
Fig. 7: CSF, ventricles and the central canal of the Spinal Cord
T
Central nervous system
(meninges)
N
A

Dura mater
SH

Arachnoid mater
Pia mater
A

Cerebral cortex
Fig. 8: Membranes of cranial meninges
PR

There are 4 ventricles- left and right ventricle in cerebrum, third ventricle in the diencephalon, fourth ventricle connects the
third through pons and medulla to the central canal.
R.

Coverings (Meninges)
D

The brain is protected by three membranous coverings called meninges which continue downwards to the spinal cord.
(i) Duramater: The outermost tough fibrous membrane (dura: tough, mater: mother).
(ii) Archnoid: The thin delicate middle layer giving a web like cushion.
(iii) Pia mater: The innermost highly vascular membrane, richly supplied with blood.
Cerebrospinal fluid is found between the Arachnoid and Dura mater membranes of the brain, which performs functions as
follows:
1. Protection of brain.
2. Removal of waste material.
3. Supply of nutrients to the nervous system.
4. Hormones are supplied to the different parts of the brain via the cerebrospinal fluid.
5. Cerebrospinal fluid acts as a shock absorber.
6. CSF fills the central i.e., spaces (ventricles) of the brain and central canal of the spinal chord.

74 Class-X BIOLOGY PW
Brain
The brain is the central information processing organ of our body and acts as the ‘command and control system’. It is located
in the head and well-protected by the skull. The bony box that houses the brain within the skull is called the cranium. Inside
the box, the brain is contained in a fluid-filled (cerebrospinal fluid) space which provides protection against shock and jerks,
works as a cushion and provides buoyancy to the brain.
The brain can be divided into three major parts namely the fore brain, mid brain and hind brain. They have different parts
that have specific functions.
Cerebrum

Cranium (skull)

Fore brain

Midbrain

R
A
M
Hypothalamus

U
Pituitary gland Pons
Hindbrain Medulla
Cerebellum K Spinal cord
T
Fig. 9: Human brain
N

(a) Fore brain: It consists of three main regions: cerebrum diencephalon (containing epithalamus, hypothalamus and
A

thalamus) and olfactory lobe. Forebrain is the main part of the brain which is involved in thinking.
SH

Cerebrum: It forms the major part of the human brain. It is made up of two halves, namely the left and right
(i)
cerebral hemispheres. It contains motor, sensory and association areas. The sensory areas receive the nerve
impulses, the association areas link this information with the previous information and other sensory informations
A

and the motor areas allows the voluntary muscles to act.


PR

Cerebrum is responsible for the intelligence, memory, and consciousness. The two cerebral hemispheres are
connected by a tract of myelinated fibres called corpus callosum. Cerebral cortex has number of prominent folds
for increasing the surface area. The folds are called gyri and grooves are called sulci. Such a system increases
R.

surface area to accommodate more nerve cells. It is believed that the higher number of convolutions in the brain
D

is due to larger number of nerve cells and hence, greater intelligence.


The inner portion of the cerebrum consists of white matter which mainly contains the axons of the neurons.
Each cerebral hemisphere have specialised areas for vision, hearing, sensation, touch, smell, and muscular
activities.
Table-2: Lobes of the cerebral cortex

S. No. Lobes of Brain Function


1. Frontal lobe Monitors complex thoughts, decision making, problem-solving and
expressing emotions.
2. Occipital lobe Receives information from the retina, and interpret visual information.
3. Parietal lobe Recognizes and processes sensory perception.
4. Temporal Hearing

Control and Coordination 75


Right and Left Brain
While both the hemispheres are used in performing tasks, each hemisphere possess some specialization towards
particular tasks. While the notion of right brain and left brain is wrong, everyone uses both hemispheres, the
specialization of hemispheres is well established.

Right hemisphere Left hemisphere


‰ Creativity ‰ Logic
‰ Music/Art awareness ‰ Spoken Language
‰ Face recognition ‰ Reasoning
‰ Intution ‰ Analytical
‰ Emotion ‰ Written language

R
Fig. 10: Activities performed by right and left hemispheres
(ii) Diencephalon: It lies on the inferior side of cerebrum. Its roof is called epithalamus, sides are called thalamus

A
and floor is termed as hypothalamus. Diencephalon has narrow cavity called third ventricle.

M
� Epithalamus: It form the roof of diencepthal. Pineal gland is found on epithalamus and controls sexual
maturity.

U
� Thalamus: The cerebrum is wrapped around a structure called thalamus. It coordinates the sensory impulses

K
from the various sense organs such as eyes, ears and skin and then relays it to the parts of the cerebrum and
hence, called as the relay center of the brain.
T
� Hypothalamus: Hypothalamus is situated below the thalamus. It forms the lower or ventral part of
N
diencephalon. It is the homeostat center of the brain and is responsible for maintaining a nearby constant
A

state i.e., homeostasis. It has control centres for hunger, thirst, fatigue, sleep, body temperature, sweating
and emotions.
SH

Knowledge Hub
A
PR

The Limbic System


‰ Limbic System (In Latin, limbic refers to the border): The portion of human brain concerned with emotion,
motivation and memory comprises the limbic system. It is made up of structures like amygdala, hippocampus etc.
R.

‰ It is also called the emotional brain.


D

(b) Mid brain: It connects forebrain to hindbrain. It is located between the thalamus/hypothalamus of the forebrain and
pons of the hindbrain. The midbrain receives and integrates visual, factile and auditory inputs.
(c) Hind brain: It consists of cerebellum, pons varolii and medulla oblongata.
(i) Cerebellum: It is the second largest part of brain. It has a convoluted surface to provide the additional space
for many more neurons. It maintains the posture and balance of the body while walking, swimming, riding,
etc, by providing exact precision and the fine control of the voluntary movements. Drinking alcohol affects the
cerebellum.
(ii) Pons: It lies just above the medulla. Pons consists of fibre tracts that interconnect different regions of the brain
and takes part in regulating respiration by regulating breathing rate through penumotaxic centre.
(iii) Medulla oblongata: Medulla oblongata is connected to the spinal cord. It contains centres for various involuntary
activities such as respiration, sneezing, coughing, and gastric secretions. It also contains centre for cardio-
vascular reflexes.

76 Class-X BIOLOGY PW
Knowledge Hub
The electrical activity of the brain can be measured by a device called Electroencephalogram (EEG). Electrical signals
produced by the brain are detected by this device, which helps in diagnosis of abnormalities in the brain, brain tumours,
and head injuries.

Spinal Cord
It is a tubular or cylindrical structure extending downwards from medulla oblongata. It is protected by the vertebral column.
It is the continuation of the brain stem.

Functions of the spinal cord


‰ Coordinates simple spinal reflexes.
‰ Maintains the transmission of messages from muscles and skin to the brain.
‰ Conducts motor responses from the brain to the muscles of the trunk and limbs.

R
Dorsal

A
Dorsal root Gray Central White Sensory

M
ganglion matter canal matter nerve
Dorsal

U
root

K
T
N
A

Motor
SH

nerve
Ventral
root
A
PR

Ventral
R.

Fig. 11: Cross section of Spinal Cord


D

Peripheral Nervous System


The Peripheral Nervous System (PNS) includes the nerves which carry impulses to and from the CNS.
PNS

Somatic Autonomic
Nervous system Nervous system

Autonomic Nervous System


It transmits impulses from CNS to the involuntary organs and smooth muscles of the body. It controls involuntary activities
of internal organs, for example, blood vessels, glands & smooth muscles of alimentary canal & uterus. It is categorized into:

Control and Coordination 77


(a) Sympathetic nervous system: It prepares the body for emergency responses. This is done by regulating the heart rate,
respiration rate, pupilary response, etc. An increase in heart rate will deliver more blood and oxygen whereas a
reduced digestion will allow the blood to be used by limbs. It can also stimulate the secretion of emergency hormones.
Sympathetic nervous system prepares the body for violent action against abnormal conditions.
(b) Parasympathetic nervous system: It is primarily responsible for the body’s rest and digestion response. It aims to
bring the body to the state of calm.
The parasympathetic nervous is more concerned with re-establishing normal conditions after the violent act is over.
It comprises of all the nerves associated with brain and spinal cord. PNS consists of two types of nerves:
PNS

Cranial nerves Spinal nerves

Arise from brain and Arise from spinal cord

R
are 12 pairs in total and are 31 pairs in
total

A
M
Animals receive a variety of external information through specialised structures called sense organs (receptors).

U
Receptors

Types of Exteroceptors Location K


Table 2: Different types of receptors

Function
T
Mechanoreceptors Touch corpuscles in skin Tangoreceptors–Pressure
N

Tactile receptors–Touch
A

Thermoreceptors Skin Frigido receptors (cold)


SH

Heat receptors (warmth)


Chemoreceptors Tongue, nasal mucosa Gustatoreceptors–Taste
A

Olfactoreceptors–Smell
PR

Statoacoustic receptors Internal ear Cochlea–Hearing


Semicircular canals–Balance and equilibrium
R.

Photoreceptors Retina of the eye Rods and cones interpret images


Rods–black and white image. Sensitive to dim light.
D

Cones–Coloured image.
Sensitive to bright light.

Try it Yourself
1. Explain the role of acetylcholine in synaptic knob.
2. Give examples of each reflex action, involuntary action and voluntary action.
3. What is the difference between a nerve and a nerve fiber? Can a nerve fibre and a nerve carry both sensory and
motor impulses? Explain.
4. Can tongue be called both a receptor and an effector? Explain.

78 Class-X BIOLOGY PW
5. Choose the correct option from the information given below regarding the reflex action.
(a) Never reaches brain.
(b) Reaches brain after the action has been taken.
(c) Is not acted or stored by brain.
(d) Is handled by spinal cord exclusively.
6. Look at the figure demonstrating the reflex knee jerk. Leg extensor Nerve
muscle Spinal
Complete the flowchart with the help of given diagram. fibre
cord
I. Stimulus ______________________________________

II. Receptor ______________________________________

Tap tendon
III. Sensory neuron here

R
Femur Pelvis

A
IV. Co-ordinator ___________________________________ Tibia

M
Lower leg jerks
forward
V. Motor neuron

U

VI. Effector _______________________________________ K
T

N

VII. Response ______________________________________


A
SH

Coordination in Plants
A

Plants do not have a specialised system like the nervous system of animals to control and coordinate their responses. Though
PR

plants are fixed to the earth, and cannot move from one place to another, their protoplasmic components are in constant
motion for transporting water, minerals etc. The parts of plant body move in response to the external stimuli. Chemical
R.

substances called phytohormones or plant growth regulators regulate the plant responses. Environmental stimulus such as
D

light or gravity change the directions of growth of plants. The causative factor of the movement is called the stimulus. Both
plants and animals respond to external stimuli through specific movements. The stimulus is sent from one part of the body
(the receiving zone) to another part of the body (the reaction zone).

Plant Movements
Plants do not move from one place to another but they do need to sense light so that they can make food (through photosynthesis),
sense rain so that they know if it is the right time for the seeds to germinate. This does get interesting as many plants have
superfast ways to trap an insect by folding their leaves. Broadly, plants show two kinds of movements: Tropic movements
and Nastic movements. Movements caused due to growth of plants are called tropic movements or tropism and those
movements which are independent of growth are called nastic movements. In nastic movements, the direction of movement
is not determined by the direction of stimulus, either internal or external.

Control and Coordination 79


Tropism (Tropos means turning)
Tropism is a growth movement induced by external stimuli such as light, chemical
or water, etc. The direction of response of the plant part shows a definite relation
to the direction of stimulus, either towards or away from it. If it is towards the
stimulus, it is called as positive tropism and if it is away from the direction of
stimulus, it is called negative tropism.

Types of Tropism
(i) Phototropism (Photo – light, tropism – turning) Phototropism is defined
as the movement of a part of plant in response to light. It is shown by stem
Fig. 12: Response of the plant to the direction
tips and root tips of plants. of light
There are two types of phototropic movements:
(a) Positive phototropism: Shoots generally grow towards the source of light (i.e., in the direction of stimulus).

R
A
M
U
K
T
N
A

Fig. 13: Positive phototropism and negative geotropism shown by shoots and roots respectively
SH

(b) Negative phototropism: The root grows away from the source of light.
(ii) Geotropism (Gravitotropism) (gravis-gravity, tropos-turning): Geotropism is the upward and downward growth
A

of shoots and roots respectively in response to the pull of earth or gravity.


PR

(a) Positive geotropism: The movement of parts of plants towards the direction of gravity is called positive
geotropism. e.g., roots always grow downwards.
(b) Negative geotropism: The movement of parts of plant against the direction of gravity is called negative geotropism.
R.

e.g., shoots usually grow upwards i.e., away from the direction of gravity.
D

Negatively
geotropic
Roots
Shoots
Positively
geotropic
Fig. 14: Plant showing geotropism

(iii) Chemotropism: Tropic movement induced by chemical stimulus is called chemotropism. If a plant part shows
movement or growth towards a chemical, it is called positive chemotropism and if the plant part shows movements or
growth away from chemical, it is called negative chemotropism. For example, the growth of pollen tube toward ovules
during fertilisation is positive chemotropism.

80 Class-X BIOLOGY PW
Pollen Extended tip of pollen tube
Growing towards ovule in
response to the chemical
stimulus (positive
Pollen tube chemotropism)

Ovule

Ovary

Fig. 15: Germination of pollen tube due to chemotropism


(iv) Hydrotropism (hydro – water, tropos – turning): Hydrotropism is the movement of part of plant along water
concentration gradient. Roots show positive hydrotropism. The positive hydrotropic response of roots is stronger than
its geotropic response. The hydrotropism process is started by the root cap sensing the water concentration and sending
a signal to the elongating part of the root. As a result, the roots start growing and moving towards water concentration.

R
A
Water

M
U
Pea seedling
Porous pot
K
T
N
Hydrotropic
curvature of root
A

Sand
SH

Fig. 16: Experimental setup for demonstration of hydrotropism


A

(v) Thigmotropism (thigmo - touch, tropos - turning): Tropic movements induced by contact or touch is called
thigmotropism. It is also known as stereotropism.
PR

Some plants have weak stems which require support for their growth. These plants grow straight until they come in
contact with an object. Upon contact, the growth rate on the side of the stem which comes in contact slows than on the
R.

side opposite to the contact. Thigmotropism can be quite rapid i.e., a tendril has been observed to encircle an object
within 10 minutes. Curling or coiling of tendrils is an example of thigmotropism.
D

Fig. 17: Thigmotropism in tendril of creeper

Control and Coordination 81


Activity
Aim: To demonstrate the response of plant towards the direction of light.
Procedure:
(i) Take a healthy plant and make such an arrangement that it receives direct sunlight and observe it for few days.
(ii) Now turn the plant such that the shoots are away from sunlight and the roots are towards sunlight. Leave it
undisturbed in this condition for few days. Again observe carefully to find the difference in the movement.
Observation:
(i) In the first case, it has been observed that the shoots bend towards the source of light and the roots bend away
from source of light.
(ii) When the plant was turned, it was observed that the growth of shoots is towards sunlight and growth of root is
away from sunlight.
Conclusion: It can be concluded that the shoots show positive phototropism while the roots show negative
phototropism.

R
A
Activity

M
Aim: To demonstrate that roots shows hydrotropism.

U
Method: Place some germinating seeds of pea or gram in moist saw dust contained in a sieve. Keep the apparatus
undisturbed for 2-3 days.
Observation: It has been observed that:
K
T
(i) The radicles pass down and come out of the sieve pores under the
N

influence of gravity.
A

(ii) After some growth, radicles move back and enter the saw dust again.
SH

Conclusion: It can be concluded that:


(i) The roots show both hydrotropic response and geotropic responses.
A

(ii) The hydrotropic response of roots is stronger than its geotropic response.
PR

Nastic Movements
R.

Nastic movements are the non-directional and immediate responses of plants to external stimuli like temperature, humidity,
light, radiation etc. Nastic movements can be seismonastic and nyctinastic movements. Nastic movements associated with
D

touch are rapid, reversible responses because they are associated with changes in the turgor pressure of cells unlike tropic
movements which are associated with the growth which is irreversible.
1. Seismonastic Movement: It is the plant response caused by the mechanical stimuli such as touch, raindrops, fast
moving wind, shock, etc.
The most common example is the leaves of plant Mimosa pudica. Pulvini are swollen areas, present at the base of
petioles, consisting of large number of loosely packed parenchymatous cells separated by intercellular spaces. If the
leaf of Mimosa pudica is touched, the turgor of the lower half of the pulvinus is lost and the leaf begins to fold up and
droop. After sometime, the cells become turgid again and the leaf attains an erect position.
2. Nyctinastic Movement: The movements involves the diurnal variations in the positions of flowers and leaves of
many plants in day and night are called nytinastic movements or sleep movements. Nyctinastic movements include
photonastic and thermonastic movements.

82 Class-X BIOLOGY PW
(i) Photonastic Movements: If the diurnal variations in the position of plant parts (e.g., flower and leaves by
plants) are caused by light stimulus is such non-directional movements are called photonastic movements.
Example is dandelion flower. It opens up in the morning in bright light and closes in the evening when light
fades.
(ii) Thermonastic Movements: If the diurnal variations in the position of plant parts (e.g., flower and leaves by
plants) are caused by the change in temperature of the surroundings such as non-directional movements are
called thermonastic movements.

Knowledge Hub
™ Tendril is a thread-like structure which is formed from modification of stems. It is used for support and attachment
by various climbing plants.
™ Pneumatophores in mangrove trees are breathing roots. They are negatively geotropic and positively phototropic.

Try it Yourself

R
A
1. Plant A is kept in light which is coming from right direction and plant B is kept in light which is coming from

M
left direction. What will happen if we interchange the positions of these two plants?
2. Give an example of:

U
(a) Phototropism (b) Geotropism
(c) Thigmotropism
(e) Chemotropism
(d) Hydrotropism
K
T
3. What do you mean by "nastic" movements?
N
A

Difference between tropic movement and nastic movement


SH

Tropic movement Nastic movement


Direction of movement occurs in the direction of the Movement is non-directional
A

stimulus.
PR

Growth takes place. No growth takes place.


These are slow movements. These are fast and rapid movements.
Example: Growth movement of shoot towards gravity Example: The curling and drooping of leaves in the
R.

sensitive plants.
D

Chemical Coordination in Plants


Plants like animals need some type of internal control and coordination if their growth and development are to proceed in
an orderly manner.

Phytohormones
The chemical coordination in plants takes place by the plant hormones or phytohormones. The plant growth regulators
(PGRs) are simple molecules of diverse chemical composition.
They are of 5 types: auxins, gibberellins, cytokinins, ethylene and abscisic acid.
The PGRs can be broadly divided into two groups based on their functions in a living plant body. One group of PGRs
are involved in growth promoting activities, such as cell division, cell enlargement, pattern formation, tropic growth,
flowering, fruiting and seed formation. These are called plant growth promoters e.g., auxins, gibberellins and cytokinins.

Control and Coordination 83


The PGRs of other group play an important role in plant responses to wounds and stresses of biotic and abiotic origin.
They are also involved in various growth inhibiting activities such as dormancy and abscission. The PGR abscisic acid
belongs to this group. The gaseous PGR, ethylene, could fit either of the groups, but it is largely an inhibitor of growth
activities.

Types of phytohormones or growth regulators

Growth promoters Growth inhibitors


‰ Auxins ‰ Abscisic acid
‰ Gibberellins ‰ Ethylene
‰ Cytokinins

‰ Auxin: Promotes cell enlargement and cell differentiation cause apical dominance stimulated by light, allows plant to
bend towards light.
‰ Cytokinin: Promotes rapid cell division, such as in fruits and seeds.

R
‰ Gibberellins: Stem elongation, delay senescence and promotes bolting.

A
‰ Abscisic acid: Promotes dormancy in seeds and buds and drought tolerance by stimulating closure of stomata, causes

M
wilting of leaves

U
‰ Ethylene: Causes fruit ripening, promotes senescence and helps breaks seed and bud dormancy.

Role of Auxin in Phototropism K


T
When growing plants detect light, auxin is synthesised at the shoot tip. This regulates the direction of growth. The action of
N
auxin hormone is explained below:
(i) When sunlight comes from above, then the auxin hormone present in the tip of the stem spreads uniformly down the
A

stem. Due to this, both the sides of the stem grow equally and the space is straight up.
SH

(ii) When the light falls only from one side of the stem (side B in Figure 2), then the auxin hormone accumulates on
another side (shady side) of the stem, away from t h e light. This is because auxin hormone prefers to stay in
A

shade.
PR

(iii) Now, more auxin hormone is present in the left side of stem as compared to right side of the plant. Due to more auxin
hormone accumulated on the left side of stem, it grows faster than its right side where auxin is absent. Since the left
side of stem grows faster and becomes longer than its right side, therefore, the stem bends towards the right side, i.e.,
R.

in the direction of light.


D

(1) (2) (3)


Fig. 18: Diagrammatic representation of the bending of a plant stem (or shoot)
towards light by the action of ‘auxin’ hormone

84 Class-X BIOLOGY PW
Chemical Coordination in Animals
The nervous coordination in animals, has certain limitations. For instance;
1. Nerve impulses can reach only those animals cells which are connected by nervous tissue.
2. Such cells, after generation and transmission of nerve impulse, take some time to reset their mechanisms before a new
impulse is generated are transmitted.
As the nerve fibres do not innervate all cells of the body and the cellular functions need to be continously regulated; a special
kind of coordination and integration has to be provided. This function is carried out by hormones.
‰ Hormones are non-nutrient chemicals which act as intercellular messengers and are produced in trace amounts.
It is a chemical substance which is produced by endocrine glands and then released into the blood stream and then
transported to the other parts of the body for its specific function.
‰ The organs which responds to hormones are known as target organs as it has receptors (molecules which bind to the
hormone and initiate a cellular response) for the hormone. Therefore, hormones act as intercellular messengers for
information transmission.
‰ Hormones are effective in minute (trace) quantities. Excess or deficiency of hormone may lead to serious consequences.

R
‰ Hormones are specific in function. Chemically they can be proteins, amino acids, steroids, peptides, polypeptides, etc.

A
M
Various Endocrine Glands of Human Beings

U
Endocrine glands lack ducts and hence, called ductless glands. They secrete the chemical substances called hormones into

K
the surrounding fluid which then enters the blood stream and travels through the body. Various Endocrine Glands of Human
Beings are as follows.
T
N
1. Hypothalamus
A

Hypothalamus is the basal part of diencephalon (forebrain) and it regulates wide spectrum of body functions. Therefore, it is
known as master of master gland (pituitary gland). It contains several groups of neurosecretory cells called nuclei which
SH

produce hormones. It controls body temperature, hunger and thirst, mood, sex drive, blood pressure and sleep. The hormones
produced by hypothalamus are of two types: the releasing hormones (which stimulate secretion of pituitary hormones) e.g.,
A

gonadotrophin releasing hormones (GnRH) stimulates the pituitary synthesis and release of gonadotrophins. On the other
hand, somatostatin from hypothalamus inhibits the release of growth hormone from pituitary. These hormones originating
PR

in the hypothalamic neurons, pass through axons and are released from their nerve endings.

Hypothalamus
R.

Pineal Pituitary Gland Pineal


Gland Gland
D

Thyroid Gland

Thymus

Adrenal
Glands
Pancreas

Testes Ovaries

Male Female
Fig. 19: Location of major endocrine glands in the body

Control and Coordination 85


2. Pituitary Gland
It is a small ovoid structure located in a bones cavity called sella tursica and is attached to hypothalamus by a stalk called
infundibulum. Pituitary gland is also known as the master gland of the body as it controls secretions of other endocrine
glands. Hormones released by pituitary gland are as follows:
(i) Growth hormone (GH): It promotes and regulates the growth of the body particularly the skeletal and muscular
systems. It’s deficiency during childhood leads to dwarfism (stunted growth) and oversecretion leads to gigantism
(abnormal growth of body). Acromegaly is a condition when GH rises to abnormal levels in adulthood and leads to
‘gorilla like’ appearance
(ii) Vasopressin or Antidiuretic hormone (ADH): Controls volume of urine by allowing kidney to reabsorb water.
(iii) Oxytocin: Stimulates vigorous uterus contractions of smooth muscles at the time of childbirth and release of milk
from mammary glands.

3. Pineal Gland
Situated on the dorsal side of the forebrain, the pineal gland serves as a crucial regulator in our body's 24-hour diurnal
rhythm. This gland secretes melatonin, a hormone with pivotal roles in maintaining the natural cycles of our sleep-wake

R
pattern and body temperature. Beyond these fundamental functions, melatonin exerts influence over various aspects of our

A
physiology, including metabolism, pigmentation, the menstrual cycle, and our overall defense capability.

M
4. Thyroid Gland
The thyroid gland consists of two lobes which are located on either side of the trachea. Both the lobes are interconnected

U
with a connective tissue called isthmus.

K
Thyroxine: It is the hormone released by thyroid gland. It increases the basal
metabolic rate of the organs and tissues of the whole body. Iodine is a part of
T
Vocal cord
thyroxine and hence, essential for the normal rate of thyroxine hormone synthesis.
N

In case, iodine is deficient in our diet, there is a possibility that we might suffer
A

from goitre (enlargement of thyroid gland). The basal metabolic rate (B.M.R.) is
increased in hyperthyroidism and reduced in hypothyroidism.
SH

Hypothyroidism: It is a condition that results from lack or deficiency of thyroid Thyroid


hormone secretion. Deficiency of iodine in our diet can result in hypothyroidism. It Trachea
A

may be due to a faulty thyroid gland or faulty secretion of TSH (Thyroid Stimulating
PR

Hormone) from pituitary. It is manifested differently in children as compared to


adults. In children, cretinism is caused due to congenital defect of either absence Fig. 20: Thyroid gland
or defect of the thyroid gland leading to stunted growth. The child has a protruding tongue and an enlarged abdomen; the
R.

mentality of the child is low and retarded. In adult women, hypothyroidism may cause irregular menstrual cycle. Myxoedema
is the condition caused by thyroid deficiency in adults. It is characterized by puffy face, thick skin, dry cough, cold and
D

loss of hair. There is a deposition of mucin and fluid retention in extracelluar spaces. BMR is lowered. Usually, thyroxine
exerts a negative feedback on TSH. Higher level of thyroxine reduce TSH release from pituitary gland. When thyroxine
amount reduces due to less iodine, this negative feedback is not sent. TSH continues to be released in higher quantities
which stimulates excess growth of thyroid gland. In this way iodine deficiency causes enlargement of the thyroid glands,
commonly called simple goitre.

Knowledge Hub
Thyroid hormones play an important role in the regulation of the basal metabolic rate. These hormones also support
the process of red blood cell formation. Thyroid hormones control the metabolism of carbohydrates, proteins and fats.
Maintenance of water and electrolyte balance is also influenced by thyroid hormones. Thyroid gland also secretes a
protein hormone called thyrocalcitonin (TCT) which regulates the blood calcium levels.

86 Class-X BIOLOGY PW
5. Parathyroid gland
In humans, four parathyroid glands are present on the back side of the thyroid gland, one
pair each in the two lobes of the thyroid gland. The parathyroid glands secrete a peptide
hormone called parathyroid hormone (PTH). The secretion of PTH is regulated by the
circulating levels of calcium ions.
PTH functions to elevate the levels of Ca2+ in the bloodstream. It achieves this by inducing
bone resorption, a process involving the dissolution and demineralization of bone.
Additionally, PTH promotes the reabsorption of Ca2+ by renal tubules and enhances the
absorption of Ca2+ from ingested food. Notably, PTH acts as a hypercalcemic hormone, Parathyroid
effectively raising blood Ca2+ levels. Collaborating with Thyrocalcitonin (TCT), PTH glands
plays a vital role in maintaining the body's calcium balance.
Fig. 21: Parathyroid gland
6. Thymus gland
Situated between the lungs and positioned behind the sternum on the ventral side of the aorta, the thymus gland is a lobular
structure crucial for the development of the immune system. This gland is responsible for secreting peptide hormones
known as thymosins, which play a very important role in the differentiation of T-lymphocytes, contributing to cell-mediated

R
immunity. Additionally, thymosins facilitate the generation of antibodies, thereby promoting humoral immunity.

A
Thymus undergoes degeneration in aging individuals, leading to a decline in thymosin production. Consequently, the immune
responses in older individuals weaken, highlighting the significance of the thymus in maintaining robust immune function.

M
7. Adrenal Gland

U
Adrenal gland Adrenal cortex
Our body has one pair of adrenal glands, one at the anterior part of each kidney.

and adrenal cortex (outside). K


Adrenal glands is composed of two types of tissues: Adrenal medulla (inside) Fat
T
Adrenal medulla secretes two hormones :adrenaline(epinephrine) and nor-
N
adrenaline(nor-epinephrine). These hormones are secreted in response to any
kind of stress and emergency situations. Thus, they are called as “fight or flight
A

hormones or emergency hormones”. Epinephrine and nor-epinephrine increase


SH

the strength of heart contraction. They also increase blood flow to the heart, Adrenal medulla
liver, skeletal muscles, and adipose tissue; dilate airways to the lungs (increase Kidney
(a) (b)
breathing rate) and increase levels of glucose and fatty acids in the blood.
A

Fig. 22: Adrenal gland


PR

8. Pancreas
Pancreas is a composite gland. It acts as exocrine as well as endocrine gland. It acts as exocrine gland because it secretes
pancreatic juice which is poured into the duodenum with the help of pancreatic duct. It acts as an endocrine gland because
R.

it contains a tissue in the form of clumps of secretory cells known as the Islets of Langerhans. The Islet cells secrete two
hormones: Insulin and Glucagon that regulates blood sugar level. Glucagon increases blood sugar levels whereas insulin
D

decrease the blood sugar levels.


Deficiency of insulin results in diabetes mellitus (hyperglycemia), a condition in which blood sugar level is high and there
is loss of glucose through urine and formation of harmful compounds known as ketone bodies.

9. Ovary
Females have a pair of ovaries present in the lower abdominal region. It is the primary female sex organ. It mainly secretes
two steroid hormones: Estrogen and Progesterone.
(i) Estrogen: Ovary is composed of ovarian follicles and stromatal tissues. The estrogen is synthesised and secreted by
the growing ovarian follicles. FSH (Follicle Stimulating Hormone) from the anterior pituitary controls the secretion
of estrogen by acting on the ovarian follicles. This hormone affects the development of female secondary sexual
characters e.g., development of breast,high pitch voice etc. It also regulates the growth of ovarian follicles
(ii) Progesterone: After ovulation (release of egg from the ovary) the ruptured follicle is converted to a structure called
corpus luteum, which secretes mainly progesterone. It supports pregnancy, and regulates menstrual cycle, etc.

Control and Coordination 87


10. Testis
The interstitial or leydig cells present in the intertubular spaces produce a group of hormones called androgens mainly
testosterone. It is mainly concerned with the development and maintenance of male secondary sexual characters like facial
hair axillary hair, low pitch choice etc. and enhancing the process of spermatogenesis (sperm production).

Feedback Control of Blood Glucose level


Insulin and glucagon get regular feedback to control the blood-glucose levels. When the blood-sugar levels are higher, the
release of insulin is triggered. This causes the cell to take up glucose and store it as glycogen in the liver.
When glucose levels fall below the threshold, the release of glucagon is triggered. This causes cells to break glycogen (stored
in the liver) into glucose and release it into blood stream.
Body cells take up glucose.
Insulin
PANCREAS

R
Liver takes up
Beta cells in pancras release glucose and

A
insulin into the blood. stores it as LIVER
glycogen.
1

M
STIMULUS:
Consumption of
Homeostasis

U
sugar increases
blood glucose level (glucose levels Blood glucose

2
STIMULUS: Blood
90mg/100mL)

K level declines.
T
glucose level falls Blood glucose
N
level rises.
Alpha cells in pancreas Liver breaks
A

release glucagon. down glycogen


and releases
SH

PANCREAS glucose LIVER


Glucagon
A

Fig. 23: Feedback mechanism for control of glucose concentration


PR

Knowledge Hub
R.

Local hormone and Pheromones


D

Local hormone: Hormone which are secreted in the local area and not to
the whole body are called local hormones. The local hormones binds to the
neighbouring cells or with the cell from where they have been secreted.
For example, prostaglandins present in the semen bind with the cells of the
cervix (in female) and cause contraction and support movement of sperm
towards the uterus.
Pheromones: These are also called sex –attractants or trail pheromones.
Pheromones are ectohormones which are released outside the body. These
are interspecific chemicals i.e., received by organisms of the same species
affecting their behaviour. For example, ants uses pheromones to provide Fig. 24: Black ants sensing the pheromones with
directions to other ants about location, to send danger signals and to attract lowered antennae to store food
mates.

88 Class-X BIOLOGY PW
Try it Yourself
1. Differentiate between endocrine and nervous system.
2. When would you expect the synapse to temporarily not work?
3. Roots are gravitropic as it grows towards the direction of gravity. Which pair of diagrams show a controlled
experiment to find out if these roots are also phototropic?
Light
Light proof box Light proof box

Light

1 2 3 4

R
(a) 1 and 2      (b) 1 and 3       (c) 2 and 3      (d) 2 and 4

A
4. Which of the following can be an effector in a reflex arc?

M
(a) A gland      (b) A light receptor      (c) The brain      (d) The spinal cord

U
K
T
N

Summary
A
SH

Control and coordination are the functions of the nervous system and hormones in our bodies. Nervous system is the
organ system present in the animals to control and coordinate different activities of the body. Nervous system comprises
A

of the brain, the spinal cord, and a huge network of nerves that are spread throughout the body. The nervous system
PR

is responsible for sending, receiving and processing messages in the form of chemical signals called as impulses. A
neuron is the basic structural and functional unit of the nervous system. Each neuron consists of three parts, namely, the
cell body or cyton, branched projections called the dendrites, and the long process from the cell body, called the axon.
R.

A reflex action, differently known as a reflex, is an involuntary and nearly instantaneous movement in response to a
stimulus. Reflex is an action generated by the body in response to the environment. Reflex arcs are formed in the spinal
D

cord itself, although the information input goes on to reach the brain. In higher animals, most sensory neurons do not
pass directly into the brain, but synapse in the spinal cord.
The nervous system is divided into two systems - the central nervous system and the peripheral nervous system.
Central nervous system includes the brain and the spinal cord. It receives information from the body and sends out
this information to particular organs. The brain has three such major parts or regions namely the fore brain, mid brain
and hind brain. Peripheral nervous system consists of the cranial and spinal nerve arises from the brain and spinal cord
respectively. Human brain is protected by the thick bones of the skull and it also contains a fluid called cerebrospinal
fluid which provides further shock absorption.
All living things respond to environmental stimuli. Plants also respond to stimuli with the help of chemical compounds
secreted by the cells. Plants being living organisms, exhibit some movements. Plants show two different types of
movement-one dependent on growth and the other independent of growth. Positive phototropism is seen in shoots
which respond by bending towards light. Negative geotropism is seen in shoots by growing away from the ground.

Control and Coordination 89


Hydrotropism is a type of growth response in which the direction is determined by the stimuli of water. Chemotropism
is a growth movement of a plant part in response to chemical stimulus, e.g., growth of pollen tubes towards ovules.
Hormones are the chemical compounds released by endocrine system. They are synthesised at places away from where
they act and simply diffuse to the area of action. Different plant hormones help to coordinate growth, development
and responses to the environment. Plant hormones secreted by the plants are auxins, gibberellins, cytokinins, abscisic
acid and ethylene. Auxins are the hormones synthesised at the tip of the stem and roots. These help the plant in growth
by cell elongation. Auxin also induces shoot apical dominance. Gibberellins help in the growth of the stem, seed
germination, bolting, and flowering. Cytokinins present in the areas of rapid cell division, such as fruits and seeds. They
also promote the opening of the stomata. Abscisic acid inhibits the growth in various parts. It is also responsible for
the closure of stomata. Its effects include wilting of leaves. Ethylene is a gaseous plant hormone which is synthesized
in tissues undergoing senescence and ripening fruits.
Endocrine system is the system formed by ductless glands which secrete chemical substances called as hormones.
Endocrine glands release hormones directly into the blood. Hormones are minute, chemical messengers thrown into
blood to act on target organs. Different types of endocrine glands present in our body are the pituitary gland, the pineal

R
gland, the hypothalamus, the thyroid, the parathyroid, the thymus, the adrenal gland, the pancreas, the testes and the
ovary.

A
A feedback mechanism regulates the action of the hormones. The timing and amount of hormones released are regulated

M
by feedback mechanisms. For example, if the sugar levels in blood rise, they are detected by the cells of pancreas which

U
respond by producing more insulin. As the blood sugar level falls, insulin secretion is reduced.

K
T
NCERT Corner
N
A

1. What is the difference between a reflex action 3. Which part of the brain maintains posture and
SH

and walking? equilibrium of the body?


Ans. Ans. Cerebellum maintains the posture and balance of the
A

Reflex action Walking body.


PR

This is a sudden It is a voluntary 4. How do we detect the smell of an agarbatti


involuntary, and action in which (incense stick)?
spontaneous response. the information is
R.

transmitted by nerve Ans. As the incense stick burns, the odour molecules
spreads in air. It is detected by olfactory receptors
D

to muscles of the legs.


It is controlled and It is controlled by present in the nose. This information is sent to
olfactory lobe in the forebrain by sensory nerves.
coordinated by spinal brain.
Forebrain interprets the smell by putting it
cord.
together with information from other receptors and
2. What happens at the synapse between two information that is already stored in the brain.
neurons?
5. What is the role of the brain in reflex action?
Ans. At the synapse, a chemical substance, i.e.,
neurotransmitter is released as electrical impulse Ans. Brain has limited role in reflex action as most of the
reaches at the end of axon of one neuron. After this, reflex action are controlled by the spinal cord which
occurs spontaneously without thinking of how to
the impulse is received to the other neuron through
respond to the stimuli. Auditory and visual reflex
the dendrites. Thus, information is transmitted from
are handled by some parts of brain.
one neuron to other neuron by synapse.

90 Class-X BIOLOGY PW
6. What are plant hormones? 10. Design an experiment to demonstrate
hydrotropism.
Ans. Plant hormones are the chemical substances which
help in controlling plant activities such as growth, Ans. Take two glass troughs A and B and fill each one
of them two-thirds with soil. In trough A and B,
flowering, height, fruiting, development of plants
place a tiny seedling. In trough B, place a clay pot
and their responses to the environment. inside the soil. Water the soil in trough A daily and
Auxins, gibberllins, cytokinins, abscisic acid uniformly. Do not give water to the soil in trough B
but keep some water in the clay pot. Leave both the
and ethylene are the different examples of plant
troughs for a 2-3 days.
hormones.
A
7. How is the movement of leaves of the sensitive
plant different from the movement of a shoot
towards light?

R
Ans

A
Movement of leaves Movement of a shoot

M
of sensitive plant towards light
B
It is a nastic movement. It is a tropic movement.

U
It does not depend It depends on the
on the direction of direction of stimulus
stimulus applied. applied.
K
T
The stimulus is touch. The stimulus is light.
N

It is caused by the It is caused by the After few days, dig up the seedlings carefully from
A

sudden loss of water unequal growth on the both the troughs without damaging their roots. This
SH

from the swellings at two sides of the shoot. has been found that the roots of seedling in trough A
the base of leaves. grows straight while the roots of seedling in trough
A

Growth does not takes Growth takes places. B is found to be bent to the right side, i.e., towards
place. the clay pot containing water.
PR

This occurs because in trough A, the root of the


8. Give an example of a plant hormone that
seedling gets water from both sides but in trough
promotes growth.
R.

B, the roots get water oozing out from the clay pot
Ans. Auxin promotes growth. which is kept on the right side. Therefore, the roots
D

of seedling in trough B grows and bends towards the


9. How do auxins promote the growth of a tendril source of water to the right side. This experiment
around a support? shows that the root of a plant shows hydrotropism,
i.e., grows towards water.
Ans. When the tip of a tendril touches a support, then the
11. How does chemical coordination take place in
auxins present in its tip move to that side of tip which
animals?
is away from the support. Due to accumulation of
Ans. Chemical coordination in animals happens via
auxins, the side of tendril away from the support
chemical messengers called hormones secreted by
grows faster and becomes longer than the side endocrine tissue or organs. These endocrine tissues
which is in contact with the support and allows the or organs lacks ducts and release their secretion in
tendril to bend around the support. their blood directly.

Control and Coordination 91


12. Why is the use of iodised salt advisable? our skeletal muscles. The breathing rate also
Ans. Thyroid gland needs iodine to maintain normal increases due to the contraction of the diaphragm
levels of thyroxine hormone. Thyroxine hormone and rib muscles. All these responses enable us to
controls all the metabolic activities of our body by face the situations of fear and danger.
regviating of BMR. Due to the deficiency of iodine, 14. Why are some patients of diabetes treated by
a disease called Goitre is caused.
giving injections of insulin?
13. How does our body respond when adrenaline is
Ans. Insulin regulates blood sugar levels. If insulin is
secreted into the blood?
not secreted in adequate amounts, the sugar level in
Ans. When adrenaline is secreted into the blood stream, the blood increases. This leads to a disorder called
the heart beat increases, so that more oxygen is diabetes mellitus.
supplied to our muscles. The blood supply to the
digestive system and skin decreases because the To treat harmful effects of increased level of blood
small arteries around the muscles of these organs sugar, the diabetic patients are treated by giving
contract. This turns the direction of blood towards injections of insulin.

R
A
M
NCERT Exercise

U
K
T
1. Which of the following is a plant hormone? 4. What is the function of receptors in our body?
N

(a) Insulin Think of situations where receptors do not work


A

(b) Thyroxine properly. What problems are likely to arise?


SH

(c) Oestrogen Ans. All information from the environment is detected


(d) Cytokinin by the specialised cells called receptors. They are
A

Ans. (d) Cytokinin is a plant hormone. located in our sense organs like ear, nose, skin,
PR

tongue and eyes or even within our organs (e.g.,


2. The gap between two neurons is called:
stomach).
(a) Dendrite
Different types of receptors can detect different
R.

(b) Synapse
stimuli, e.g., gustatory receptors detects taste while
(c) Axon
D

olfactory receptors detects smell.


(d) Impulse
This information is passed on to integration centers
Ans. (b) Synapse is also known as neuronal junction. It
for processing.
is the site of transmission of impulses between two
neurons. If receptors do not work properly, the changes in
the environment could not be perceived and the
3. The brain is responsible for:
necessary action could not be undertaken. This
(a) Thinking
may bring harm to us or prevent us from acquiring
(b) regulating the heart beat a resource. For example, if skin receptors are
(c) Balancing of the body damaged, and if one person accidentally touches an
(d) All of the above extremely hot object, then hands might get burned
Ans. (d) The brain is responsible for thinking, regulating the as the damaged receptor cannot perceive the external
heart beat and balancing of the body. stimuli of heat and pain.

92 Class-X BIOLOGY PW
5. Draw the structure of a neuron and explain its 8. How does chemical coordination occur in plants?
function.
Ans. Chemical coordination in plants takes place with
Ans. Neuron is the structural and functional unit of nervous the help of plant hormones. Five most studied plant
system. Three main parts of neuron:
hormones are: Auxin, cytokinin, gibberellin, abscisic
(i) Dendrites.
acid and ethylene.
(ii) Axons
They are secreted in response to changes in
(iii) Cell body
environment which drive several processes in plants.
Light stimulates the formation of auxin which drives
Cell body growth of plants. Hence, stimulating the plant to grow
Nucleus
in a particular direction. Cytokinin promotes cell
division. Gibberellin is responsible for the growth of
Dendrite
the stem. Ethylene stimulates fruit ripening.
Axon
9. What is the need for a system of control and

R
coordination in an organism?

A
Ans. An organism needs control and coordination system

M
Nerve
ending for the following reasons:

U
(i) To perceive the availability of resource (food,
Function: The function of neuron is to transmit
information in the form of electrical signals from one K sunlight) and organise necessary action to procure
it. e.g., plants need to detect sunlight and grow in
T
neuron to other neuron. that direction to photosynthesise. Animals need
N

6. How does phototropism occur in plants? to see/smell food to procure it for their energy
A

Ans. The growth-driven movement of any part of a plant in needs.


SH

response to light stimulus is called phototropism.


(ii) To protect the body of the organisms from the
Phototropism in plants occurs due to the phyto -
hormone ‘‘Auxin’’. When light falls on one side of a harmful changes in the environment.
A

plant, the auxin hormone is accumulated on the darker 10. How are involuntary actions and reflex actions
PR

side causing it to expand and grow faster. So, the plant different from each other ?
appears to bend towards light.
Ans. Differences between involuntary and reflex actions
R.

The shoot of plant grows towards the source of light.


Thus, stem shows positive phototropism and roots are as follows:
D

grow away from the source of light and light thus, Involuntary actions Reflex actions
and show negative phototropism.
These actions can take These actions are
7. Which signals will get disrupted in case of a spinal place without any sudden responses to
cord injury? conscious thinking but stimuli without any
Ans. Spinal cord serve as a link between brain and the body are controlled by brain. conscious thinking or
besides being the center for integration for several integration by the brain
reflex arcs. In case of a spinal cord injury: Involuntary actions are Reflex actions may
(i) Most of voluntary actions associated with the site controlled by brain. include paths inside
of injury will get impacted as messages to and from spinal cord or brain
brain will be interrupted. Example: Breathing, Example: Sneezing,
(ii) Reflex actions that have spinal cord as its beating of heart, etc. coughing, blinking of
integration center will not take place. eyes, etc.

Control and Coordination 93


11. Compare and contrast nervous and hormonal 12. What is the difference between the manner in
mechanisms for control and coordination in which movement takes place in a sensitive plant
animals. and the movement in our legs?
Ans. Ans.
Nervous mechanism Hormonal mechanism Movement in a Movement in legs of a
The signals travel The signals travel sensitive plant human
quickly, hence, it is a fast slowly, hence, it is a
The leaves of a sensitive Movement in legs is
process but the effects slow process but effects
plant like Mimosa pudica in control of nerves
are short-lived. are prolonged.
are sensitive to touch delivering commands to
Signals travel via Signals travel through and these movements are muscles
the neurons. blood. called nastic movements.
It transmits information It transmits information
in the form of electrical in the form of chemical They are controlled It is controlled by brain
impulses. messengers (hormones). by changes in turgor and spinal cord and
It does not control It controls metabolism. pressure of the cells. can involve conscious

R
metabolism. thinking or can be the

A
result of a reflex action.
Growth is not affected. Growth is affected.

M
U
Quick Recall K
T
N

Fill in the Blanks True and False Statements


A
SH

1. Receptors are structures which are able to detect 1. Mylein sheath is formed by nodes of ranvier.
________________. 2. The information acquired at the end of the dendritic
A

2. ____________ is the main thinking part of the brain. tip of a nerve cell, sets off a chemical reaction that
creates an electrical impulse.
PR

3. _____________ is the homeostat center of the brain.


3. Chemical coordination is seen in both plants and
4. _____________ and _____________ comprises the animals.
R.

brain stem. 4. Spinal cord has gray matter inside and white matter
D

5. Breathing in human is controlled by ______ of hind outside.


brain. 5. Hormones are produced in trace amounts.
6. _______ is called as relay center of the brain. 6. Ethylene promotes rapid cell division in fruits and
seed.
7. Mimosa pudica shows _______________ movement.
7. Abscisic acid promotes cell division.
8. When growing plants detect light, a hormone called
8. Hypothalamus is the main link between nervous
________________ is synthesized at the shoot tip.
system and endocrine system.
9. Plant hormone ____________ promotes cell 9. Adrenal glands are found in anterior part of each
division. kidney.
10. Hypothalamus stimulates the ________________ 10. Testosterone is responsible for female secondary sex
gland to release growth hormone. characters.

94 Class-X BIOLOGY PW
Match the Following (R) Glucagon (iii) Pregnancy
(S) Oxytocin (iv) Pancreas
1. Match column-I and column-II and choose the correct (T) Vasopressin (v) Antidiuretic
option. (U) Progesterone (vi) Mammary
Column-I Column-II gland
(P) Olfactory receptors (i) Taste (a) P-(i), Q-(ii), R-(iv), S-(vi), T-(v), U-(iii)
(Q) Thermoreceptors (ii) Vision (b) P-(ii), Q-(i), R-(iv), S-(vi), T-(v), U-(iii)
(R) Gustatory receptors (iii) Smell (c) P-(ii), Q-(i), R-(iv), S-(vi), T-(iii), U-(v)
(S) Photoreceptors (iv) Temperature (d) P-(ii), Q-(i), R-(v), S-(iv), T-(iii), U-(vi)
(a) P-(iv) Q-(iii) R-(i) S-(ii) 5. Match the given columns and choose the correct
(b) P-(iii) Q-(iv) R-(ii) S-(i) option.
(c) P-(iii) Q-(i) R-(iv) S-(ii) Column-I Column-II
(d) P-(iii) Q-(iv) R-(i) S-(ii) (P) Adrenal gland (i) Growth hormone
2. Match column-I and column-II and choose the correct (Q) Pituitary gland (ii) Adrenaline

R
option. (R) Thyroid gland (iii) Testosterone

A
Column-I Column-II (S) Testes (iv) Thyroxine

M
(P) Cranium (i) Corpus callosum (a) P-(iii) Q-(iv) R-(ii) S-(i)

U
(Q) Cerebral Cortex (ii) White matter (b) P-(iv) Q-(iii) R-(ii) S-(i)
(R) Olfactory lobe (iii) Smell (c) P-(ii) Q-(i) R-(iv) S-(iii)
(S) Outside butterfly (iv) Protective bony K(d) P-(iv) Q-(i) R-(iii) S-(ii)
T
structure of spinal covering
N
cord
Multiple Choice Questions
(a) P-(iv) Q-(i) R-(iii) S-(ii)
A

(b) P-(iii) Q-(ii) R-(iv) S-(i) 1. Which of the following statements is correct about
SH

(c) P-(iv) Q-(i) R-(ii) S-(iii) receptors?


(d) P-(ii) Q-(iv) R-(iii) S-(i) (a) Gustatory receptors detect taste while olfactory
A

3. Match the given columns and choose the correct receptors detect smell.
PR

option. (b) Both gustatory and olfactory receptors detect smell.


(c) Auditory receptors detect smell and olfactory
Column-I Column-II
receptors detect taste.
R.

(P) Cell division (i) Abscisic acid


(d) Olfactory receptors detect taste and gustatory
(Q) Wilting (ii) Gibberellin
receptors smell.
D

(R) Hastens fruit ripening (iii) Ethylene


2. In a synapse, chemical signal is transmitted from:
(S) Stem growth (iv) Cytokinin
(a) dendritic end of one neuron to axonal end of
(a) P-(iii) Q-(iv) R-(ii) S-(i)
another neuron.
(b) P-(iv) Q-(iii) R-(ii) S-(i)
(b) axon to cell body of the same neuron.
(c) P-(iv) Q-(iii) R-(i) S-(ii)
(c) cell body to axonal end of the same neuron.
(d) P-(iv) Q-(i) R-(iii) S-(ii)
(d) axonal end of one neuron to dendritic end of
4. Match column-I with Column-II and choose the another neuron.
correct option.
3. In a neuron, conversion of electrical signal to a
Column-I Column-II chemical signal occurs at/in
(P) Hypothyroidism (i) Dwarfism (a) cell body (b) axonal end
(Q) Growth hormone (ii) Goitre (c) dendritic end (d) axon

Control and Coordination 95


4. Which is the correct sequence of the components of a 10. When a person is suffering from severe cold, he or she
reflex arc? cannot __________.
(a) Receptors → Muscles→ Sensory neuron → Motor (a) differentiate the taste of an apple from that of an
neuron → Spinal cord ice cream.
(b) differentiate the smell of a perfume from that of
(b) Receptors → Motor neuron → Spinal cord → an agarbatti.
Sensory neuron → Muscle (c) differentiate red light from green light.
(c) Receptors → Spinal cord → Sensory neuron → (d) differentiate a hot object from a cold object.
Motor neuron → Muscle 11. Node of ranvier is a structure concerned with:
(d) Receptors → Sensory neuron → Spinal cord → (a) an axon (b) dendrite
Motor neuron → Muscle (c) spinal cord (d) optic nerve
5. Which of the following statements are true? 12. Which of the following can be an effector in a reflex arc?
(i) Sudden action in response to something in the (a) a gland (b) a light receptor
environment is called reflex action. (c) the brain (d) the spinal cord
(ii) Sensory neurons carry signals from spinal cord 13. The left and right cerebral hemispheres of brain are
to muscles. connected by:

R
(a) corpus callosum (b) nerve cells
(iii) Motor neurons carry signals from receptors to
(c) glial cells (d) osteoblast cells

A
spinal cord.
14. Acetylcholine is a:
(iv) The path through which signals are transmitted

M
(a) neurotransmitter (b) nerve impulse
from a receptor to a muscle or a gland is called
(c) optic nerve (d) olfactory nerve

U
reflex arc.
15. Which of the following is not a characteristic of plant
(a) (i) and (ii) (b) (i) and (iii)
(c) (i) and (iv) (d) (i), (ii) and (iii) K
hormone?
(a) They are produced in high amount.
T
6. Which of the following statements are true about the brain? (b) These are small inorganic compounds.
N
(i) The main thinking part of brain is hind brain. (c) Plant hormones can both promote and inhibit the
growth of plants.
A

(ii) Centres of hearing, smell, memory, sight etc are


(d) Both (a) and (b)
located in fore brain.
SH

(iii) Involuntary actions like salivation, vomiting,


blood pressure are controlled by the medulla in
Assertion & Reason Type Questions
A

the hind brain. Direction: In the following Questions, the Assertion and
Reason have been put forward. Read the statements carefully
PR

(iv) Cerebellum does not control posture and balance


of the body. and choose the correct alternative from the following:
(a) (i) and (ii) (b) (i), (ii) and (iii) (a) Both Assertion (A) and Reason (R) are true and
R.

(c) (ii) and (iii) (d) (iii) and (iv) Reason (R) is the correct explanation of Assertion (A).
7. Spinal cord originates from: (b) Both Assertion (A) and Reason (R) are true and
D

(a) cerebrum (b) medulla Reason (R) is not the correct explanation of Assertion (A).
(c) pons (d) cerebellum (c) Assertion (A) is true but Reason (R) is false.
8. Iodine is necessary for the synthesis of which (d) Assertion (A) is false but Reason (R) is true.
hormone?
1. Assertion (A): Plants lack the nervous system, but
(a) Adrenaline (b) Thyroxine they do coordinate.
(c) Auxin (d) Insulin Reason (R): It is so because of phytohormones.
9. Choose the incorrect statement about insulin. 2. Assertion (A): Phototropism is caused by auxin.
(a) It is produced in pancreas. Reason (R): When light is coming from one side of
(b) It regulates growth and development of the body. the plant, auxin diffuses towards the shady side of
(c) It regulates blood sugar level. the shoot causing it to grow more as compared to the
(d) Insufficient secretion of insulin will cause diabetes. other side of plant.

96 Class-X BIOLOGY PW
3. Assertion (A): Ethylene when applied to unriped (c) Statement-I is correct & Statement-II is incorrect.
fruit to make them ripened.
(d) Statement-I is incorrect & Statement-II is correct.
Reason (R): Ethylene delay senescence of leaves and
mobilise the nutrients to other organs. 1. Statement-I: Reflex arc is the route taken by nerve
4. Assertion (A): Two hormones, glucagon and insulin impulse and response in the reflex action.
regulates the blood sugar level in the body.
Statement-II: Reflex arcs are more complex.
Reason (R): Glucagon decreases blood-sugar level
whereas insulin increases blood sugar level. 2. Statement-I: Mylein sheath is found in all neurons.
5. Assertion (A): Failure of secretion of growth hormone Statement-II: Mylein sheath improves speed of
causes dwarfism in the child.
conduction.
Reason (R): Growth hormone stimulates the body
growth and elongation of long bones. 3. Statement-I: Brain is processing center in many
animals.
Statement Type Questions Statement-II: Brain consists of neurons only.

R
Directions: These questions consist of two statements each, 4. Statement-I: Tropic movements are irreversible.

A
written as Statement-I and Statement-II. While answering
Statement-II: Tropic movements are slow in action.

M
these questions, you are required to choose any one of the
following four responses. 5. Statement-I: Plant hormones travel through phloem

U
(a) Both Statement-I and Statement-II are correct. only.
(b) Both Statement-I and Statement-II are incorrect. KStatement-II: Plant hormones are not produced in seed.
T
N
A
SH
A


Formulate the possible reason for the above
Very Short Answer Type Questions observations of Amrit.
PR

1. List two body functions that will be affected if 4. Why is the flow of signals in a synapse from axonal
end of one neuron to dendritic end of another neuron
cerebellum gets damaged.
R.

but not the reverse?


2. Name the parts of the following regions of the human 5. Define reflex action. State its significance.
D

brain responsible for:


6. Name one organ where gustatory receptor and one
(a) Region for sight olfactory receptor present in human beings.
(b) Region which controls salivation 7. At the time of puberty, both boys and girls show
3. Amrit takes three jar A, B and C. lots of changes in appearance. Name the hormones
responsible for these changes.
Jar A has ice cold water, jar B has lukewarm water and
jar C has water at normal temperature. 8. Why the endocrine glands release their secretions
directly into the blood?
Amrit placed his finger in jar A and B and felt the
9. An old man is advised by his doctor to take less sugar
water to cold and hot respectively.
in his diet. Name the disease from which the man is
Now, again he placed finger in jar B, C and A in same suffering. Mention the hormone due to imbalance
sequence. Soon, he was not able to distinguish hot and of which he is suffering from this disease. Which
cold water in the jar. endocrine gland secretes this hormone?

Control and Coordination 97


10. Name the endocrine gland which secretes growth 7. Sympathetic nervous system prepares you for
hormone. What will be its effect on a person if there is: emergency. Justify the statement.
(a) Deficiency of growth hormones 8. A certain chemical in the nervous system makes you
(b) Excess secretion of growth hormones feel “high”, a rewarded feeling. It binds to a certain
receptor. A famous drug “Heroin” has a molecular
11. Name the gland and the hormone secreted by the gland,
shape which resembles dopamine. Think about the
which are associated with the following problems.
effect of the drug.
(i) a girl has grown extremely tall.
9. Complete the following.
(ii) a woman has a swollen neck.
(a) Acetylcholine: Neurotransmitter : : Adrenaline :
12. Why is the use of iodised salt advisable? Name the __________.
disease caused due to deficiency of iodine in our diet (b) Cerebrum : Forebrain : : Cerebellum : ________
and state its one symptom.
(c) Thyroxine : Thyroid gland : : Insulin __________.
13. Name one plant hormone which inhibits growth. (d) Thermoregulation : Hypothalamus : : Body balance
Write its one more function. :__________.
14. What type of movement is shown by Mimosa putica 10. Answer the following

R
plant leaves when touched with a finger?
(a) Name the endocrine gland associated with brain.

A
15. Define chemotropism with an example.
(b) Which gland secretes digestive enzymes as well

M
16. State how concentration of auxin stimulates the cells as hormones?
to grow longer on the side of shoot which is away

U
from light? (c) Name the endocrine gland associated with kidneys.
17. Samir brought 200 unripe mangoes. His mother
suggested he should keep them in a closed room and K(d) Which endocrine gland is present in males but not
in females?
T
cover it with papers. What would be the reason behind
N
this? 11. Write in tabular form the location and function of the
hormones secreted by each of the following glands
A

present in the human body below.


Short Answer Type Questions
SH

(a) Pituitary gland


1. Name the system which facilitates communication (b) Thyroid gland
A

between central nervous system and the other parts of (c) Pancreas
the body. Mention two types of nerves it consists of 12. Name the plant hormones responsible for the following
PR

along with their organs of origin. functions:


2. State the function of receptors in our body. Think of (i) growth of the stem
R.

any three situations where receptors in the body do (ii) promotes cell division
not work properly. Mention the problems which are (iii) wilting of leaves
D

likely to arise. (iv) elongation of cells


3. What constitutes the central and peripheral nervous 13. (a) Plants do not have any nervous system but yet,
systems? How are various parts of central nervous if we touch a sensitive plant, some observable
system protected? changes take place in its leaves.
4. Differentiate myelinated and non-myelinated nerve Explain how could this plant respond to the
fibre. externals stimuli and how it is communicated.
5. Give the functions of hind brain (b) Name the hormone that needs to be administered
to
6. Neurotransmitters are critical to nervous system. A
(i) increase the height of a dwarf plant.
compound synthesized by a medicine making company
bears similarity with the receptor of a neurotransmitter (ii) cause rapid cell division in fruits and seeds.
“X”. This neurotransmitter communicates pain. What 14. Write a short note on plant hormone "Auxin" and
will be the effect of the compound? Explain your Answer. Gibberellin.

98 Class-X BIOLOGY PW
7. “Nervous and hormonal systems together perform
Long Answer Type Questions the function of control and coordination in human
beings.” Justify the statement.
1. What is reflex arc? Draw a labelled diagram to show
reflex arc on touching a very hot object. 8. Define geotropism. Draw a labelled diagram of a
plant showing geotropic movements of its parts.
2. Name the labelled A, B, C, D, E in the following
diagram. 9. What are plant hormones? Name four different types
of plant hormones and state one function of each.
A
Case-Based Type Questions
D
Case Study-I
E
Nervous system controls and coordinates the body organs
to maintain a physiological balance, called homeostasis
B inside the body. It operates through nerve impulses which
are conducted quickly and unidirectionally at the synapses

R
C through the nerve fibres of the neurons. Neurons are the

A
structural and functional units of nervous system. A diagram
of a neuron has been shown below.

M

Observe the diagram and give the answers of the questions

U
3. Label the diagram listed below the diagram.

A
K P
T
N
B
A

C R S
SH

D Q
E
1. Match the Column-I and Column-II and choose the
A

correct option.
4. Explain with the help of an example, the feedback
PR

mechanism for the regulation of hormonal secretion Column-I Column-II


in the human body. P (i) Cyton
5. Give reasons: Q (ii) Synaptic vesicle
R.

(a) Pituitary is often termed as master endocrine gland. R (iii) Afferent nature
D

(b) Pancreas helps in digestion and also regulates blood S (iv) Efferent nature
sugar level. (a) P-(iii), Q-(i), R-(iv), S-(ii)
(c) Adrenal glands are known as glands of emergency (b) P-(i), Q-(ii), R-(iii), S-(iv)
6. Complete the table: (c) P-(iv), Q-(ii), R-(iii), S-(i)
(d) P-(ii), Q-(iv), R-(i), S-(iii)
Name of the Endocrine Function
Hormone Gland 2. In a neuron, electrical signal is converted to a chemical
signal at/in:
(i) (ii) Deposits extra glucose
(a) P (b) S
of blood as glycogen
(c) R (d) T
Growth Hormone (iii) (iv)
3. In a neuron, incoming information is received by:
(v) Thyroid (vi)
(a) S (b) R
Oxytocin (vii) (viii) (c) Q (d) P

Control and Coordination 99


4. The gap between S of one neuron and P of another 1. Identify the endocrine gland. Where is it located?
neuron is:
2. Why is the above gland referred to as the ‘Master
(a) Dendrite (b) Synapse
(c) Axon (d) Cyton gland’?

3. In the given diagram, identify the hormone released


Case Study-II
by posterior lobe which causes diabetes insipidus and
Given below is an experimental setup to demonstrate a
particular tropic movement in germinating seeds. Study the how does this differ from diabetes mellitus?
diagram and answer the following questions: 4. Which lobe of the above gland secretes:
2
(a) Oxytocin (b) ACTH
Moist sawdust
Germinating seed (c) Growth hormone
1
Brick Case Study-IV
1. Label the parts 1 and 2 Thyroid Stimulating Hormone (TSH) stimulates thyroid

R
gland to produce thyroxine.
2. Name the tropic movement shown by part 1.

A
Table: TSH levels during pregnancy
3. Part 1 is affected by two stimuli. Name them.

M
Which one of the two is stronger? Stage of Normal Low High

U
4. What is thigmotropism? Give one example. pregnancy (mU/L) (mU/L) (mU/L)
First trimester 0.2- 2.5 < 0.2 2.5 - 10
5. What is meant by ‘Positive’ and ‘Negative’ tropic
movements in plants? K
Second trimester 0.3 – 3.0 < 0.3 3.01 – 4.5
T
Third trimester 0.8 – 5.2 < 0.8 > 5.3
N
Case Study-III
It is important to monitor TSH levels during pregnancy.
A

The diagram given below represents an endocrine gland in


the human body. High TSH levels and hypothyroidism can especially affect
SH

Study the diagram and answer the following questions. chances of miscarriage. Therefore, proper medication and
consultation with a doctor is required to regulate/control the
A

proper functioning of the thyroid gland.


PR

1. Give the full form of TSH.

Infundibulum 2. State the main function of TSH.


R.

3. Why do TSH levels in pregnant women need to be


D

Anterior lobe monitored?


Posterior lobe
4. A pregnant woman has TSH level of 8.95 mU/L. What
care is needed for her?

100 Class-X BIOLOGY PW


Competitive Level

Type of Neurons
Based on the number of axon and dendrites, the neurons are divided into three types:
(i) Multipolar: Neurons with one axon and two or more dendrites; found in the cerebral cortex.
(ii) Bipolar: Neurons with one axon and one dendrite; found in the retina of the eye.
(iii) Unipolar: Neurons with only one axon; found usually in the embryonic stage.

Plant Hormones and Their Functions


Terms related to plant growth and development

R
‰ Apical dominance: A phenomenon by which apical buds do not allow the growth of lateral buds.

A
‰ Abscission: Separation of plant parts naturally.

M
‰ Senescence: It is the loss of plant cell’s power to divide causing ageing in plants.
‰ Bolting: Elongation of stem which was reduced earlier.

U
Table-3: Plant Hormones and Their Functions

S. No.
1.
Plant Hormones
Auxins (Naturally
K Function
It promotes cell enlargement and xylem differentiation in plants.
T
‰
occurring auxin is indole It causes apical dominance in plants.
N
‰
3-acetic acid): It is ‰ It also promotes stem and fruit growth.
A

synthesized at shoot tips


‰ It induces parthenocarpy (i.e., the formation of seedless fruits without
SH

and root tips. fertilisation) in number of plants such as banana, grapes.


‰ These are used as herbicides.
A

‰ During tissue culture, the concentration ratio of auxin and cytokinin affects the
plant growth as follows:
PR

 High auxin to cytokinin initiate root development in the seed.


 Low auxin to cytokinin initiate shoot development in the seed.
R.

2. Gibberellins (Gibberellic ‰ It increases the length of grapes stalks.


acid): It is synthesized ‰ It also promotes growth in stems and fruits.
D

in the areas of rapid cell ‰ It also promotes bolting (stimulation in the internode growth just before
division, such as in fruits reproduction and flowering) in beet, cabbages, etc.
and seeds. ‰ It also induces parthenocarpy in many plants (e.g., Tomato).
‰ When gibberellin is higher in concentration as compared to ABA, seeds
germinate whereas when gibberellin is low in concentration as compared to
ABA, seed dormancy gets promoted.
3. Cytokinins: It is ‰ It promotes cell division in plants along with auxins
synthesized in the root ‰ It helps to produce new leaves, chloroplasts in leaves, etc.
apices, developing shoot ‰ Overcome apical dominance.
buds, young fruits.
‰ It increases shelf life of vegetables and keep them fresh for several days, retards
senescence and abscission.

Control and Coordination 101


S. No. Plant Hormones Function
4. Ethylene (only gaseous ‰ It causes horizontal growth of seedlings.
phyto­ hormone): It is ‰ It helps in breaking the dormancy in buds and seeds.
synthesized in large ‰ Most widely used PGR in agriculture.
amounts by tissues under-
‰ It helps in fruit ripening.
going senescence and
ripening fruits.
5. Abscisic Acid or ABA ‰ It inhibits plant growth and seed germination.
(stress hormone or dormin) ‰ It also promotes the closing of stomata and thus causes wilting of leaves.
‰ It increases the tolerance of plants to various kinds of stresses.
‰ It can be used in prolonging dormancy of buds, storage organs and seeds.

Activity

R
A
Discovery of Auxin (1928)
Aim: To find the cause of bending of stem.

M
Procedure: F.W.Went carried out three experiments as mentioned below:

U
Case-A: Tip of coleoptiles were removed.

K
Case-B: The agar blocks were placed on decapitated coleoptile tips.
Case-C: The detached coleoptile was placed on the agar block, later this agar block is placed over the decapitated
T
coleoptile.
N

Observation:
A

Case-A: The removed tips did not grow, which means something essential for plant growth was absent.
SH

Case-B: No response was shown by plants.


Case-C: The plant started growing, which means that some substances transferred from the coleoptile tip, which
in turn transferred to the decapitated coleoptile.
A

Conclusion: The tip is the source of a chemical that causes bending in plants.
PR

Discovery of Plant Hormones


R.

1. Auxin: Charles Darwin and Francis Darwin (1880) firstly reported the presence of growth hormone in the coleoptile
tip of canary grass (Phalaris canariensis) when he described the effect of light and gravity, in his book “Power of
D

movements in plants”. He found that coleoptile tip of canary grass bend towards light.
Examples:
 Natural auxin: Indole-3-acetic acid or IAA and indole butyric acid (IBA)
Synthetic auxins: 2, 4-D (2, 4 Dichlorophenoxy acetic acid), and NAA (Napthalene acetic acid)
 
2. Gibberellin: The effect of gibberellins had been observed over a century ago. Japanese farmers noted some plants in rice
fields were taller, thinner than the normal plants and named this disease as ‘Bakane disease’ or Foolish seedling disease.
Kurosawa (1926) discovered that the causal organism was a fungus called Gibberella fujikuroi.
3. Cytokinin: F. Skoog and his co-workers observed that from the internodal segments of tobacco stems, the callus
(a mass of undifferentiated cells) proliferated only if, in addition to auxin, the nutrient medium was supplemented with
extracts of vascular tissues, yeast extract, coconut milk or DNA. Zeatin, (natural cytokinin), was obtained from unripe
maize grains or kernels.

102 Class-X BIOLOGY PW


4. Ethylene: H.H. Cousins (1910) confirmed the release of a volatile substance from ripened oranges that hastened the
ripening of stored unripened bananas. Later, this volatile substance was identified as ethylene by R. Gane (1934).
Ethylene is synthesized in large quantity by ripened fruits and senescent organs.

Human Endocrine System


Table-4: List of glands, their hormones, and their effects

Gland Hormone Effects


Hypothalamus; basal Releasing Hormones Promotes the release of hormones from pituitary.
part of diencephalon GHRH (Growth-hormone releasing hormone):
(forebrain) control release of GH
TRH (Thyrotrophin releasing hormone): Controls
release of TSH from anterior pituitary
GnRH (Gonadotrophin releasing hormone): Controls
release of FSH and LH from anterior pituitary.

R
CRH (Corticotropin releasing hormone): Controls

A
release of ACTH from anterior pituitary

M
Inhibiting Hormone Inhibits release of hormones from pituitary.

U
GHIH (Growth hormone inhibiting hormone)

Pituitary: a small ovoid ACTH (adrenocorticotrophic K


somatostatin inhibits release of GH
Controls release of hormones from adrenal cortex.
T
structure attached to hormone)
N
hypothalamus by a stalk TSH (Thyroid Stimulating Controls release of hormones from thyroid gland.
A

called infundibulum; Hormone)


consists of three lobes-
SH

Follicle Stimulating Hormone (FSH) Controls production of gametes in males and females.
anterior, middle and
posterior Luteinising Hormone (LH) Controls release of ovum from ovary in female and
A

1. Anterior lobe of production of testosterone in males.


PR

Pituitary Prolactin (PRL) Controls milk production in pregnant females.


2. Intermediate lobe of MSH (Melanocyte stimulating Regulates pigmentation of skin.
Pituitary hormone)
R.

3. Posterior lobe of ADH (Anti-diuretic hormone)/ Acts on kidney tubules and stimulates reabsorption
D

Pituitary Vasopressin (made by hypothalamus) of water and electrolytes by distal tubules.


Oxytocin (made by hypothalamus) Stimulates uterine contraction during childbirth and
milk ejection.
Pineal Gland ; Small Melatonin Sleep-wake cycle, body temperature, pigmentation,
gland reddish-grey in menstrual cycle, defense capability.
colour, about the size
of a pea, located on the
dorsal side of forebrain.
Thyroid Gland; Small, 1. Thyroxine Metabolism of carbohydrate, lipids and fats and
butterfly shaped glands regulation of basal metabolic rate (BMR).
2. Calcitonin
located on either side of
(Hypocalcemic hormone) Calcitonin lowers blood calcium level.
trachea.

Control and Coordination 103


Gland Hormone Effects
Parathyroid Gland Parathormone PTH Increases blood calcium levels by increasing release
Small ovoid pea shaped (Hypercalcemic hormone) of calcium from bone.
glands embedded like Stimulates the process of bone reabsorption.
buttons behind the
thyroid gland.
Thymus Thymosin (peptide hormone) Controls antibody production (Hormonal immunity).
Located just behind the Major role in differentiation of T-lymphocytes (cell-
breast bone (sternum) mediated immunity).
between two lungs.
Adrenal Cortex Mineralocorticoids e.g., Controls reabsorption and secretion of electrolytes
Outer part of adrenal Aldosterone like sodium and potassium.
gland Glucocorticoids e.g., cortisol Involved in carbohydrate metabolism.

R
Sex Corticoidse Secretes androgens

A
Adrenal Medulla Adrenaline/Epinephrine Flight or Fight Response (increase alertness, pupilary

M
Inner part of adrenal gland Nor-adrenaline/ norepinephrine dilation, Piloerection, sweating, rate of respiration
strength of heart contraction.

U
Heart ANF (Atrial Natriuretic Factor) Reduces blood pressure
Stomach Gastrin
K
Stimulates secretion of hydrochloric acid and
pepsinogen.
T
Kidney Erythropoietin Increases red blood cell formation (erythropoiesis)
N

Liver Angiotensinogen Component of renin-angiotensin system (RAS) that


A

regulates blood pressure and fluid balance.


SH

Table-5: Summary of disorders related with hormones


A

Hormone Under- Symptoms Overproduction Symptoms


Production
PR

Growth hormone Dwarfism Retarded growth Gigantism Excessive growth


(GH)
R.

Antidiuretic Diabetes insipidus Dehydration, thirst SIADH (syndrome of Increased body weight and
D

hormone (ADH)/ inappropriate ADH water content.


Vasopressin secretion)

Thyroxine Myxoedema, Low metabolic Hyperthyroidism, Characterised by enlargement


cretinism rate; low body (Grave disease/ of the thyroid gland,
temperature exopthalamic, goitre) protrusion of eyeballs.
Increased BMR and body
temperature.
Insulin Diabetes mellitus High levels of Hyperinsulinemia Low blood sugar levels
glucose in blood.

104 Class-X BIOLOGY PW


Estrogen (females) Hypogonadism Sterile female, Adrenogenital syndrome Overproduction of androgens
lack of (by adrenal cortex leading to
secondary sexual masculine body)
characteristics
Precocious puberty Premature sexual maturation
Androgens Hypogonadism Sterile male, Gynaecomastia Abnormal production of
(males) lack of estrogens, enlarged breasts
secondary sexual
characteristics Precocious puberty Premature sexual maturation

Feedback Mechanism
‰ A process by which a product controls its own production is known as a feedback mechanism.
‰ This regulates the production of a variety of hormones.

R
Positive Feedback Mechanism

A
‰ A positive feedback reinforces the stimulus.

M
‰ Oxytocin (a pituitary hormone) stimulates mammary gland to release milk. The suckling action of baby stimulates

U
oxytocin production and secretion into the blood, which then causes milk to be let down into the breast. The positive
feedback cycle is maintained until the baby stops sucking.
K
T
Simple neuroendocrine pathway Example: oxytocin signaling
N

STIMULUS Suckling
A
SH

Neurosecretory
Hypothalamus cell
A
PR

Posterior Hormone Oxytocin ( )


Positive feedback

pituitary

Blood
R.

vessel
D

Circulation
throughout
the body via
blood

Target Smooth muscle in


cells mammary glands

Response Milk release

Fig. 25: Positive feedback mechanism of oxytocin

Control and Coordination 105


Negative Feedback Mechanism
‰ When a response reduces the stimulus, we say it exerts negative feedback
‰ Thyroxine levels are detected and controlled by the hypothalamus and pituitary gland.
‰ Thyroxine is an inhibitor of TSH and TRH release. TRH (a releasing hormone of the hypothalamus) stimulates pituitary
to release TSH which in turn stimulates thyroid gland to secrete thyroxine hormone.
‰ Low levels of thyroxine in blood triggers the production of TRH (Thyrotropin releasing hormone), which in turn triggers
the pituitary to release TSH (Thyroid stimulating hormone), causing the thyroid to produce more thyroxine. As a result,
levels of thyroxine in blood returns to normal.
‰ Thyroxine (the response) here was reducing the stimulus (by inhibiting the release of TSH)
STIMULUS 1. Thyroid hormone levels drop below
the normal range. Sensory neurons
respond by sending nerve impulses
Neurosecretory cell
to neurosecretory cells in the
in the hypothalamus
hypothalamus.

R
TRH 2. Neurosecretory cells secrete
Negative feedback

A
thyrotropin-releasing hormone (TRH
) into the blood, which carries it to

M
the anterior pituitary

U
K
3. TRH causes the anterior pituitary to
secrete thyroid-stimulating hormone
(TSH), also known as thyrotropin( )
T
TSH into the circulatory system
Anterior
N
pituitary
A
SH

Circulation
throughout the
body via blood
A
PR

Thyroid
gland
4. TSH stimulates endocrine cells in
the thyroid gland to secrete thyroid
hormone (T3 and T4 ) into the
R.

circulatory system
D

Thyroid
hormone

Circulation
throughout 5. Thyroid hormone levels increase
the body in the blood and body tissues.
via blood Thyroid hormone acts on target
cells throughout the body to control
bioenergetics; help maintain normal
blood pressure, heart rate, and
Response muscle tone.

6. As levels return to the normal range, thyroxine blocks TRH release from the
hypothalamus and TSH release from the anterior pituitary, forming a negative-
feedback loop that prevents overproduction of thyroid hormone
Fig. 26: Negative feedback mechanism for control of thyroxine hormone

106 Class-X BIOLOGY PW


1. Fight-or-flight reactions cause activation of: 8. Coordination via the nervous system tends to differ
(a) the adrenal medulla, leading to increased secretion from that produced by the endocrine system because
of epinephrine and nor-epinephrine. the nervous system:
(b) the pancreas, leading to reduction in blood sugar (a) Is quick, precise and localized.
levels. (b) Is slower and more pervasive.
(c) the parathyroid glands, leading increased metabolic (c) Does not require conscious activity.
rate.
(d) Secrete hormones.
(d) the kidney, leading to suppression of renin-
angiotensin-aldosterone pathway. 9. Which of the following plant function is controlled by
auxins?
2. Centre of hunger and thirst is located in:

R
(a) Apical dominance
(a) Forebrain (b) Hypothalamus

A
(c) Mid-brain (d) Spinal cord (b) Seed dormancy
(c) Bolting

M
3. Coordination is achieved through nervous system as
well as endocrine system by respective agents like: (d) Stem elongation

U
(a) vitamins and proteins 10. The given diagram shows some of the features of
(b)
(c)
neurotransmitters and hormones
hormones and sugars
Khuman skin.
Hair
T
(d) sugar and hormones
N

4. Which of the following is not a reflex action? Hair erector


A

(a) Blinking of eyes due to bright light Muscle


SH

(b) Salivation on seeing food Nerve


(c) Sweating Temperature endings
receptor
A

(d) Withdrawl of hand on touching some hot object Information


Information
PR

5. Which hormone regulates the water balance in the to brain from brain
body?
Which part of the brain coordinates the information
(a) Insulin (b) Thyroxine
labelled in the diagram?
R.

(c) Testosterone (d) Vasopressin


(a) Medulla (b) Hypothalamus
D

6. The correct sequence of meninges of brain from (c) Cerebrum (d) Cerebellum
outside to inside is:
11. Which of the following receptors is incorrectly paired
(a) Dura mater → arachnoid → pia mater
with their senses?
(b) Arachnoid → dura mater → pia mater
(a) Chemoreceptors - Chemicals
(c) Pia mater → dura mater → arachnoid
(d) Dura mater → pia mater → arachnoid (b) Photoreceptors - Pain
(c) Thermoreceptors - Heat
7. Which part of the human brain controls body
temperature? (d) Nociceptors - Pain
(a) Pituitary 12. Which plant hormone promotes dormancy in seeds
(b) Cerebellum and buds?
(c) Hypothalamus (a) Auxin (b) Ethylene
(d) Pons (c) Cytokinin (d) Abscisic acid

Control and Coordination 107


13. The secretion of which hormone leads to physical 20. Cretinism results due to:
changes in the body when you are 10-12 years of age? (a) Excess secretion of growth hormone.
(a) Estrogen from testes and testosterone from ovaries. (b) Under secretion of thyroid hormone.
(b) Estrogen from adrenal gland and testosterone from (c) Excess secretion of adrenal gland.
pituitary gland. (d) Under secretion of growth hormone.
(c) Testosterone from testes and estrogen from ovaries. 21. A person entering an empty room suddenly finds a
(d) Testosterone from thyroid gland and estrogen from snake right in front on opening the door. Which one
pituitary gland. of the following is likely to happen in this neuro-
14. Which of the following endocrine glands does not hormonal control system?
exist in pairs? (a) Sympathetic nervous system is activated releasing
(a) Testes (b) Adrenal epinephrine and nor-epinephrine from adrenal
medulla.
(c) Pituitary (d) Ovary
(b) Neurotransmitters diffuse (rapidly across) the
15. Dandelion flowers open the petals in bright light sympatic cleft and transmit a nerve impulse.
during the daytime but close the petals in dark at
(c) Hypothalamus activates the para-sympathetic

R
night. This response of dandelion flowers to light is
division of brain.
called:

A
(d) Sympathetic nervous system is activated releasing
(a) Phototropism (b) Thigmonasty epinephrine and epinephrine from adrenal cortex.

M
(c) Chemotropism (d) Photonasty
22. Given below is a table comparing the effects of

U
16. Rhythm of sleep or biological clock in our body is sympathetic and parasympathetic nervous system for
controlled by:
(a) Pituitary gland (b) Pineal gland Kfour features (1-4). Which one feature is correctly
described?
T
(c) ACTH (d) ADH Sympathetic Parasympathetic
Feature
N

17. Identify which of the following statements about nervous system nervous system
A

thyroxin is incorrect? 1. Salivary Stimulates


Inhibits secretion
glands secretion
SH

(a) Thyroid gland requires iodine to synthesize


thyroxine. 2. Pupil of
Dilates Constricts
(b) Thyroxine is secreted by thyroid hormone. the eye
A

(c) It regulates protein, carbohydrates and fat 3. Heart rate Decreases Increases
PR

metabolism in the body. 4. Intestinal


Stimulates Inhibits
(d) Iron is essential for the synthesis of thyroxin. activity
(a) 1 (b) 2
R.

18. A big tree falls in a forest, but its roots are still in
contact with the soil. The branches of this fallen tree (c) 3 (d) 4
D

grow straight up (vertically). This happens in response 23. How do parasympathetic neural signals affects
to: working of heart and digestion?
(a) Water and light (a) Both heart rate and digestion rate increases.
(b) Water and minerals (b) Heart rate decreases but digestion increases.
(c) Gravity and water (c) Reduces both heart rate and digestion.
(d) Light and gravity (d) Heart rate is increased but has no effect on
digestion.
19. A growing seedling is kept in a dark room. A burning
lamp is placed near to it for a few days. The top part 24. Pneumatophores of mangrove are example of:
of seedling bends towards the burning candle. This is (a) Thermonasty
an example of: (b) negative phototropism
(a) Chemotropism (b) Hydrotropism (c) positive geotropism
(c) Phototropism (d) Geotropism (d) negative geotropism

108 Class-X BIOLOGY PW


25. Given below is the incomplete table about certain (c) The chemicals released from the axonal end of one
hormones, their source glands and one major effect neuron cross the synapse and generate a similar
of each on the body of humans. Identify the correct electrical impulse in a dendrite of another neuron.
option for the blanks X, Y and Z. (d) A neuron transmits electrical impulses not only to
Gland Secretion Effect on body another neuron but also to muscle and gland cells.
Maintenance of 27. The growth of tendril in pea plants is due to:
X Oestrogen secondary sexual (a) effect of light
characters (b) effect of gravity
Raises blood sugar (c) rapid cell divisions in tendrillar cells that are away
Pancreas Y from the support
level
(d) rapid cell divisions in tendrillar cells in contact
Anterior Oversecretion leads
Z with the support
pituitary to gigantism
28. The movement of sunflower in accordance with the
Options: path of sun is due to:
X Y Z (a) phototropism (b) geotropism
(a) Ovaries Glucagon Growth hormone (c) chemotropism (d) hydrotropism

R
(b) Placenta Insulin Vasopressin 29. Involuntary actions in the body are controlled by:

A
(c) Ovaries Thyroxine Calcitonin (a) medulla in fore brain

M
(d) Testes Glucagon Calcitonin (b) medulla in mid brain
26. Which of the following statements about transmission (c) medulla in hind brain

U
of nerve impulse is incorrect? (d) medulla in spinal cord
(a) Nerve impulse travels from dendritic end towards
axonal end. K
30. A doctor advised a person to take an injection of
insulin because:
T
(b) At the dendritic end, electrical impulses bring about (a) his blood pressure was low
N
the release of some chemicals which generate an (b) his heart was beating slowly
A

electrical impulse at the axonal end of another (c) he was suffering from goitre
neuron. (d) his sugar level in blood was high
SH
A

Competitive Corner
PR

1. How many nerves pairs arise from spinal cord? II. Plants hormone gibberellins helps in growth of a
 (Gujrat 2013) stem.
R.

(a) 21 (b) 31 III. Cytokininis inhibits cells division.


D

(c) 41 (d) 51 IV. Abscisic acid promote growth in plants.


2. If the apical parts of the negative geotropic roots of (a) I and III are correct
mangrove plant are pasted with wax, which function
(b) II and IV are correct
of these roots will be affected? (Rajasthan 2013)
(c) I and II are correct
(a) Exchange of O2 and CO2
(b) Water absorption (d) I and IV are correct
(c) Fixation of plant 4. If a rat is given an injection of sodium iodide with
(d) Photosynthesis radioactive iodine, then in which of the following
most of iodine would be incorporated?
3. Choose the correct option from the following.
 (Haryana 2015)
 (Chandigarh 2013)
I. In light, hormone auxin helps the cells to grow (a) Cartilage (b) Thyroid
longer in plants. (c) Parathyroid (d) Lymph nodes

Control and Coordination 109


5. Read the following statements and select the correct 10. The hormone that promotes lactation and simulates
option. (Karnataka 2017) the uterus to contract is ________. (Kerala 2017)
I. Auxins help to prevent fruit and leaf drop at early (a) Calcitonin (b) Thyroxine
stages. (c) Oxytocin (d) Vasopressin
II. Abscisic acid promotes seed germination.
11. The middle layer of brain meninges is________.
(a) ‘I’ is false and ‘II’ is true
 (West bengal 2017)
(b) ‘I’ is true and ‘II’ is false
(a) Dura matter
(c) Both ‘I’ and ‘II’ are true
(b) Pia matter
(d) Both ‘I’ and ‘II’ are false
(c) Arachnoid
6. What is the correct direction of flow of electrical
(d) Sub-arachnoid space
impulses in nerve cells? (Punjab 2017)
12. Which of the following is a plant hormone?
(a)  (Rajasthan 2018)
(a) Insulin (b) Thyroxine
(c) Cytokinin (d) Oestrogen

R
(b) 13. Which of the following helps in formation of insulin?

A
 (Delhi 2018)
(a) Islets of Langerhans (b) Pituitary gland

M
(c) (c) Thyroid gland (d) Adrenal gland

U
14. Choose the correct option to complete ‘P’, ‘Q’, ‘R’

(d) Kand ‘S’ in the following table.


Hormone Function
(Delhi 2019)
T
A Stimulates growth in all organs
N

7. Which one of the following hormones is not produce B Stimulates pituitary to release growth
A

from anterior lobe of pituitary gland? hormone


SH

 (West bengal 2017) C Controls blood sugar level


(a) GH (b) ADH D Regulates BMR
(c) ACTH (d) TSH
A

(a) P – Insulin, Q-Thyroxine, R-Growth hormone


8. Our skin becomes dark in colour when exposed to
PR

releasing hormone, S-Growth hormone releasing


excess of sunlight. It is due to presence of: hormone Release Factor
 (Jharkhand 2017)
(b) P – Growth hormone releasing hormone, Q-
(a) Carotene (b) Melanin
R.

Insulin, R- Thyroxine, S-Growth hormone


(c) Flavoxanthin (d) Haematoxylin releasing hormone Release Factor
D

9. Which part of brain shows pituitary gland? (c) P – Thyroxine, Q- Insulin, R-Growth hormone
 (Chandigarh 2017) releasing hormone, S-Growth hormone releasing
R hormone Release Factor
Cranium (d) P – Growth Hormone, Q- Growth hormone
releasing hormone Release Factor, R- Insulin,
S S-Thyroxine.
P Q 15. What would happen to the person if cerebellum of his
Cerebellum brain is damaged?  [NTSE 2013]
Pons
Spinal cord (a) He will lose his memory power.
Medulla
(b) He will not be able to swallow food properly.
(a) Q (b) S (c) He will be unable to coordinate and stand properly.
(c) P (d) P (d) He will lose his power of vision and hearing.

110 Class-X BIOLOGY PW


16. Observe the experimental sets [X] & [Y]. 19. A squirrel was eating a fruit on the ground. Suddenly,
 [NTSE 2018] it was attacked by a dog. The squirrel rushed to the
(X) (Y) tree immediately and saved itself from the dangerous
Test tube attack. What immediate changes are most likely to
have taken place in the body of the squirrel?
Water  (NTSE 2019)
(a) Blood flows to the stomach for rapid digestion.
Cotton (b) Adrenaline was secreted in the blood by the adrenal
Plug glands
Plant (c) Heart beat becomes faster and pumps more blood,
so that muscles get more oxygen
Day 5 (d) Adrenocorticotrophic hormone is secreted in the
Day 1 blood and blood flows more towards the vital
organs.
Observe the test tube X & Y. From the list given Select the correct combination of options given below.
below, choose the combination of responses of shoot (a) A and B (b) A and C
and root that are observed in Y. (c) B and C (d) C and D

R
(a) Positive phototropism and positive geotropism 20. Positions of endocrine glands are labelled A–E in

A
(b) Negative phototropism and positive geotropism the given diagram. Match the symbols of glands in

M
(c) Positive phototropism and negative geotropism column-I with the type of hormone it secretes given
(d) Only negative phototropism in column-II. (NTSE 2019)

U
17. In a hypertensive patient, the systolic pressure
is increased to 150 mm of Hg. Which part of the
brain would be involved in the regulation of blood
K E
T
D
pressure?  (NTSE 2018)
N

(a) Medulla
A

C
(b) Cerebrum
SH

(c) Cerebellum B
A
(d) Hypothalamus
A

18. Match the items given in Column-I with the most


PR

appropriate ones in Column-II and choose the option


with the correct matching pairs: (2019)

Column-I Column-II Column-I Column-II


R.

P. Cranial nerves (i) Testosterone A. (i) Progesterone


D

Q. Leydig cells (ii) Natural reflex B. (ii) Insulin


R. Acetylcholine (iii) 12 pairs C. (iii) Parathyroid hormone
S. Spinal nerves (iv) Prolactin D. (iv) Melatonin
T. Sneezing (v) Neurotransmitter E. (v) Follicle stimulating hormone
(vi) 18 pairs (vi) Thyroxine
(vii) 31 pairs (vii) Aldosterone
(viii) Conditioned reflex Choose the correct combination from the following.
(a) P-(iii), Q-(i), R-(v), S-(vii), T-(ii) (a) A–(i), B–(ii), C–(vii), D–(iii), E–(v)
(b) P-(vii), Q-(v), R-(ii), S-(i), T-(iii) (b) A–(i), B–(iv), C–(ii), D–(iii), E–(vi)
(c) P-(vii), Q-(ii), R-(v), S-(iv), T-(iii) (c) A–(v), B–(ii), C–(iv), D–(iii), E–(vii)
(d) P-(ii), Q-(i), R-(v), S-(iv), T-(vii) (d) A–(v), B–(iv), C–(vii), D–(iii), E–(ii)

Control and Coordination 111


21. Charles Darwin observed that seedlings grow towards Column A Column B
light. He called this response ‘phototropism’. In an
(x) Low TRH, Low (i) Anterior pituitary
experiment, two light sources are used to illuminate
TSH and Low fails to produce
each seedling. Each source is indicated by circle in
T3, T4 TSH
the diagrams below. The larger circle represents a
light source with twice the illumination than the light (y) High TRH, High (ii) Thyroid fails to
source represented by the smaller circle. (2013) TSH and Low produce T3 and
T3, T4 T4
(z) High TRH, low (iii) Hypothalamus
TSH and Low fails to secrete
T3, T4 TRH
(i) (ii)
Which of the following is the correct match for the
above?(2013)
(a) (x)-(i); (y)-(ii); (z)-(iii)
(b) (x)-(iii); (y)-(ii); (z)-(i)

R
(iii) (iv) (c) (x)-(ii); (y)-(i); (z)-(iii)

A
Which of the above responses would be observed? (d) (x)-(iii); (y)-(i); (z)-(ii)

M
(a) only (iv) 23. Rahul sprayed a chemical "X" on a plant with rosette
habit. After few days, he found the internodal distances

U
(b) only (ii)
(c) (i) and (iii) to have increased suddenly. The chemical ‘X’ might
(d) (i) and (iv) Kbe:
(a) Ethylene
(IQQJS 2019)
(b) Abscisic acid
T
22. The following flow chart represents the feedback
(c) Auxin (d) Gibberellic acid
N
loops that regulate secretion of thyroid hormones
(T3 and T4). Such secretion essentially regulates the 24. The autonomous nervous system regulates involuntary
A

basic metabolism rate in mammals. The (‘+’) and functions of the body and can be subdivided into the
SH

(‘–’) signs represents positive and negative regulation, sympathetic and the parasympathetic nervous system.
respectively. Both of these systems control the same group of body
functions, but have opposite effects on the functions
A

Hypothalamus
they regulate. The sympathetic nervous system
PR

prepares the body for intense physical activity like the


TSH Releasing Hormone (TRH)
fight-or-flight response. The parasympathetic nervous
system has the opposite effect and relaxes the body
R.

Anterior Pituitary and inhibits or slows many high energy functions.


Which of the following involuntary effects in the
D

Thyroid stimulating body are brought about by the sympathetic nervous


hormone (TRH)
system during a fight-or-flight situation?
Hypothalamus I. Increased salivation
II. Increased digestion
T3 and T4
III. Loss of bowel and bladder control
Three disease conditions are being studied where (x) IV. Body shivering
the anterior pituitary fails to produce TSH, (y) the V. Crying
thyroid gland fails to produce T3 and T4 and (z) the
VI. Pupil dilation (IQQJS 2021)
hypothalamus fails to secrete TRH. In the table below,
match the hormone levels in column A with the disease (a) I, II and VI (b) I, IV and V
conditions in column B. (c) III, IV and VI (d) III and V

112 Class-X BIOLOGY PW


4. Dendritic membrane can change in response to
School Level charges in environment or forces.
As a result, the impulse is created which travels through
axon to synaptic knob which releases neurotransmitters
Quick Recall that carries information to the next neuron.
Fill in the Blanks These neurotransmitters are not found in dendrites and
hence, the flow of signal is unidirectional in a neuron.
1. Stimuli 2. Forebrain
3. Hypothalamus 4. Pons and medulla oblongata 5. Reflex action is an automatic, sudden, involuntary
response to a sudden change in the environment.
5. Medulla oblongata 6. Thalamus
7. Nastic 8. Auxin As reflex action does not involve integration by the

R
9. Cytokinin 10. Pituitary CNS, it is a direct route which reduces response time
and saves us from potent danger. Reflex actions are

A
True and False Statements also simple and very basic response paths developed

M
1. False 2. True 3. True over time.
4. True 5. True 6. False 6. Tongue has gustatory receptors and nose has olfactory

U
7. False 8. True 9. True receptors.
10. False
K
7. Testosterone in males and estrogen in females is
responsible for the changes.
T
Match the Following
8. Hormones are secreted by endocrine glands. These
N
1. (d) 2. (a) 3. (d) 4. (b) 5. (c)
glands do not have ducts to carry these hormones.
A

Multiple Choice Questions Thus, they are released directly in the blood stream
that carries them to target organ.
SH

1. (a) 2. (d) 3. (b) 4. (d) 5. (c)


6. (c) 7. (b) 8. (b) 9. (b) 10. (b) 9. The man is suffering with the disease diabetes
11. (a) 12. (a) 13. (a) 14. (a) 15. (d) mellitus.
A

Insulin hormone secreted by pancreas is responsible


PR

Assertion & Reason Type Questions for this disease.


1. (a) 2. (a) 3. (c) 4. (c) 5. (a) 10. Pituitary gland.
Statement Type Questions (i) Deficiency of growth hormone causes dwarfism
R.

1. (c) 2. (d) 3. (c) 4. (a) 5. (b) (short height).


D

(ii) Excess secretion of growth hormone cause


Subjective Questions gigantism in a person.
11. (i) Growth hormone, (ii) Thyroxine
Very Short Answer Type Questions 12. Iodine is a component of thyroxine hormone which
controls the basal metabolism. Hence, iodine must be
1. Cerebellum coordinates muscular movements, both
a component of our diet.
voluntary and involuntary, initiated by cerebrum.
Deficiency might cause hypothyroidism and goitre
It also maintains equilibrium and posture. Hence, characterized by swollen neck and lethargy.
these actions will be affected.
13. Abscisic acid. It increases tolerance of plants to
2. (a) Cerebrum various kind of stresses.
(b) Medulla oblongata
14. When we touch a sensitive plant like Mimosa pudica,
3. The reason maybe that continuous stimulation has the base of its leaves loses turgor and they fold. This is
depleted the neurotransmitters in the synaptic knobs. a nastic movement as it is not associated with growth.

Control and Coordination 113


15. Chemotropism is a type of growth based response 5. The parts and functions of hind brain are:
towards the chemicals e.g., the growth of pollen tube Pons: It is a part of brain stem that connects brain with
towards embyryo sac when pollen grain lands on stigma. spinal cord besides the role in breathing.
16. Auxin forms in the shoot tip but diffuse toward the Medulla: contains centres for breathing,
part which is in shade/away from the light.
swallowing, regulation of heart beat.
The concentration of auxin on shady part stimulates
the cells in this part to elongate. Thus, the side of Cerebellum: Muscular coordination, maintaining
shoot on this side grows longer than the other part. equilibrium, balance and posture of the body.
Hence, plant bend towards light. 6. The neurotransmitter will bind to receptor and hence,
17. Covering it with papers will increase the concentration prevent it from binding to the natural receptor.
of ethylene and hence, speed up the ripening process
of mangoes. This will reduce the response. Hence, a person will not feel
pain as the neurotransmitter involved in pain is blocked.
Short Answer Type Questions
7. Sympathetic nervous system activates emergency
1. Peripheral nervous system comprises of nerves arising
mode of the body by diverting blood flow towards
from brain and spinal cord.
limbs for active movement like escaping danger
Two types of nerves namely cranial and spinal nerves

R
or fight. It also increases blood flow by increasing
carry this out. Cranial nerves originate from brain and

A
strength of contraction and speed of heart contractions.
spinal nerves originate from spinal cord.
It increases the breathing rate to increase availability of

M
2. Receptors detect the changes in our internal and
oxygen. It inhibits digestion. Hence, we can say it prepares
external environments.

U
the body for emergency situations.
Several times, they do not work. During intense
fever, we lose the sense of taste because the gustatory
receptors present on tongue are not working. During K
8. Since the drug resembles dopamine, it will bind to the
receptor and will initiate the same feeling.
T
cold, we lose the sense of smell because the olfactory Also, the usual amount of dopamine would not give
N
receptors are not working. A mild burn can densensitize the same strength of feeling and hence, the person
the skin for some time but a significant burn can cause will become dependent on the drug leading to drug
A

a permanent loss of sensation in the burnt area. addiction.


SH

3. Brain and spinal cord comprise the central nervous 9. (a) hormone (b) hind brain
system, which stores and integrates information and (c) pancreas (d) cerebellum
A

pass on the information to the organs. 10. (a) Hypothalamus (b) Pancreas
PR

Peripheral nervous system comprises of nerves arising (c) Adrenal gland (d) Testes
from brain and spinal cord.
11.
Brain is protected by membranous meninges and
R.

further by bony box called cranium. The spinal cord is Name of


Location Function
protected by the vertebral column formed from bony gland
D

disc like vertebrae and cartilaginous discs. (a) Pituitary Base of Pituitary is the master
4. gland hypothalamus gland as it controls
several other glands by
Myelinated nerve fibre Non-myelinated nerve
its hormones, eg., TSH
fibre
from pituitary controls
The axon is covered The axon is not covered
thyroid. Besides, it
with a myelin sheath. by a myelin sheath.
also secretes hormone
Schwann cell forms this
myelin sheath. prolactin, oxytocin,
ADH
The Node of the Ranvier Node of Ranvier is
is found here and nerve absent and nerve (b) Thyroid In the neck Secretes thyroxine
impulses shows saltatory impulses does not show gland region which controls basal
conduction. saltatory conduction. metabolism

114 Class-X BIOLOGY PW


(c) Pancreas In the Secretes insulin which 3.
abdominal reduces blood-sugar Cerebrum (A)
region below levels and glucagon Thalamus (B)
the stomach which increases Pons (C)
blood-sugar levels. Cerebellum (D)
Medulla oblongata (E)
12. (i) Giberellin (ii) Cytokinin
(iii) Abscisic Acid (iv) Auxin 4. A process by which a product controls its own
13. (a) Plants use hormones to communicate. They also production is known as a feedback mechanism. This
show movements due to growth or changes in turgor. regulates the production of a variety of hormones.
When we touch a sensitive plant like Mimosa pudica, Thyroxine levels are detected and controlled by
the base of its leaves loses turgor and they fold. the hypothalamus and pituitary gland. Thyroxine
(b) (i) gibberellin is an inhibitor of TSH and TRH release. TRH from
(ii) cytokinin hypothalamus stimulates pituitary to release TSH
which in turns stimulates thyroid gland to secretes
14. Auxins: Auxin induces cell elongation, induces
thyroxine.
apical dominance, initiates root formation, prevent
Low levels of thyroxine in blood triggers the

R
abscission, induces parthenocarpy and is also used as
production of TRH (Thyrotropin releasing hormone),
herbicide in synthetic forms.

A
which in turn triggers the pituitary to release TSH
Gibberellin promotes stem elongation, seed (Thyroid stimulating hormone), causing the thyroid to

M
germination, breaks dormancy, causes elongation of produce more thyroxine. As a result, thyroxine levels
internodes and induces parthenocarpy.

U
in blood are returned to normal.
Long Answer Type Questions Thyroxine (the response) here was reducing the
1. Reflex arc is the path followed by a reflex action. In K
stimulus (by inhibiting the release of TSH). This is an
example of negative-feedback mechanism
T
the diagram below, as hands touch the hot pan, afferent 5. (a) Pituitary is often called as master gland because
neuron transmit the information to the spinal cord
N
it controls and coordinates the secretion of all the
where the relay neuron passes it to the efferent neuron other endocrine glands.
A

which causes the muscle to contract and pull our hand. (b) Pancreas secretes pancreatic juice and hormones
SH

Spinal cord Message to


brain
insulin and glucagon. Pancreatic juice helps in
(CNS)
digestion whereas insulin and glucagon regulates
blood sugar level.
A

(c) Adrenaline is secreted by adrenal glands. During


Sensory neuron emergency conditions, adrenaline is secreted in
PR

Motor
neuron large amounts. As a result, the heart beats faster
Receptors = Heat/Pain Relay neuron and rate of respiration in increased in order to
Receptors in skin supply more oxygen to our muscles. The blood
R.

Effector (Muscle in arm)


to the digestive system and skin is reduced due
to contraction of muscles around small arteries in
D

these organs. This diverts the blood to our skeletal


muscles. All these responses together enables the
2. Hypothalamus (A) animal body to get ready to deal with the situation.
6.

Thyroid Gland (D) Name of the Endocrine Function


Hormone Gland
Thymus (E)
(i) Insulin (ii) Pancreas Deposits extra
glucose of blood as
Adrenal Glands (B) glycogen
Pancreas (C) Growth (iii) Pituitary (iv) G
 rowth of the
Hormone body

Control and Coordination 115


(v) Thyroxine Thyroid (vi) regulates basal Case-Based Type Questions
metabolic rate
Oxytocin (vii) Posterior (viii) Milk ejection Case Study-I
lobe of 1. (a) 2. (b) 3. (d) 4. (b)
pituitary
Case Study-II
7. Control and coordination requires establishing 1. 1-Radicle, 2-Plumule
communication between body parts to ensure multiple 2. Hydrotropism and geotropism
functions to happen at the same time to ensure a larger 3. Part 1 is affected by water and gravity, water is stronger
function. Sight of food (message) is sent by neurons to stimuli as radicles are curving upwards towards moist
the brain which initiates salivation to ensure digestion sawdust.
begins. As food reaches stomach, the stretch receptors 4. Thigmotropism is movement towards touch, e.g.,
stimulate the release of hormone gastrin which tendrils of pea plant grow towards support.
increases stomach’s motility and secretion. 5. Positive tropic movement means movement towards
Brain and spinal cord allow us to detect the food and the source of stimulus and negative tropic movement

R
take appropriate action to get the food inside (picking means movement away from the source of stimulus.
up a plate of samosa and eating it). At the same

A
time, brain controls the digestive system so, that it is Case Study-III

M
ready to digest. Changes in environment, external or 1. The given gland is pituitary and it is located at the
internal to body, can also trigger release of chemical base of hypothalamus in the brain.

U
messengers called hormones. 2. It is called the master gland as it controls secretions of
These chemical messages are being sent all over
the body by endocrine glands via blood to control Kvarious other endocrine glands by its own hormones,
e.g, TSH (Thyroid stimulating hormone) released by
T
distantly located organs. pituitary controls the secretion of thyroid gland.
N
3. The deficiency of ADH (Anti-diuretic hormone)
This regular stimulation and action of nervous and
or vasopressin causes diabetes insipidus as less
A

endocrine system controls and coordinates action in


ADH means less absorption of water from filtrate
organisms.
SH

in kidneys and hence, voluminous urine discharge.


8. Geotropism is the growth of plant parts towards or Diabetes mellitus is caused by insulin deficiency.
away from direction of gravity. Stem is negatively Insulin maintains blood sugar level.
A

geotropic as it grows away from the direction of 4. (a) Posterior, (b) Anterior, (c) Anterior
PR

gravity whereas roots are positively geotropic as they


grows towards the direction of gravity. Case Study-IV
1. Thyroid stimulating hormone (TSH).
R.

2. TSH from pituitary gland stimulates thyroid gland to


Negatively
release thyroxine.
D

geotropic
3. Thyroxine regulates basal metabolism which is very
Positively crucial at the time of pregnancy as maternal body
geotropic feeds the child. Thyroxine also inhibits TSH through
9. Plant hormones are chemical substances that regulate negative feedback. Hence, TSH levels in blood are in
growth in plants. turn regulated by Thyroxine which is dependent on
(i) Auxins: Auxin induces cell elongation, maintains iodine. If the level of thyroxine in blood is low and
apical dominance, initiates root formation. TSH is high, the chances of miscarriage increases.
(ii) Cytokinin: promotes cell division. So, to protect the developing foetus, TSH level must
be maintained.
(iii) Gibberellin: promotes stem elongation.
4. If TSH levels are high, one of the possible reason is
(iv) Ethylene: promotes fruit ripening and senescence. less production of thyroxine which might be due to
(v) Abscisic acid: Abscisic acid induces stomatal poor levels of iodine in her diet. She should look at
closure, and seed dormancy. her salt consumption as salt is a good source of iodine.

116 Class-X BIOLOGY PW


11. (c) The middle layer of brain meninges is arachnoid
Competitive Level space.
12. (c) Cytokinin is a plant hormone.
Multiple Choice Questions 13. (a) Islets of langerhans is a tissue inn pancreas
composed of alpha,beta and gamma cells. Beta
1. (a) 2. (b) 3. (b) 4. (c) 5. (d) cells of this tissue synthesize and secrete insulin.
6. (a) 7. (c) 8. (a) 9. (a) 10. (b) 14. (d) P- Growth Hormone, Q- Growth hormone releasing
11. (b) 12. (d) 13. (c) 14. (c) 15. (d) hormone Release Factor, R- Insulin, S-Thyroxine.
16. (b) 17. (d) 18. (d) 19. (c) 20. (b)
15. (a) When touched the stimulus reaches the base of
21. (a) 22. (b) 23. (b) 24. (d) 25. (a) the leaf and the water in the vacuoles of the cells
26. (b) 27. (c) 28. (a) 29. (c) 30. (d) of the leaf loses water to the adjacent cell. All
the water escapes the leaf which then becomes
Competitive Corner flaccid. This causes the leaves to close. This is
1. (b) 31 pairs of nerves arise from the spinal cord. due to the passing of impulse which causes the
2. (a) Negatively geotropic roots of mangrove plants serve change of turgor pressure.

R
the purpose of the exchange of oxygen and carbon 16. (a) Positive phototropism and positive geotropism.
dioxide. These are otherwise called breathing

A
17. (a) Hypertension is the condition in which the
roots or pneumatophores. Some lateral roots of blood pressure of the concerned person is highly

M
mangroves become specialized as pneumatophores elevated. The systolic blood pressure increases to
in saline mudflats, pneumatophores are lateral roots 150 mm of Hg. The baroreceptors in the heart can

U
that grow upward (negative geotropism) for varying detect the change in the blood pressure and send
distances and function as the site of oxygen intake.
3. (c) Plants have certain chemical substances in them K signals to the medulla oblongata to regulate the
pressure. The hypertensive situation is controlled
T
that transmit messages for maintaining different by medulla oblongata.
functions of the body. One of the important
N

function of auxins is that they help in the growth of 18. (a) P-(iii), Q-(i), R-(v), S-(vii), T-(ii)
A

the plants, like the growth of the stem.Gibberellins 19. (c) Immediate changes in a squirrel’s body to fight or run:
™ Adrenaline causes an increase in heart-pumping
SH

help in the growth of different plant parts like stems.


Cytokinins help in cell division and differentiation, rate and respiratory rate.
and so option iii is incorrect. Abscisic acid retards ™ It suppresses the activities of the digestive system
A

plant growth, so option iv is also incorrect. and reproductive systems for a certain period of time.
™ The amygdala responds to the threat
PR

4. (b) The dietary iodine serves as a source and thus, the


™ It also causes increased blood pressure, pupil
injected radioactive iodine would be incorporated dilation, etc.
into the thyroid gland. ™  The hypothalamus activates the sympathetic
R.

5. (b) Auxin delays the abscission of young leaves and nervous system and releases adrenaline from the
fruits. Its effect is through non-formation of the glands near the kidneys.
D

abscission zone below a leaf or fruit. Abscisic 20. (a) A–(i), B–(ii), C–(vii), D–(iii), E–(v)
acid inhibits seed germination.
21. (d) The flower will bend towards the light because of
6. (c) The correct direction of the flow of nerve impulse is
phototropism.
from dendrites → cell body → axon → synapse →
dendrites. 22. (d) (x)-(iii); (y)-(i); (z)-(ii)
7. (b) ADH/ antidiuretic hormone or vasopressin is 23. (d) GA hormone is responsible for increasing the
secreted by the posterior lobe of the pituitary gland. length of internodes.
8. (b) Our skin becomes dark in colour when exposed to 24. (c) Sympathetic nervous system effects on the
excess of sunlight. It is due to presence of melanin body includes: Dilated pupils, Inhibit salivation,
pigment in the skin. Relaxes bronchi, Increased heartbeat, Slows down
9. (a) Q represents pituitary gland. digestion, Stimulates glucose release, Inhibit
10. (c) Oxytocin promotes lactation and simulates the activity of intestines, Adrenaline production,
uterus to contract. Reduces blood flow.

Control and Coordination 117


3
CHAPTER
How Do Organisms
Reproduce?

School Level

Introduction
What is world’s largest organism? Was your answer the blue whale or the Red Wood trees of California. It is wrong!

R
What is further interesting is the title goes to a fungus -Armillaria ostoyae! This fungus grows across 2384 acres in Blue
Mountains of Oregon. But, has reproduction got something to do with it or was this fungus always this big?

A
‰ Reproduction is the production of new organisms from the existing organisms of the same species. It is essential for the

M
survival of species present on this earth. The process of reproduction ensures continuity of life on earth.

U
‰ It is not an essential life process, this means that it is not necessary to reproduce in order to survive, yet it is a unique
property of living beings.
K
‰ In reproduction, a lot of energy is needed to produce a new individual.
T
N
Reproduction, Variation & Evolution
A

‰ Reproduction at the cellular level involves making similar or dissimilar body designs through the genetic material
SH

(DNA) present in the chromosomes within the nucleus.


‰ DNA in the nucleus of cell is the source of information for making proteins. Any change in the information leads to the
formation of different proteins, which eventually lead to altered body designs. We term this as variation.
A

‰ The basic event in reproduction is creation of DNA copies in a reproducing cell. This process is called DNA replication.
PR

A cell, after replication, will divide into two; each new cell gets a copy of each DNA with its cellular apparatus.
‰ Complete accuracy in DNA copying reactions leads to the formation of two exactly identical cells but error is inevitable.
R.

Although these errors are corrected systematically, some errors will escape correction and a variation results
D

‰ Variations refer to the differences in structure, physiology and other traits found in individuals of species. The variations
may offer an advantage in the changing environment or may be detrimental to the organism. It might also be neutral. If it
is advantageous or disadvantageous, it will impact the reproductive capacity of the organism. The species may increase
or decrease in population. This is evolution.
‰ If there were a population of bacteria living in temperate water and if the water temperature were to be increased by
global warming, most of these bacteria would die but the few variants resistant to heat would survive and grow further.
A variation could be useful for the survival of species over time.

Asexual reproduction
‰ Production of offspring by a single parent without the formation and fusion of gametes.
‰ It is a more primitive type of reproduction. It occurs much faster and requires less energy.
‰ Asexual reproduction is observed mostly in unicellular organisms such as bacteria, protozoans, some plants like algae,
fungi, bryophytes, etc., and some multicellular animals such as sponges, coelenterates, certain worms and tunicates.
However, it is absent in higher invertebrates and all vertebrates. (However some animals undergo a process called as
parthenogenesis)
‰ The new individuals produced by asexual reproduction are exact copies of their parents (genetically identical), i.e., there
is no or little variation.

Types of asexual reproduction


Fission, fragmentation, regeneration, budding, vegetative propagation and spore formation are the various types of asexual
reproduction.

1. Fission
‰ Many single-celled organisms reproduce by this process where a cell divides into two halves and rapidly grows into an
adult. It occurs in two steps, the division of the nucleus followed by the division of cytoplasm.
Types of fission

R
(i) Binary fission

A
 In this type of reproduction, the nucleus first divides into two nuclei (karyokinesis) which is followed by the division

M
of cytoplasm (cytokinesis). Finally the cell disintegrates into two daughter cells which grow fully and divide again.
It can be seen in bacteria, yeast, Amoeba, Leishmania and Euglena.

U
Splitting can happen across different planes

K
™ Longitudinal binary fission takes place along the longitudinal axis of a cell. E.g., Euglena, Leishmania
™ Transverse binary fission takes place along the transverse axis of the organism. E.g., Paramecium, Planaria,
T
diatoms and bacteria.
N
A
SH
A
PR

Fig. 1: Transverse binary fission in Amoeba Fig. 2: Longitudinal binary fission in Leishmania
R.

(ii) Multiple fission


D

Parent cell divides into many small daughter cells simultaneously. It occurs in many protozoans such as Plasmodium,
Amoeba (in unfavourable conditions) and Monocystis.

Fig. 3: Multiple Fission in Plasmodium

How Do Organisms Reproduce? 119


The process of multiple fission involves the following steps:
‰ Under unfavourable conditions such as lack of food, oxygen, excessive heat, cold or drought, etc., a cyst (protective
wall) is formed around the parent cell. Inside the cyst, the nucleus of the parent cell divides multiple times to produce a
large number of nuclei. Each daughter nuclei is surrounded by a small amount of cytoplasm and a thin membrane. As a
result, many daughter cells are formed inside the cyst.
‰ Under favorable conditions, the breakdown of cyst releases the daughter cells. All the daughter cells are identical. Each
daughter cell gives rise to a new organism.
Thus, multiple fission results in producing large numbers of offspring.

2. Budding
‰ A small part of the body of the parent organism develops a bud as an outgrowth due to repeated cell division at a specific
site which then detaches and become new independent organism. It is observed in Yeast (Saccharomyces) and Hydra.
Bud

R
A
M
U
Parent cell
Fig. 4: Budding in Yeast
Tentacles
K
T
N
A

Ultimately Fully mature


SH

Body Bud Hydra


develops bud bud
Fig. 5: Budding in Hydra
A

3. Fragmentation
PR

Due to mechanical injury or similar reasons the body of a simple organisms could break into fragments. These fragments
can mature to form individual organisms. It is commonly observed in algae (Spirogyra), fungi, bryophytes and some
R.

marine ribbon worms.


D

Grows
Fragment 1

Grows
Fragment 2

Grows
Fragment 3
Fig. 6: Fragmentation in Spirogyra

120 Class-X BIOLOGY PW


4. Regeneration
The ability to give rise to a new individual from the parent individual's body parts is called regeneration. It is carried out
by specialised cells which proliferate and make large number of cells. Hydra and Planaria can be cut into any number
of pieces and each piece grows into a complete organism.

R
A
Fig. 7: Regeneration in Planaria

M
5. Spore formation

U
‰ It takes place in multi-cellular organisms.
‰ It can be observed on the bread mould (Rhizopus). Fungi have thread-
like structures called hyphae. K
T
‰ Sporangiophore is a type of hyphae bearing a specific reproductive
N
part, called sporangium, which produces spores.
A

‰ The nucleus of developing sporangium develop several times.


SH

‰ Each nucleus within the developing sporangium gets surrounded by a


bit of cytoplasm and develops into a spore.
‰ The spores mature inside a sporangium and are dispersed by rupturing
A

of the sporangial walls.


PR

Fig. 8: Spore formation in Rhizopus


‰ Each spore grows into a new hypha after germinating in new habitat
or substratum. Example: Rhizopus, Mucor etc.
R.

6. Vegetative Propagation
It is the simplest method of asexual reproduction in plants. The vegetative parts like the
D

apical bud
root, stem, leaf, etc, develop into new plants under appropriate conditions. Vegetative
reproduction is classified into two types: natural and artificial. lateral bud
(eye)
Natural Methods of Vegetative Propagation scale leaf
Roots, stems or leaves are the vegetative structure for reproduction.
‰ By roots: The tuberous roots of sweet potato bear adventitious buds (bud-bearing roots).
These adventitious roots when planted in the soil, can give rise to new plants. lenticels
‰ By stems: In common grass and mint, new roots and shoots develop at specific regions stem end
called as nodes. Their stems grow horizontally parallel to the ground. The roots grow Fig. 9: Vegetative propagation
downward into the soil and the shoots grow upward into new plants. through tuber
Ginger is a modified stem with nodes, internodes and scaly leaves. The axillary bud
growing out from the nodes develop into a new plant.

How Do Organisms Reproduce? 121


‰ By leaves: Leaves of some plants such as Bryophyllum, produce buds in the notches along the leaf margin. When such
leaves fall on the moist soil, these buds develop into new plants.

Fig. 10: Leaf of Bryophyllum with adventitious buds

Knowledge Hub
Mosses and liverworts, for example, often have cup-like structures on their body (thalli) that contain specialized
structures called gemmae. Gemmae which are green, multicellular asexual buds that become detached from the
parent body and can germinate to form new plants, which are genetically identical to the parent.

R
A
Advantages of Vegetative Propagation

M
‰ It is a rapid, cheap and easy method of reproduction for the multiplication of plants.
‰ Plants raised by vegetative propagation can bear flowers and fruits earlier than those produced from seeds.

U
‰ The propagation of plants is also possible that have lost the capacity to produce seeds. E.g., banana, seedless grapes,
rose and jasmine.
K
T
Limitations of Vegetative Propagation
N

‰ Since it produces genetically identical plants, new varieties cannot be obtained.


A

‰ The plants are more susceptible to diseases because there is no genetic variation.
SH

‰ There is severe competition between the members of the same species due to overcrowding of many plants near the
parent plants. This leads to production of weak plants with stunted growth.
A
PR

Sexual reproduction
Sexual Reproduction is defined as the production of offspring by the fusion of two haploid gametes (one from male parent
R.

and other from female parent) to form a diploid zygote which develops into a mature organism.
D

After attainment of maturity, all sexually reproducing organisms exhibit events and processes that have remarkable
fundamental similarity, even though the structures associated with sexual reproduction are indeed very different. The events
of sexual reproduction though elaborate and complex, follow a regular sequence. Sexual reproduction is characterised by
the fusion (or fertilisation) of the male and female gametes, the formation of zygote and embryogenesis. For convenience
these sequential events may be grouped into three distinct stages namely, the pre-fertilisation, fertilisation and the
post-fertilisation events.

Events of Sexual reproduction


(a) Gametogenesis - Formation of male and female gametes.
(b) Fertilization - Fusion of gametes
(c) Zygote formation
(d) Development of zygote into an embryo and embryo to an adult individual.

122 Class-X BIOLOGY PW


Sexual reproduction has several advantages over asexual reproduction because it involves
‰ Fusion of male and female gametes coming from male and female organisms. Since the fusing gametes come from two
different and sexually distinct individuals, the offspring exhibit diversity of characters.
‰ Meiosis during gametogenesis provides opportunities for new combinations of genes. It plays a prominent role in the
origin of new species and leads to variation required for evolution.

Sexual reproduction has a few disadvantages as well


‰ Organisms spend much energy in production of gametes.
‰ Organisms spend energy in attracting mates or transferring gametes.

Variation During Sexual Reproduction


‰ Gamete formation, inside the gonads of diploid organisms, involves meiosis or reduction division. The gamete mother
cell is diploid (2n), i.e., it has two sets of chromosomes.

R
‰ This single diploid cell divides by meiosis to form 4 haploid daughter cells. Each daughter cell becomes a gamete. Each

A
gamete possesses a single set of chromosomes. That means this division involves copying of the DNA as well as the

M
cellular apparatus.

U
‰ There is a stage in such nuclear division where crossing over of chromosomes takes place. This is a very important
step which results in a slight different composition of chromosomes in gametes. Fusion of these gametes results in the
formation of slightly different individuals which show variations. K
T
‰ The variations which lead to the appearance of such characters which fit to the changing environment result in the survival
N

of the species. Chances of variation, therefore, are much more in sexual mode of reproduction as compared to asexual
A

reproduction. Moreover, chances of the production of compatible generations are also higher in sexual reproduction.
SH

Sexual Reproduction in Flowering Plants


A

‰ In flowering plants or angiosperms, sexual reproduction takes place through flowers. Although it is pertinent to stress,
PR

not all plants are flowering plants.


‰ Flower is a modified condensed shoot meant for sexual reproduction.
R.

‰ Flowers may be unisexual or bisexual. In bisexual flowers, both male and female reproductive structures are present,
D

e.g., Hibiscus, mustard, while in unisexual flowers, only single reproductive structure, either male or female, is present,
e.g., papaya, watermelon.

Phases in life cycle of a Plant


‰ All organisms have to reach a certain stage of growth and maturity in their life, before they can reproduce sexually.
That period of growth is called the juvenile phase. It is known as vegetative phase in plants. This phase is of variable
durations in different organisms.
‰ The end of the juvenile/vegetative phase which marks the beginning of the reproductive phase can be seen easily in the
higher plants when they come to flower.
‰ Reproductive phase is the phase that occurs when an individual organism reaches reproductive maturity and becomes
sexually active, as the name implies. They reproduce and create progeny.

How Do Organisms Reproduce? 123


SEXUAL REPRODUCTION
IN PLANTS

Pre-fertilisation Fertilisation Post-fertilisation


events events
Fusion of Male
Zygote develops
and Female
Gametogenesis Gamete-transfer into Embryo
gametes
Ovule changes
Pollination
Generation of Generation of into seed
male Gametes female Gametes
i.e. Egg cell Integuments →
seed coat
Each pollen
grain produces Ovary develops
2 male gametes into fruit

R
Structure of a Flower

A
A typical angiospermous flower consists of four whorls

M
attached on the receptacle thalamus. The floral axis is

U
made up of two parts – a stalk and a thalamus (swollen
end of stalk). A typical flower consists of four different
kinds of whorls, namely, calyx (sepals), corolla (petals), K
T
androecium (stamens) and gynoecium (carpels). They are
called whorls as they are arranged in concentric fashion.
N

Calyx and corolla are the accessory organs while


A

androecium and gynoecium are the reproductive organs.


SH

Flowers could be unisexual (papaya, watermelon)


consisting of only one of the sexual whorl. Bisexual
A

flowers contain both reproductive whorls.


1. Calyx: It is the outermost green whorl and consists
PR

Fig. 11: Longitudinal section of flower


of sepals. They are leaf-like and provide protection
to the flower bud before it opens.
R.

2. Corolla: It is the second whorl, inner to calyx, and consists of petals. Petals are generally large, coloured and bright.
Corolla helps in attracting insects for pollination.
D

3. Androecium: It is the third whorl, inner to corolla, and consists of stamens (male reproductive parts).
Each stamen has two parts: Filament or stalk and anther. Anther produces pollen grains which are yellowish in colour.
Each pollen grain produces two male gametes.
4. Gynoecium: It is the fourth and innermost whorl which consists of carpels (female reproductive parts). Carpel or pistil
is present in the centre of flower.
Each carpel has three parts: Stigma, Style and Ovary.
(i) Stigma: The terminal part of style is called as stigma. The stigma is normally rough, hairy or sticky to hold
pollen grains during pollination process.
(ii) Style: From the top of the ovary arises a long, elongated structure called as style.
(iii) Ovary: It is a basal, swollen part of the pistil. The ovary is the future fruit. Inside it has ovules, which will form
the seed post-fertilisation.

124 Class-X BIOLOGY PW


Pollination
The transfer and deposition of pollen grains shed from the anther to the stigma of a flower is termed as pollination.
Types of pollination:
1. Self pollination: It is the process of transfer of the pollen grains from the anther to the stigma of the same flower. It is
further divided into two types:
(i) Autogamy: Transfer of the pollen grains from the anther to stigma of the same flower, e.g., wheat, rice, etc.
(ii) Geitonogamy: Transfer of pollen grains from the anthers of one flower to the stigma of another flower of the
same plant, e.g., corn, castor.
Significance of self-pollination:

 Not dependent on pollinating agents.
 Less wastage of pollen grains.
 Preserve parental characters, thus maintaining the purity of species.

R
A. Pollen B.

A
Anther Egg cell Pollen

M
Sperm Anther

U
Pollen
tube
K Ovule
T
Ovule
N

Female Male
A

Bisexual/
Hermaphrodite/ Unisexual/
SH

Monoecious Dioecious
Fig. 12: Pollination in different types of flowers
A

2. Cross-pollination/Xenogamy: Pollen grains are transferred from the anther of one flower to the stigma of another
flower borne on a different plant of the same species.
PR

Significance of cross-pollination:

 Increase in yield and adaptability.
R.

 Since it involves two varieties of the same or different species, it can eliminate defective traits and produces new
varieties.
D

 It also leads to the production of hybrids.

Agents of Pollination
‰ The agents or media which can transfer the pollen grains are called as agents of pollination.
‰ Plants use two abiotic (wind and water) and one biotic (animals) agents to achieve pollination.
‰ Majority of plants use biotic agents for pollination. Only a small proportion of plants use abiotic agents. Pollen grains
coming in contact with the stigma is a chance factor in both wind and water pollination. To compensate for this
uncertainties and associated loss of pollen grains, the flowers produce enormous amount of pollen when compared to
the number of ovules available for pollination.
‰ Insects come to flowers in greed for rewards. These rewards are sweet sugar substances-nectar-produced by flowers.
While obtaining nectar, pollen grains stick to the insects and are transferred as the insect hops on to the next flower of
the same kind. This throughout evolution has increased the reproduction efficiency of flowering plants by a lot.

How Do Organisms Reproduce? 125


Agents of
pollination

Abiotic agents Biotic agents

Water Wind Insect Birds Bats


(Hydrophily) (Anemophily) (Entomophily) (Ornithophily) (Chiropterophily)

Fertilisation
Fertilisation is the process of the fusion of the male and female gametes. As the pollen grains fall on the stigma, they
absorb water and nutrients. A mature pollen grain consists of two male gametes.
The pollen tube, after reaching the ovary, penetrates the embryo sac. The pollen tube bursts open due to which the male

R
gametes get released into the embryo sac.

A
‰ One of the male gamete fuses with the egg and this results in the formation of zygote (diploid cell). This is termed as
syngamy.

M
‰ The other male gamete fuses with the polar nuclei to produce triploid primary endosperm. As this involves the fusion

U
of three haploid nuclei, it is termed triple fusion nucleus. The triple fusion nucleus is also called endosperm nucleus

K
because it develops into the endosperm. Endosperm is filled with reserve food materials and are used for the development
of the embryo.
T
This process of fusion of one of the male gametes with egg and the other male gamete with the secondary nucleus is
N

known as double fertilisation.


A

Post Fertilization Changes in a Flower


SH

Post Fertilization changes


A

Zygote Embryo
Ovule Seed
PR

Integuments Seed coat


Ovary Fruit
R.

Knowledge Hub
D

‰ True fruits are those part which are formed completely from ovary of a flower e.g., Mango.
‰ False fruits are those which develops from any other part of flower like thalamus e.g., Apple, Strawberry.

Structure of Seed
Seed is a fertilised ovule that contains the embryo. It is the final product of sexual reproduction. Seeds of different plants
vary in their size and shape.
Every seed has outer coverings called seed coats which develop from the integuments of the ovule. The hilum is a scar
on the seed coat through which the developing seeds were attached to the fruit. One end of embryo axis bears radicle
and other end bears plumule. The radicle when elongates gives rise to primary root whereas the plumule gives rise to
aerial shoot.

126 Class-X BIOLOGY PW


Germination of Seed
‰ As the seed matures, its water content is reduced and seeds
become relatively dry (10-15 per cent moisture by mass). Seed coat
The general metabolic activity of the embryo slows down. Hilum
micropyle
The embryo may enter a state of inactivity called dormancy,
or if favourable conditions are available (adequate moisture,
oxygen and suitable temperature), they germinate.
‰ Seed is the basis of our agriculture. Dehydration and
Fig. 13: Germination of seed
dormancy of mature seeds are crucial for storage of seeds
which can be used as food throughout the year and also to raise crop in the next season.
‰ Germination is the process by which the dormant embryo within the seed becomes active, grows and forms a seedling
or a young plant capable of independent existence.

Activity

R
A
Aim: To show that water, oxygen and suitable temperature is required for the germination of seed.

M
Procedure: Label three containers A, B and C and put dry cotton wool in the bottom of each.

U
Place equal numbers of soaked seeds in all three as shown in the figure in a normal room condition. Leave A quite

K
dry; add water to B to make the cotton wool moist; add water to C until all the seeds are completely covered. Put
lids on the containers and leave them all at room temperature for a week
T
A B C
N
A
SH

Soaked peas, dry cotton wool Soaked peas, wet cotton wool Soaked peas, covered cotton water
Observation: We will observe that seeds in the container B will germinate normally. Those in A will not germinate.
A

The seeds in C may have started to germinate but will probably not be as advanced as those in B because the seeds
in container C do not have oxygen.
PR

Conclusion: Water is required for germination.


R.

Try it Yourself
D

1. What is the advantage to the male and female gamete in plants that we do not have?
2. If cross pollination is advantageous, why not all plants show cross pollination?
3. Now that you understand the importance of pollinators, if an insecticide spread on crops kills bees what
would be the impact of food available to humans?

Sexual Reproduction in Animals


‰ The reproductive organs are known as gonads. The male gonad is testis and the female gonad is ovary. Testis produces
the male gametes called sperms or spermatozoa. Ovary produces female gametes called eggs (ova).
‰ In lower organisms such as Hydra, tapeworm, earthworm, etc., the male and female sex organs are present in the same
organisms. Such animals are said to be bisexuals or hermaphrodites.

How Do Organisms Reproduce? 127


SEXUAL REPRODUCTION
IN ANIMALS

Pre-fertilisation Fertilisation Post-fertilisation


events events
Fusion of male
and female
Gametogenesis Gamete-transfer
gametes

Transfer of
Spermatogenesis Oogenesis
sperms to the Gestation Child birth/
female genital Parturition
Synthesis of Synthesis of ovum/ tract Period of 9
sperm in testis egg cell in ovaries
months of
which starts at which starts
during embryonic
pregnancy
puberty
development

R
Reproduction in Humans

A
M
Primary and Secondary Sexual Characters

U
Primary characteristics are present at birth and comprise the external and internal genitalia. These distinguishes one sex

K
from the other. It includes testis and penis in males and ovary and vagina in females. Sexual maturity begins around 8 years
T
in girls and 10 years in boys. This period is referred to as adolescence. During this period, physiological and anatomical
N
changes are observed that makes girls and boys sexually functional and capable of reproduction. They can reproduce by
the end of puberty but they should wait until they are mentally and socially mature to handle the responsibility.
A
SH

Secondary sexual characteristics are other somatic features of individuals that appear during puberty. They also
distinguish the two sexes of a species, but are not directly related to the reproductive system. These characteristics are
A

expressed due to hormones released at puberty.


PR

Secondary Sexual Characters-at puberty stage (10-15 years old)


In males In females
Appearance of hairs over the face, under armpit and Appearance of hairs under arm pit and around the sex
R.

around the sex organ organ


D

Deepening of voice Sharpening of voice


Muscularity, widening of shoulder Softening of skin, widening of hip bone
Enlargement of the sex organ Enlargement of breast and the sex organ
Starting of sperm production Starting of menstruation
On the basis where the development of zygote takes place, animals are categorised into two types:
Oviparous Viviparous
The development of the zygote occurs outside the body The development of the zygote occurs inside the body
of the female parents in oviparous organisms. of the female parent in viviparous organisms.
These animals lay eggs and the fertilized eggs have a These animals give birth to young ones.
calcareous shell to protect them from harsh environment.
E.g., Reptiles, amphibians, birds. E.g., Human, whales, mice, etc.

128 Class-X BIOLOGY PW


The Male Reproductive System
Male reproductive system consists of:
‰ A pair of testes.
‰ A paired duct system consisting of vas deferens,
epididymis, ejaculatory duct and urethra.
‰ Accessory glands like seminal vesicles
(Paired), Bulbourethral / Cowper's (Paired)
glands and prostate glands (Single).
‰ Male external genitalia - Penis.

Testes (singular: testis)


It is the primary sex organ. The testes or testicles
are paired oval glands and are situated outside
the abdominal cavity within a pouch called

R
Fig. 14: Diagrammatic representation of human male reproductive system
scrotum. Scrotum act as a thermoregulator thereby

A
maintaining a temperature 2-2.5 °C lower than the

M
normal internal body temperature. A lower temperature is necessary for the maturation of sperms.
Testis also produces the male sex hormone, testosterone.

U
Knowledge Hub K
T
N
™ Luteinising hormone secreted by the anterior pituitary gland stimulates the Leydig cells/ Interstitial cells present in
the interstitial spaces in testis which secretes testosterone hormone in males.
A
SH

Path of Sperm
‰ Epididymis is a tightly coiled structure containing a series of thread-like tubules that form a comma-shaped structure
A

on the posterior side of the testes. It temporarily stores the sperm produced by the testis.
PR

‰ Vasa deferentia (singular: Vas deferens) is also known as sperm duct. It emerges from the epididymis and ascends along
the posterior side of the testis to become associated with the blood vessels and nerves that supply the testes. It comes out
of the scrotum, passes over urinary bladder and receives a duct from seminal vesicle to form ejaculatory duct.
R.

‰ Ejaculatory duct: This leads the sperm to urethra.


D

‰ Urethra: The male urethra extends from the urinary bladder to the distal end of the penis. The urethra is a common
passage for both urine and male reproductive fluids. Since, both of these fluids do not exit through the urethra at the
same time therefore, there is no mixing.

Accessory Glands
‰ Seminal vesicles also called seminal glands are present behind the urinary bladder and the rectum. Its secretion acts as the
energy source for the sperms. The alkaline nature of the fluid helps to neutralise the acid in the female reproductive tract.
‰ Prostate gland is a single bilobed structure. It surrounds the urethra close to its origin from the bladder. The prostatic
secretion provides to sperm motility and viability.
‰ Cowper's glands are two small ovoid glands which open into the urethra just before it enters the penis. It secretes mucus
that provides lubrication at the end of the penis during sexual intercourse. The alkaline nature of the secretion protects
the, passing sperms by neutralizing the acids coming from urine in the urethra.

How Do Organisms Reproduce? 129


Structure of Sperm
Head
A sperm is a haploid male gamete which fuses with the Tail Middle Piece
ovum to form the diploid zygote. Sperm composed of (Flagellum)
four parts - the head, neck, a middle piece and a tail.
‰ Head is almost oval in the human sperm. It contains Neck
cap-like structure, acrosome and haploid nucleus Mitochondrian Acrosome
Centriole
enclosed by a thin membrane. Acrosome is filled Plasma
with enzymes that help fertilisation of the ovum. Membrane Nucleus

‰ Neck consists of two centrioles Fig. 15: Structure of spermatozoan


‰ Middle Piece consists mitochondria which produces energy for the movement of tail that facilitate sperm motility.
‰ Tail is a eukaryotic flagella that helps the sperm move

Penis
The penis is the male reproductive organ for sexual act or copulation. It helps in the transfer of sperm cells from the male
to the vagina of the female. It is made up of special tissue that helps in the erection of the penis.

R
A
Try it Yourself

M
1. What is the function of the following glands in male reproductive organ?

U
(a) Cowper's gland (b) Prostate gland
2. What is the function of the following parts in spermatozoan?
(a) Middle piece (b) Acrosome
K
T
N

The Female Reproductive System


A

The female reproductive system consists of


SH

‰ A pair of ovaries.
‰ A pair of fallopian tube or the oviducts.
A

‰ Uterus
PR

‰ Vagina
‰ Accessory glands
R.

‰ External genital organs.


Fig. 16: Diagrammatic representation of human t
D

reproductive system
Ovaries
It is the primary sex organ. These are a pair of almond-shaped female
gonads dark black in colour present on each side of the lower abdominal.
Corona radiata
They are connected to the pelvic wall and uterus by ligaments. The ovary
Zone pellucida
contains ovarian follicles. Follicles are bag-like structures that produce
the female gamete, the ovum. It also produces female sex hormones, i.e., Nucleolus
estrogen and progesterone. Nucleus
Cytoplasm
Structure of Ovum:
‰ Ovum is round, nonmotile haploid cell and its size is 0.1mm (larger Plasma membrane
than other body cell).
‰ Ovum is enclosed by inner thin transparent called zona pellucida and
outer thick multilayerd corona radiata. Fig. 17: A Human Ovum

130 Class-X BIOLOGY PW


Fallopian tubes or Oviducts
It extends from the periphery of each ovary to the uterus. Near the ovary, it possess finger-like projections called fimbriae.
Fimbriae helps in collection of the ovum after ovulation(release of ovum). Towards the uterus end it provides a route for
the sperm to reach the ovum and after fertilization carry the fertilized ovum to the uterus. In the mid, generally the sperm
and ovum fuse and hence the oviduct is the site of fertilization.

Uterus or womb
‰ It is like an inverted pear, muscular thick walled but distensible bag like organ. It is supported by ligaments attached to
the pelvic wall. It protects and nourishes the developing embryo. The uterus opens into vagina through a narrow part,
cervix.
‰ The sperms deposited in the vagina pass through the uterus to reach the fallopian tube.
‰ It has an inner glandular layer called endometrium that lines the uterine cavity and is the site where developing embryo
(if fertilization has occurred) attaches for getting nutrition from the mother. This is called implantation.
‰ Uterus undergoes cyclic changes every 28 days due to hormones. It thickens and develops rich vascular supply to prepare
for implantation. If fertilization does not occur, and hence implantation does not occur, the blood and endometrial tissue

R
is discarded as menses or commonly called as periods.

A
‰ When implantation does not occur, the endometrium undergoes cyclical changes during menstrual cycle.

M
Vagina

U
‰ The uterus opens into the vagina through cervix.

K
‰ It is the female copulatory organ, birth canal as well as passage way for menstrual flow. In young females, the opening
of the vagina is partially covered by a thin mucous membrane called the hymen.
T
N
Accessory Glands
A

‰ Bartholin's glands are a pair of small glands. These are situated at the left and right of vaginal opening. Their secretion
lubricates and neutralises the acidity of vagina.
SH

‰ Mammary glands are paired structures containing glandular tissues and variable amount of fats. The glandular tissue of
each breast is divided into 15-20 mammary lobes containing clusters of cells called alveoli. The cells of alveoli secrete
A

milk, which is stored in the cavities (lumen) of alveoli.


PR

The Menstrual Cycle


Menstrual cycle is also called as the ovarian cycle or uterine cycle or the female reproductive cycle. It is a cyclic repetition
R.

of events that occur in females in their reproductive years. The ovaries produce one ovum every month on the 14th day
D

and subsequently prepares for pregnancy. If it does not occur, endometrial discard or menstruation occurs during the
first 5 days of the cycle.
The first menstruation begins at puberty and is called menarche. Stoppage of menstruation permanently at the age
of 45-50 years is called menopause which is explained later in the chapter. The cycle also stops temporarily during
pregnancy.

Fertilization and Embryogenesis


It is the fusion of the nucleus of a sperm, of an egg cell to form a diploid zygote. The diploid zygote contains 46
chromosomes in the nucleus, i.e., 23 from the sperm cell and 23 from the egg cell.
Fertilization should occur within 12-24 hours after ovulation.
Once the egg cell has accepted one sperm cell, it immediately develops a membrane around itself, to prevent other sperm
cells from entering it. Once fertilization takes place, a diploid zygote is formed.

How Do Organisms Reproduce? 131


From fertilization to birth
Zygote undergoes several mitotic divisions or cleavages and gives rise to
a multicellular stage known as embryo. The embryo is implanted inside
the endometrium lining of the uterus.
After implantation, a structural and functional unit is formed between the
developing embryo (foetus) and maternal body called placenta.

Gestation (Pregnancy)
The embryo gets nutrition from the mother’s blood with the help of a
special tissue called placenta. This is a disc which is embedded in the
uterine wall. It contains villi on the embryo’s side of the tissue. On the
mother’s side are blood spaces, which surround the villi. This provides
a large surface area for glucose and oxygen to pass from the mother to
Fig. 18: Location of Placenta in uterus
the embryo. The developing embryo will also generate waste substances

R
which can be removed by transferring them into the mother’s blood
through the placenta. The development of the child inside the mother’s body takes approximately nine months. This is

A
called the gestation period.

M
U
Knowledge Hub
K
‰ Placenta also acts as an endocrine tissue and produces several hormones like human chorionic gonadotropin
T
(hCG), human placental lactogen (hPL), estrogens, progestogens, etc.
N
‰ When a female takes a pregnancy test hCG is detected in the urine.
‰ Progesterone is also known as the pregnancy hormone as its maintains the endometrium during gestation.
A
SH

The child is born as a result of rhythmic contractions of the muscles in the uterus and the process in known as child
birth/ parturition.
A

The mammary glands of the female undergo differentiation during pregnancy and starts producing milk towards the end
of pregnancy by the process called lactation. This helps the mother in feeding the new- born.
PR

Reproductive Health
R.

According to the World Health Organisation (WHO), reproductive health means a total well-being in all aspects of
D

reproduction, i.e., physical, emotional, behavioural and social.


‰ Sexual act is an intimate and personal choice, however, it is influenced by society and the environment. Pressure from
family, and peers to produce a child, apparent or explicit, affects this choice.
‰ Because of the highly intimate nature, two people engaging in sex must know about possible risks of unhealthy choices
during and about sex.
‰ Government policies affect our decision too. The size of population is determined by the rates of birth and death in a
given population. The size of the human population is a cause for concern for many people. This is because an expanding
population lead to a scarcity of basic requirements, i.e., food, shelter and clothing. Inequality is the main reason for poor
standard of living but an extra large population could play a relatively lesser role.
‰ Prejudices related to gender could fuel illegal killing of infants (infanticide used to be witnessed in India against girl
child) and disrupt gender ratio.
Reproductive health aims to educate individuals about healthy ways to address this very natural course.

132 Class-X BIOLOGY PW


Knowledge Hub
Female individual after giving birth to child, feed the child with her own milk. The milk produced during the initial
few days of lactation is called Colostrum. Colostrum is thin yellow coloured milk also called as fore milk. Milk is
rich in protein (lactoprotein and lactalbumin) and low in fat content. It also contains antibodies like IgA and IgG
that provide first immunity to the child

Try it Yourself
1. When does ovulation takes place?
2. Which hormone is secreted by corpus luteum?

Sexually Transmitted Diseases (STDs)

R
‰ Many infectious diseases which are spread by sexual contact are called Sexually Transmitted Diseases (STDs),

A
e.g. AIDS, Hepatitis, genital warts, gonorrhoea, syphilis, etc.

M
‰ STDs occur mostly in the individuals who are involved in sexual activities with many partners.

U
AIDS
K
T
The word AIDS stands for Acquired Immuno Deficiency Syndrome. This means deficiency of the immune system,
N
acquired during the lifetime of an individual indicating that it is not a congenital disease. ‘Syndrome’ means a group of
A

symptoms. AIDS was first reported in 1981 and in the last twenty-five years or so, it has spread all over the world killing
more than 25 million persons. AIDS is caused by the Human Immunodeficiency Virus (HIV), a member of a group of
SH

viruses called retrovirus, which have an envelope enclosing the RNA genome.
A

Transmission of HIV-infection generally occurs by


PR

‰ Sexual contact with an infected person


‰ By transfusion of contaminated blood and blood products.
R.

‰ Sharing infected needles as in the case of intravenous drug abusers.


‰ From an infected mother to her child through the placenta.
D

So, people who are at high risk of getting this infection includes - individuals who have multiple sexual partners, drug
addicts who take drugs intravenously, individuals who require repeated blood transfusions and children born to an HIV
infected mother.

Preventive measures
One could be free of these infections by following the simple principles given below:
‰ Avoid sex with unknown partners/multiple partners.
‰ Always use condoms during coitus.
‰ In case of doubt, one should go to a qualified doctor for early detection and get complete treatment if diagnosed with
disease.

How Do Organisms Reproduce? 133


Knowledge Hub
It is important to note that HIV/AIDS is not spread by mere touch or physical contact; it spreads only through body
fluids. It is, hence, imperative, for the physical and psychological well-being, that the HIV/AIDS infected persons
are not isolated from family and society. There is always a time-lag between the infection and appearance of AIDS
symptoms. This period may vary from a few months to many years (usually 5-10 years).

Methods of Prevention of STDs


(i) The people should be educated about various STDs.
(ii) Avoid sex with unknown or multiple partners.
(iii) Always use a condom during coitus.

Methods of Contraception

R
A
Childbearing is a task that requires preparedness and responsibility. Individuals must be able to have sex without

M
necessarily bearing child. Contraceptive devices are tools that can help avoid unwanted pregnancy. People should be
educated about sexual conduct and healthy ways to evaluate their choices about sex, family planning and population

U
in family and society. Various contraceptive options are available such as barrier method, chemical method, IUDs &
surgical methods.
K
1. Behavioural method: Couples may choose to avoid sex from 10th to 17th day of the menstrual cycle as this is the period
T
when there is a chance that the ovum will be available for fertilization.
N

2. Barrier methods:
A
SH

(i) These are the physical devices that prevent the entry of sperm by
forming a mechanical barrier so that it does not reach the egg.
A

(ii) It includes use of condoms. These are made up of thin rubber or latex
Male condoms Female condoms
sheath which covers the penis before coital activity.
PR

Fig. 19: Male and Female condoms


(iii) Diaphragms and cervical caps are also used. They are also made up
of rubber and are inserted in vagina of female.
R.

3. Chemical methods:
D

(i) It includes the use of some chemicals which are spermicidal. They may be in form of tablets, jellies, paste
and creams introduced in the vagina before coital activity.
(ii) Another chemical method is the use of oral contraceptive pill which prevents ovulation from the ovary by
inhibiting the secretion of FSH and LH hormone. FSH and LH hormones ares responsible for maturation of
ovum and release respectively.
4. Surgical methods:
(i) Tubectomy is a surgical method which involves cutting of fallopian tubes in females and vasectomy involves
cutting of vas deferens of each side in males.
(ii) Surgical removal of testes is known as castration and surgical removal of ovaries is known as ovariectomy.
(iii) Although quite effective, the reversibility of these methods is less successful.

134 Class-X BIOLOGY PW


Ureter
Bladder Seminal vesicles

Vasa deferentia

Vasectomy

Urethra
Prostate

Bulbourethral gland

Epididymis

R
A
Testes

M
Fig. 20: Vasectomy

U
Fallopian Fundus of Cauterized
tube Uterine
cavity
K
uterus
T
N
A
SH

Ligament Fimbriae
Ovary of ovary
A
PR

Tied and cut Cauterized Banded


R.
D

Fig. 21: Tubectomy


5. IUDs
(i) Intra Uterine Devices (IUDs) are fitted in the uterus. They help to prevent fertilization.
(ii) IUDs disrupts the motility of sperms which prevents them from fertilizing the egg.
(iii) They can cause side effects due to irritation of uterus. Examples: Copper T and loops.
(iv) CU-T is the most widely used method in India.

Medical Termination Pregnancy (MTP) or Induced Abortion:


It is the process of terminating pregnancy by medication or surgery. It is performed by a professional gynaecologist as the
process comes under minor surgery. If the pregnancy is in early stages (7 to 8 weeks) it can be terminated by medicines

How Do Organisms Reproduce? 135


Why do women choose MTP?
‰ When women conceive accidentally
‰ In case of rape.
‰ If the situation is life threatening
When do women not choose MTP?
‰ When the gestation period extends to 9 weeks
‰ If intrauterine device used
‰ If any blood pressure complications
‰ If mother is suffering from ovarian enlargement

Summary

R
™ Reproduction is a biological process by which parents can produce their offspring and transfer their characteristic

A
features to offspring in the form of genes.

M
™ Asexual reproduction is the type of reproduction in which only one parent is engaged to form the exact copy of
themselves. Examples include – fission, regeneration, fragmentation, spore formation.

U
™ Plant propagate by a method in which new plants are formed from the vegetative parts such as leaves, stem.

K
™ Flowers are the main reproductive part of plants, have four whorls, sepals, petals, stamen and pistil from outer to
inner whorl.
T
™ Transfer of pollen grain from the anther to the stigma of female part of the flower is called pollination.
N
™ Fertilization is a process in which nucleus (male nucleus) from pollen grain fuses with the egg cell in ovule to
A

make the zygote.


™ One of the male gamete fuses with the egg cell, this is called syngamy.
SH

™ Second male gamete fuses with the two polar nuclei, this is called triple fusion. So syngamy and triple fusion is
unitedly called as double fertilization.
A

™ Male reproductive system constitutes organs that unitedly work as a system that produces male gamete. They not only
PR

produce male gamete but also discharge the male gamete in female reproductive tract and produces male sex hormones.
™ Testis is the main part of the male reproductive system because it is the site for sperm formation. It is located
outside the body in a pouch-like structure called scrotum.
R.

™ Female reproductive organ consists of a pair of ovaries, oviduct (a pair of fallopian tube), uterus, vagina, external
genitilia and breasts.
D

™ Fertilisation happens in the oviduct.


™ Implantation is the attachment of blastocyst to the uterine wall.
™ Menstrual cycle is a repetitive series of hormonal changes in women's body every month. In every cycle body
undergoes changes for the pregnancy.
™ At the age of 13 females get first period that process is called menarche and at the age of 50 menstruation stops-
menopause.
™ Parturition is the birth of a full grown young one from the mother’s uterus at the end of gestation period.
™ Birth control is the regulation of conception by preventive method or devices used to limit the number of children.
™ A vasectomy is a surgical procedure in which vas deferens is cut and tied.
™ Tubectomy is a major surgical procedure in which the fallopian tubes are cut open and clipped or tied up to block
the passage of the egg into the uterus.

136 Class-X BIOLOGY PW


NCERT Corner
1. What is the importance of DNA copying in various groups of living organisms, including
reproduction? fungi, plants, and some bacteria. Spores can
Ans. DNA is the genetic material present in the nucleus withstand harsh environmental conditions, such
cells of all organisms. DNA carries the genetic as extreme temperatures, drought. This resilience
information from generation to generation. It is
allows them to endure conditions that might be
therefore possible for the organism to produce
organism of its own type due to DNA copying harmful to the parent organism. Spores can remain
only. For the inheritance of traits of the parent, dormant until more favorable conditions arise for
DNA copying is a must. DNA copying also brings growth and reproduction.
about variation, which forms the basis for the
5. Can you think of reasons why more complex
origin of new species, i.e., speciation.
organisms cannot give rise to new individuals
2. Why is variation beneficial to the species but

R
not necessarily for the individual? through regeneration?

A
Ans. Sometimes for a species, the environmental Ans. More complex organisms cannot give rise to new

M
conditions change so drastically that their survival individuals through regeneration because complex
becomes difficult. For example, if the temperature

U
organisms have organ-system level of organization.
of water increases suddenly, then most of the
bacteria living in that water would die. Only few
variants that are resistant to heat would be able to
K
Specialised cells are organised as tissues, and
tissues are organised into organs, which then have
T
survive. However, if these variants were not there, to be placed at definite positions in the body.
N

then the entire species of bacteria would have been


6. Why is vegetative propagation practised for
A

destroyed. Thus, the variations help in the survival


growing some types of plants?
SH

and are beneficial of the species. However, all


variations are not necessarily beneficial for the Ans. Vegetative propagation is practiced for growing
individual organisms. some types of plants because of following
A

3. How does binary fission differ from multiple advantages:


PR

fission?
™ It is used for the development of a plant in
Ans. In binary fission a single cell divides into two which viable seeds are not formed or very few
R.

equal halves, e.g., Amoeba and bacteria while in


seeds are produced.
multiple fission, a single cell divides into many
D

daughter cells simultaneously, e.g., Plasmodium. ™ It helps to produce plants in new areas where

4. How will an organism be benefited if it the germination of seed is failed to produce


reproduces through spores? mature plant due to change in environmental
Ans. Reproduction via spores offers several advantages conditions and the soil.
to organisms, and this method is observed in ™ It is more rapid, easier and cheaper method.

How Do Organisms Reproduce? 137


NCERT Exercise

1. Asexual reproduction takes place through budding in fertilized egg. Thus, the inner lining (endometrium)
(a) Amoeba (b) Yeast of the uterus gets thickened and is supplied with
(c) Plasmodium (d) Leishmania blood to nourish the embryo. If the egg does not get
fertilised, then the lining of the uterus slowly breaks
Ans. (b) Asexual reproduction takes place through and comes out through vagina as blood and mucous.
budding in Yeast.
7. Draw a labelled diagram of the longitudinal section
2. Which of the following is not a part of the female of a flower.
reproductive system in human beings?
Ans. The labelled diagram of the longitudinal section of
(a) Ovary
a flower is given below:
(b) Uterus
(c) Vas deferens
(d) Fallopian tube

R
Ans. (c) Vas deferens is not a part of the female

A
reproductive system in human beings.

M
3. The anther contains
(a) Sepals (b) Ovules

U
(c) Carpel (d) Pollen grains Receptacle
Radicle
Ans. (d) The anther contains pollen grains.
4. What are the advantages of sexual reproduction K
8. What are the different methods of contraception?
T
over asexual reproduction? Ans. The contraceptive methods can be broadly divided
N

Ans. Advantages of sexual reproduction: into the following types:


A

™ In sexual reproduction, more variations are ™ Natural method: It involves preventing the
produced. Therefore, it ensures survival of chances of meeting of sperms and ovum. In this
SH

species in a population. method, the sexual act is avoided from day 10th
™ The newly formed individual has characters to 17th of the menstrual cycle because during
A

of both the parents. This is because in sexual this period, ovulation is expected and therefore,
reproduction, genetic material from both the chances of fertilization are very high.
PR

the parents mixed (recombination of parent ™ Barrier method: Barriers are available for
chromosomes) by fertilization. both males and females. Condoms are barriers
made up of thin rubber which covers the penis
R.

5. What are the functions performed by the testis in


human beings? in males and vagina in females.
D

™ Oral contraceptives: In this method, pills or


Ans. Functions of testes:
drugs are taken orally. They change hormonal
™ Regulate the development and maturation
balances that prevent the release of eggs and
of sperms, which contain haploid set of
thus fertilization cannot occur.
chromosomes of father.
™ Surgical methods: Some surgical methods,
™ Produce a hormone called testosterone, which
such as tubectomy and vasectomy, can also be
regulates sexual development.
used to block the gamete transfer. Vasectomy
6. Why does menstruation occur? involves the blocking of vas deferens to prevent
Ans. Menstruation is a process in which blood and the transfer of sperms. Similarly, fallopian tubes
mucous flows out through the vagina. This process of the female can be blocked by tubectomy so
is repeated every month at an average interval that the egg will not reach the uterus.
of about 28/29 days because one egg is released ™ IUCDs: Some devices called as Intrauterine
from the ovary every month and at the same time, contraceptive devices are placed in the uterus of
the uterus (womb) prepares itself to receive the females. E.g., Copper T and Lippes loop.

138 Class-X BIOLOGY PW


9. How are the modes for reproduction different in stability to population of species by giving birth to
unicellular and multicellular organisms? new individuals.
Ans. For unicellular organisms, reproduction occurs by It also ensures the continuity of a species. By
the division of the entire cell. Fission, budding, reproduction, organisms form a large number of
regeneration are some modes of reproduction in new individuals out of which several lose their life
unicellular organisms. In multicellular organisms, and only some survive. These surviving organisms
specialised reproductive organs are present. replace the naturally dying members of the
Therefore, they can reproduce by vegetative population. Hence the population as a whole is not
propagation, spore formation, etc. In more complex affected and remains stable.
multicellular organisms the mode of reproduction 11. What could be the reasons for adopting
is sexual reproduction. It is found in humans and contraceptive methods?
plants. Ans. Contraceptive methods are mainly adopted because
10. How does reproduction help in providing stability of the following reasons:
to populations of species? ™ To prevent unwanted pregnancies.
Ans. Reproduction is a natural process of producing ™ To control population growth or birth rate.

R
new individuals of the same species from existing ™ To provide protection against various sexually

A
organisms of a species, thus, it helps in providing transmitted diseases.

M
U
Quick Recall K
T
N
A

Fill in the Blanks True and False Statements


SH

1. In Spirogyra, asexual reproduction takes place through


1. In organisms such as _____, the splitting of the two
fragmentation.
cells during division can take place in any plane.
A

2. The DNA copying mechanisms generate variations


2. ______ is common method of multiplication of Yeast
PR

which are always useful for maintaining the survival


and Hydra. of the species.
3. ______ bearing anthers which produce pollen are the 3. Vegetative propagation produces plants that are
R.

male reproductive parts of a flower. genetically identical to the parent plant.


4. The female gamete in angiosperms is inside ______ 4. All plants bear flower as tool for reproduction.
D

of ovule. 5. Unisexual flowers generate more variation than


5. In cross-pollination, Pollen grains are transferred from bisexual flowers.
______ of stamen to ______ of carpel (same/different) 6. The ovary of a flower grows into a fruit.
flower. 7. Menopause is the onset of menstruation
6. ______ matures to produce shoot. 8. The maternal blood supply mixes frequently with
7. ______ is the male gonad. the foetal blood supply during the exchange of waste
materials and nutrients.
8. ______ collects the female gamete from the ovary
9. In mammals including humans, fertilization takes place
9. ______ is known as the birth canal externally.
10. Surgically when vasa deferens tube is cut and tied, it 10. Sexually transmitted diseases can be prevented by
is called ______. using condoms.

How Do Organisms Reproduce? 139


4. Match the given columns:
Match the Following
Column-I Column-II
1. Match the organisms given in column-I with their (P) Germination (i) The pollen
respective reproduction methods in column-II. transferred from one
flower to another
Column-I Column-II (Q) Pollination (ii) The process in
(P) Planaria (i) Binary fission which embryo
(Q) Fungi (ii) Asexual spores develops into
seedling
(R) Yeast (iii) Budding
(R) Menstruation (iii) Fertilised egg
(S) Amoeba (iv) Regeneration in humans gets
(a) P-(i) Q-(ii) R-(iii) S-(iv) implanted in
(b) P-(iv) Q-(ii) R-(iii) S-(i) (S) Uterus (iv) Process occurs when
egg in humans is
(c) P-(ii) Q-(iii) R-(i) S-(iv) not fertilised

R
(d) P-(iv) Q-(i) R-(iii) S-(ii) (a) P-(ii) Q-(i) R-(iv) S-(iii)

A
2. Match the column-I with column-II. (b) P-(i) Q-(ii) R-(iv) S-(iii)

M
(c) P-(i) Q-(ii) R-(iii) S-(iv)
Column-I Column-II

U
(d) P-(i) Q-(iii) R-(ii) S-(iv)
(P) Stamens (i) Calyx
(Q) Pistil (ii) Corolla K
5. Match the column-I with column-II:
T
(R) Sepals (iii) Androecium Column-I Column-II
N
(P) External (i) Fertilization
(S) Petals (iv) Gynoecium
fertilization
A

(a) P-(iii) Q-(iv) R-(i) S-(ii) (Q) Female gamete (ii) Ovary
SH

(b) P-(i) Q-(ii) R-(iii) S-(iv) (R) Fusion of male and (iii) Frog
female gametes
(c) P-(ii) Q-(i) R-(iii) S-(iv)
A

(S) Main female (iv) Testis


(d) P-(iii) Q-(ii) R-(i) S-(iv) reproductive organ
PR

3. Match the column-I with column-II. (T) Main male (v) Ovum
reproductive organ
Column-I Column-II
R.

(a) P-(i) Q-(ii) R-(iii) S-(iv) T-(v)


(P) Bisexual flowers (i) Bryophyllum
(b) P-(iii) Q-(v) R-(i) S-(ii) T-(iv)
D

(Q) Unisexual flowers (ii) Mango


(R) Plants that reproduce (iii) Maple (c) P-(ii) Q-(iii) R-(iv) S-(i) T-(v)
by leaves (d) P-(ii) Q-(iv) R-(i) S-(iii) T-(v)
(S) Winged seeds dispersed (iv) Papaya
by wind Multiple Choice Questions
(T) Seeds & fruits (v) Mustard
dispersed by animals 1. Which of the following is not an outcome of variations
present in population?
(a) P-(iii) Q-(iv) R-(i) S-(ii) T-(v)
(a) Bacterial resistance to heat.
(b) P-(v) Q-(iv) R-(i) S-(iii) T-(ii) (b) Different colour of eyes.
(c) P-(ii) Q-(i) R-(iii) S-(v) T-(iv) (c) Survival of species over time.
(d) P-(iii) Q-(v) R-(i) S-(iv) T-(ii) (d) Maintenance of body design features.

140 Class-X BIOLOGY PW


2. If a starfish is cut into pieces, each piece grow into a 12. Consider the following statements:
complete animals. The process is called (i) Copper-T is a contraceptive device used by women.
(a) Regeneration (ii) Sexually transmitted disease can be prevented by
(b) Reproduction using condoms.
(c) Healing of wounds (iii) The ovulation takes place 10-12 days after the start
(d) Fragmentation of mensuration.
3. Spiny or sticky pollen grains and large, attractively (iv) In human-beings, male can produce sperms upto
coloured flowers are associated with the age of 45-50 years.
(a) Hydrophily Which of these statement(s) is/are correct?
(b) Entomophily (a) (i) and (ii) (b) (ii) and (iii)
(c) Ornithophily (c) (i) only (d) (i), (ii) and (iii)
(d) Anemophily
4. From among the sets of terms given below, identify Assertion & Reason Type Questions
those that are associated with the gynoecium.
Read the following Assertion and Reason and select the

R
(a) Ovule, ovary, embryo sac, tapetum
correct option.

A
(b) Thalamus, pistil, style, ovule
(a) Both Assertion (A) and Reason (R) are true and
(c) Stigma, ovule, embryo sac, placenta

M
Reason (R) is the correct explanation of Assertion (A).
(d) Ovule, stamen, ovary, embryo sac

U
5. The development of a seedling from an embryo under (b) Both Assertion (A) and Reason (R) are true and
Reason (R) is not the correct explanation of Assertion (A).
appropriate condition is called
(a) Regeneration (b) Germination K
(c) Assertion (A) is true but Reason (R) is false.
T
(c) Spore formation (d) Budding (d) Assertion (A) is false but Reason (R) is true.
N

6. Ovulation is controlled primarily and directly by 1. Assertion (A): Amoeba shows multiple fission during
A

(a) LH (b) FSH unfavourable conditions.


SH

(c) Estrogen (d) Progesterone


Reason (R): Chances of survival are less during
7. Which of the following organs of the female reproductive unfavourable conditions.
system sheds its lining during menstruation?
A

2. Assertion (A): Sexual reproduction increases


(a) Ovary (b) Uterus
PR

genetic diversities and plays a role in origin of


(c) Vagina (d) Fallopian tubes
new species.
8. After fertilization the fertilized egg or the zygote gets
Reason (R): Sexual reproduction involves formation
R.

implanted in the
of gametes and fusion of gametes.
(a) Vagina (b) Ovary
D

(c) Uterus (d) Fallopian tube 3. Assertion (A): The offsprings produced as a result of
asexual reproduction are called clones.
9. The gestation period in human beings is about:
(a) 250 days (b) 300 days Reason (R): The DNA of clones is exactly similar to
the parent.
(c) 280 days (d) 200 days
4. Assertion (A): A flower is a modified shoot.
10. Which of the following method of contraception
protects from acquiring sexually transmitted diseases? Reason (R): All the floral parts are borne on the
(a) Surgery (b) Oral-pills thalamus.
(c) Condoms (d) Copper-T 5. Assertion (A): Generally, mother is blamed for the
11. Tubectomy is carried out by blocking the birth of girl child in the family.
(a) Oviduct (b) Uterus Reason (R): Father is responsible for the sex of the
(c) Vas deferens (d) Vagina child.

How Do Organisms Reproduce? 141


3. Statement-I: Angiosperms are the only plants with
Statement Type Questions flowers.
Read the following statements and select the correct
options. Statement-II: Flowers increase the efficiency of

(a) Both Statement-I and Statement-II are correct. reproduction for plants
(b) Both Statement-I and Statement-II are incorrect. 4. Statement-I: The male hormone called testosterone
(c) Statement-I is correct & Statement-II is incorrect. is produced by testes significantly after puberty or by
mature males.
(d) Statement-I is incorrect & Statement-II is correct.
1. Statement-I: Asexual reproduction pre-dates sexual Statement-II: Testosterone production is stimulated

reproduction. by FSH.
Statement-II: Asexual reproduction is simpler 5. Statement-I: Reproductive health should be taught to
2. Statement-I: Asexual reproduction is exposed to no young students
variations at all.
Statement-II: STDs cannot be prevented with the help

Statement-II: Asexual reproduction is a slower force
for evolution than sexual reproduction of contraceptives.

R
A
M
U
Very Short Answer Type Questions K
8. Why cannot fertilisation take place in flowers if
pollination does not occur?  (2020)
T
1. Organisms have a varied body design. Name the
N
9. In tobacco plant, the male gametes have twenty four
property which gives the basic difference in body
A

design. chromosomes.
SH

2. What methods will you use for growing jasmine and What is the number of chromosomes in the female
rose plants?
gamete? What is the number of chromosomes in the
A

3. Name the method by which Spirogyra reproduce


zygote?
PR

under favourable conditions. Is this method sexual or


asexual? 10. Give two reasons for the appearance of variations
4. How will an organism be benefitted if it reproduces among the progeny formed by sexual reproduction.
R.

through spores?  (2018-19)


D

5. Would a Planaria cut vertically into two halves


11. Fertilisation is the process of
regenerate into two individuals? Complete D and E
by indicating the regenerated regions. (a) transfer of male gamete to female gamete.
(b) fusion of nuclei of male and female gamete.
(c) adhesion of male and female reproductive organs.
(d) the formation of gametes by a reproductive organ.
 (2020)
A B C D E
12. How does the embryo gets nourishment from the
6. Differentiate between pollen grain and ovule.
mother?
7. Distinguish between unisexual and bisexual flowers
giving one example of each. 13. How do oral contraceptive help in avoiding pregnancies?

142 Class-X BIOLOGY PW


2. Why are budding, fragmentation and regeneration all
Short Answer Type Questions considered as asexual types of reproduction? With
1. Name the type of reproduction in which two individuals neat diagrams explain the process of regeneration in
are formed from a single parent and the identity of the Planaria. (Exemplar) (2019)
parent is lost. Write in brief about the first and the final 3. Write two points of difference between asexual and
stage of the process of this reproduction. (2018) sexual types of reproduction. Describe why variations
2. Name the process by which an Amoeba reproduces. are observed in the offspring formed by sexual
Draw the various stages of its reproduction in a proper reproduction.  (2018-19)
sequence.  (2018) 4. (a) Draw a diagram showing germination of pollen on
3. A student is viewing under a microscope a permanent stigma of a flower and mark on it the following
slide showing various stages of asexual reproduction by organs/parts:
budding in Yeast. Draw diagrams of what he observes. (i) Pollen Grain
(in proper sequence)  (2019) (ii) Pollen tube
4. What is carpel? Write the function of its various parts. (iii) Stigma

R
 (2019) (iv) Female germ cell

A
5. Distinguish between cross-pollination and self- (b) State the significance of pollen tube.

M
pollination. Mention the site and product of (c) Name the parts of flower that develop after
fertilization in a flower. (2018, 2019) fertilization into

U
6. (a) Mention the changes which the uterus undergoes, (i) Seed
when
(i) it has to receive a zygote. K
(ii) Fruit (2020)
5. Distinguish between pollination and fertilisation.
T
(ii) no fertilization takes place. Mention the site and product of fertilisation in a flower.
N

(b) State the function of placenta. (2020) Draw a neat, labelled diagram of a pistil showing pollen
A

tube growth and its entry into the ovule.  (2019)


7. (a) What are Sexually Transmitted Diseases (STD) ?
SH

List two viral and two bacterial STDs. (2020) 6. (a) What provides nutrition to human sperms ? State
the genetic constitution of a sperm.
(b) What is contraception? (2018, 2019)
A

(b) Mention the chromosome pair present in zygote


8. (a) ‘‘Use of a condom is beneficial for both the sexes
PR

which determines the sex of (i) a female child, and


involved in a sexual act.’’ Justify this statement
(ii) a male child (2020)
giving two reasons.
7. Based on the given diagram answer the questions given
R.

(b) What is sex selective abortion? How does it affect


a healthy society? (State any one consequence) below:
D

 (2020) B

A
Long Answer Type Questions Bladder

1. (a) Define vegetative propagation. List its two methods.


(2019)
(b) Why is this mode practised for growing some types
of plants? (2020) del. D
(c) Explain the process of budding in Hydra with the C
help of labelled diagrams.
(d) We can develop new plants from the leaves of
Bryophyllum Comment. (2020) Testis

How Do Organisms Reproduce? 143


(a) Label the parts A, B, C and D. (i) Arrange the letters given below each diagram in a
(b) Name the hormone secreted by testis and mention logical sequence to show the correct order in the
its role. process of fertilisation.
(c) State the functions of B and C in the process of (ii) Where does fertilisation normally take place?
reproduction. (2020) What is "Implantation that follows fertilisations?
(iii) Mention the chromosome number of the egg and
8. (a) Write the function of following parts in human
zygote in humans.
female reproductive system:
(iv) Explain the term 'Gestation'. How long does
(i) Ovary
Gestation last in humans?
(ii) Oviduct (v) Draw a neat, labelled diagram of a mature human
(iii) Uterus  (2018, 2020) sperm. (2018)
(b) Describe in brief the structure and function of 11. (a) Draw a diagram of human female reproductive
placenta. (2018) system and label the parts
9. The diagram given below represents a system in the (i) which produce an egg.

R
human body. (ii) where fertilization takes place.
(b) Why has Government of India prohibited prenatal

A
Study the diagram and answer the following
sex determination by law? State its benefits in the

M
questions:
long run.
2

U
Case-Based Type Questions
K
T
5
Case Study-I
N
1 Preeti collected some pond water in a test tube which
3
A

6 was dark green in colour. She took out dark green


4 colouredmass from it and separated its filaments by
SH

using needles. Some filaments are broken down into


(a) Identify the system. small fragments. She put these small filaments in a Petri
A

dish containing clean water. After few days, she observed


(b) Label the parts marked 5 and 6.
PR

that the small fragments produce complete filaments.


(c) Name the two hormones secreted by 1.
1. The mass of green filament was of
(d) Mention the number and the name of the part
(a) Spirogyra
R.

involved in fertilization and implantation from the


above diagram. (b) Volvox
D

(c) Fucus
(e) Mention the surgical methods of contraception in:
(d) Fungal filaments or hyphae
1. Human males.
2. Human females. (2019) 2. The small fragment gave rise to new filament. What
does it indicate?
10. (a) Given below are diagrams showing the different
(a) Spirogyra reproduces asexually through
stages in the process of fertilisation of an egg in the regeneration.
human female reproductive tract.
(b) Spirogyra reproduces asexually through
Study the diagrams and answer the questions: fragmentation.
(c) Spirogyra reproduces asexually through binary
fission.
(d) Spirogyra reproduces asexually through vegetative
A B C D propagation.

144 Class-X BIOLOGY PW


3. Can you identify an organism which reproduces in 4. Select the correct statement.
similar way as Spirogyra? (a) Only multicellularo r g a n i s m s c a n u n d e r g o
(a) Yeast fragmentation.
(b) Both unicellular and multicellular organisms can
(b) Cyanobacteria
undergo fragmentation
(c) Entamoeba (c) Fragmentation is sexual mode of reproduction.
(d) Volvox (d) Fragmentation is seen only in fungi.

Case Study-II
Species Nectar Nectar Concentration Seeds per Visits per Rooted branches
volume (% weight of sucrose flower Flower per gram shoot
(μL) Total weight) weight
M. rupestris (A) 4.93 13.6 2.2 0.22 0.673
M. eastwoodiae (B) 4.94 19.8 25 0.74 0.488

R
M. nepalensis (C) 20.25 17.1 102.5 1.08 0.139

A
M. verbenaceus (D) 38.96 16.9 155.1 1.26 0.091

M
M. cardinalis (E) 50.00 19.9 283.7 1.75 0.069

U
The data for 5 different flowering species visited by Case Study-III
hummingbirds is given. Look at the data above and K
The reproductive organs in human beings become functional
T
evaluate on the following
after attaining sexual maturity. This age is called puberty.
N
1. Which of the following species looks like they are Puberty in males is attained at an age of 13-14 years while
witnessing sexual reproduction significantly? in females, it is attained at an age of 11- 12 years. Changes
A

(a) A and B (b) C, D and E in the body at puberty, such as increase in breast size in
SH

(c) All of them (d) None of them girls and new facial hair growth in boys, are signs of sexual
2. If a disease that infects humming birds spreads which maturation. The primary sex organs – the testis in the
A

species will be least impacted? males and the ovaries in the females – produce gametes,
i.e, sperms and ovum, respectively, by the process called
PR

3. If a pathogen which infects Mimulus comes through


gametogenesis. The male sex accessory ducts include rete
which of the species have better chances of surviving?
testis, vasa efferentia, epididymis and vas deferens. Sertoli
4. The pathway to synthesise red coloured petals in a plant cells provide nutrition to the developing spermatozoa. The
R.

X consumes ATP. This red colour was important to its testes are situated outside the abdominal cavity within a
recognition by the pollinator. A Pathogen reduced the
D

pouch called scrotum. The scrotum helps in maintaining the


population of its pollinator to colour blind individuals. low temperature of the testes (2–2.5°C lower than the normal
The plant population is expected to evolve. Which of internal body temperature) necessary for spermatogenesis.
the following will be least expected change?
1. What is the puberty age in human females?
(a) red flowers remain as widespread as before.
(a) 8-10 (b) 11-12
(b) spread of plants with black and white flowers.
(c) 13-14 (d) 14-16
(c) increase in fragrance of flowers.
(d) to increase root/stem branching. 2. Why are testes located outside the abdominal cavity
in scrotum?
5. According to Royal Botanical Garden Kew, out of all
(a) Formation of sperms requires a higher temperature.
vascular plants 94% are flowering plants. Flowers often
have colours or fragrance which are due to compounds (b) Formation of sperms requires a lower temperature.
that consume ATP to be synthesized. What explanation (c) Formation of sperms requires more space
you might have for this observation? (d) Sperms get nutrition from scrotum

How Do Organisms Reproduce? 145


3. Testis in males and ovaries in the females produce 4. Which of the following is not an accessory duct of
respectively males?
(a) Ovum and sperms (a) Fallopian tube
(b) Ova and spermatozoa (b) Vasa efferentia
(c) Oogonia and spermatozoa (c) Epididymis
(d) Sperms and ovum (d) Vas deferens

Case Study-IV
Look at the diagram below to understand the set of events and the various hormonal levels in a woman during the
menstrual cycle. FSH and LH are hormones secreted from the Pituitary gland. These hormones control the production and
release of ova. Estrogen and Progesterone are hormones released from the ovary. Study the given graphical representation
of the menstrual cycle below and answer the following questions.

menstruation uterus lining thickens ready to receive breaks down if menstruation


embryo no implantation
takes place

R
A
M
corpus luteum developing corpus luteum
breaks down

U
follicle maturing ovulation

K
T
N
FSH and LH
pro
ns
A

ge ges
ro ter
est
SH

o
one
A

menstruation menstruation
PR

DAYS 1 2 3 4 5 6 7 8 9 10 11 12 13 14 15 16 17 18 19 20 21 22 23 24 25 26 27 28 1 2 3
start of end of copulation could
R.

menstruation menstruation result in fertilisation


D

1. A deficiency in which of the following hormone could (c) It is not controlled by hormones
cause menstruation? (d) Oestrogens
(a) FSH (b) Progesterone 4. Assume a similar role of hormones in Cows. To
(c) LH (d) Estrogen maximise dairy production a cow breeder wants
2. Which set of hormones initiate the thickening of uterus? the cow to give multiple ova. He intends to get this
(a) FSH and LH (b) Oestrogens fertilised in testubes using sperms of best bull. He then
wishes to transfer this ‘super cow” embryo to multiple
(c) LH only (d) FSH only
surrogates in order to get a herd of super cows. Which
3. Which hormones seem to initiate maturation and hormones will help him get the cows to ‘super ovulate’,
release of ova? release multiple ova.
(a) Progesterone (a) Progesterone (b) Oestrogen
(b) FSH and LH (c) FSH and LH (d) None

146 Class-X BIOLOGY PW


Competitive Level

How do Organisms Reproduce?


How are number of chromosomes maintained in sexually reproducing organisms?
Significance of Meisois
First
2n = 4 2n = 4 2n = 4 2n = 4
Generatioṇ

Meiosis Meiosis Meiosis Meiosis

n n n n
Gamete Gamete Gamete Gamete

R
Second

A
2n = 4 Generatioṇ 2n = 4

M
Meiosis Meiosis

U
n
Gamete K n
Gamete
T
N
2n = 4
A

Third Generatioṇ
SH

If NO Meiosis occurs
First
A

2n = 4 2n = 4 2n = 4 2n = 4
Generatioṇ
PR

No Meiosis No Meiosis
R.

2n 2n 2n 2n
Gamete Gamete Gamete Gamete
D

Second
4n = 8 Generatioṇ 4n = 8

No Meiosis No Meiosis

4n 4n
Gamete Gamete

8n = 16

Third Generation
DNA content will continue to increase by
doubling in each sexual fusion.

How Do Organisms Reproduce? 147


Amniocentesis
Amniocentesis is a foetal test used in prenatal diagnosis of chromosomal abnormalities, foetal infections and sex
determination. It is based on the chromosomal pattern in the amniotic fluid surrounding the developing embryo. In 1994,
amniocentesis had been banned in India, under the Preconception and Prenatal Diagnostic Techniques Act because this
method could reveal the sex of the foetus. Since a girl child is not accepted in many parts of India, the female foetus is
aborted in most of the cases causing Female Foeticide. To stop this, amniocentesis was banned in India.

Ultrasound
Amniotic fluid transducer
withdrawn
into syringe Placenta
Uterus Fetus

Amniotic fluid

R
A
Cervix

M
Fig. 22: Procedure of Amniocentesis

U
Tissue Culture K
T
Traditional breeding techniques struggled to meet the growing demand for crop improvement. To address this challenge,
N

scientists developed tissue culture as an alternative technology. The process of tissue culture is explained as follows:
A

‰ Tissue culture involves regenerating whole plants from explants—any part of a plant grown in a test tube under sterile
conditions and special nutrient media. This remarkable capacity, known as totipotency, allows the generation of an entire
SH

plant from any cell or explant.


‰ The tissue divides rapidly to form a small group of undifferentiated cells called callus. A small portion of this callus is
A

transferred to another medium containing hormones for growth and differentiation.


PR

‰ The process relies on nutrient media providing a carbon source (e.g., sucrose), inorganic salts, vitamins, amino acids,
and growth regulators like auxins and cytokinins.
‰ By application of these methods it is possible to achieve propagation of a large number of plants in very short durations.
R.

This method of producing thousands of plants through tissue culture is called micro-propagation.
D

‰ Each of these plants will be genetically identical to the original plant from which they were grown, i.e They are
somaclones.
‰ This technique is commonly used for ornamental plants Chrysanthemum, Asparagus and many other plants are
propagated by using this technique.

Parthenognesis and Parthenocarpy


Parthenognesis Parthenocrapy
It is the process of forming an animal without fertilization. It is the process of forming a fruit without fertilization.
E.g., Honey bee, Whiptail lizard. E.g., Banana.
In case of honey bee, the fertilized egg develop into female and unfertilized egg develops into male. Sperms are formed
by mitotic division and ova develop by meiotic division.

148 Class-X BIOLOGY PW


Queen

Fertilized egg Unfertilized egg


(Diploid) (Haploid)

Female offspring Male offspring

(Worker or Queen) (Drone)

R
A
M
Fig. 23: Parthenogenesis in Honey Bee

U
Artificial methods of Vegetative Propagation
K
Some plants can be propagated artificially. The methods of artificial propagation include grafting, layering, cutting and
T
tissue culture. Monocot plants lack cambium therefore grafting is not possible in monocot plants. Cambium activity is
N

essential for the union of stock and scion.


A

‰ Grafting: Shoot of a plant species and root system of another plant species are cut and joined. The joint is covered and
SH

waxed to allow the tissues to join. The technique of joining vegetative parts of two different plants and allow them to
grow as a single plant is called grafting. This characteristic is unique to plants. Grafting is generally performed between
related varieties and species. It is generally used for the propagation of plants like mango, guava, apple, pear, Citrus,
A

rubber and peaches.


PR
R.
D

Fig. 24: Grafting of two related varieties of plants


‰ Cutting: A cutting is a vegetative plant part which is severed from the parent plant in order to
regenerate itself, thereby forming a whole new plant. This method is commonly employed for
rose, lemon, blackberry, sugarcane, etc. Fig. 25: Cutting

How Do Organisms Reproduce? 149


‰ Layering: It is method of the development of roots on a stem while it is still attached to the parent plant. The stem that
develops adventitious roots while still attached to the parent plant is known as layer. Layering method is commonly
employed for jasmine, Magnolia, etc.

Fig. 26: Layering

R
A
Sexual Reproduction in Flowering Plants

M
U
Gamete Producing Organs in Angiosperms
Pollen grains
K
Plants show alternation of generation. This means there are two kinds of
T
individuals in plants life cycle- a sporophyte (which will produce spores)
N
and a gametophyte (which will produce gametes). The mango tree you see
is a sporophyte. Mango flowers are parts of sporophyte.
A

Anther
SH

Anther Stamen
‰ Anther contains spore producing organs- pollen sacs. Pollen sacs have
A

microsporocytes which are going to produce microspore (the male


spores) via meiosis
PR

Filament
‰ Pollen sacs are present at the four corners of a bilobed anther.
‰ Microspores build cell wall and are now called a pollen grain (male
R.

Fig. 27: Pollen Sacs inside


gametophyte). It divides via mitosis to produce a vegetative cell and
D

a generative cell.
‰ The vegetative cell forms the pollen tube and the generative cell further divides via mitosis to form 2 male gametes.
‰ Pollen grain has two layers, i.e., the outer exine and the inner intine.
‰ Exine is made up of sporopollenin (highly resistant fatty acid that are not degraded by enzyme) whereas intine is made
up of pectin and cellulose.
‰ Pollen grains are well preserved as fossils because of the presence of sporopollenin. As sporopollenin is one of the
most resistant organic materials known. It can withstand high temperatures and strong acids and alkali. No enzyme that
degrades sporopollenin is so far known.
‰ Pollen grain exine has prominent apertures called germ pores where sporopollenin is absent. The pollen tube germinates
from the germ pore.

150 Class-X BIOLOGY PW


Ovule: Chalazal end
‰ Ovule is protected by integuments. Antipodals
‰ Integuments open at one end called micropyle. The other end is called
chalaza.
‰ Ovules inside ovary produces megaspores (the female spores) via meiosis.
‰ One megaspore survives and three die.
‰ This undergoes 3 mitosis events to produce the embryo sac (female
gametophyte), a 7 celled- 8 nucleated structure. Polar nuclei
 3 Cells towards chalazal end are called as Antipodal cells which provide Central cell
Egg
nutrition (each with 1 haploid nuclei).
 The cell in the center is called central cell with 2 nuclei.
Synergids
 2 synergids towards micropyle guide the path of the pollen tube (each with
1 haploid nuclei). Filiform
apparatus
 1 female gamete/egg cell (haploid nuclei) towards the micropyle. Micropylar end

Double Fertilisation Fig. 28: Embryo sac inside ovule

R
DOUBLE

A
FERTILIZATION

M
U
Syngamy/ Triple
Fertilisation Fusion
K
T
One male gamete The 2nd male gamete fuses with the 2
N
fuses with the egg cell polar nuclei called as triple fusion which
and forms of diploid results in the formation of triploid forms
A

zygote called endosperm.


SH

Knowledge Hub
A
PR

Endosperm development precedes embryonic development as endosperm provide nourishment to the developing
embryo.
R.

Reproduction in Humans Leyding cell Capillary


D

Testis & Seminiferous Tubule


‰ Each Testis has around 250 compartments called testicular tubules and one to 3
Sertoli cell
highly coiled structure called seminiferous.
‰ Internally seminiferous tubule is lined by the germinal epithelium.
‰ Majority of cells are in cuboidal shape called spermatogenic cells and
some pyramidal shape cells called Sertoli cells or nurse cells are present in Spermatogonium
between. They are the special cells which are present in the seminiferous Fig. 29: Transverse large section of Testis
tubules of testes. They provide support and nutrition to the sperms.
‰ Spermatogenic cells produces sperm by the process of spermatogenesis and Sertoli cells provide nourishment to the
developing sperms.
‰ In between the seminiferous tubule, small group of cells called interstitial cells or Leydig’s cells are present. It produces
testosterone hormone.

How Do Organisms Reproduce? 151


Spermatogenesis
The process of synthesis of sperms is called spermatogenesis which starts at puberty.
2n

Spermatogonium
Mitosis

2n 2n

Spermatogonium Spermatogonium

2n

Primary spermatocyte

Meiosis I
2n 2n

R
Secondary Secondary

A
spermatocyte spermatocyte

M
Meiosis II

n n n n

U
Spermatids Spermatids
Spermatogenesis

n n n K n
T
N
4 spermatozoa
‰ It takes place in seminiferous tubules of the testis. Seminiferous tubule inner lining is made up of germinal
A

epithelium.
SH

‰ Germ cells in seminiferous tubule called spermatogonia undergo mitotic division to increase their number as shown in
the figure.
A

‰ In growth phase, spermatogonia stop dividing and grow in size (accumulates cytoplasm and replicate its DNA) and now
called Primary spermatocytes. Growth phase is also called spermatocytogenesis.
PR

‰ In maturation phase, primary spermatocyte undergoes reduction division (meiotic division) and form secondary
spermatocytes. Secondary spermatocytes undergo second meiotic division and produces spermatids.
R.

‰ The spermatids are transformed into spermatozoa. The transformation of spermatids into spermatozoa is known as
spermiogenesis.
D

Hormonal control of Spermatogenesis


HYPOTHALAMUS

GnRH (Gonadotropin releasing hormone)

Anterior lobe of pituitary gland

LH FSH
(Luteinising hormone) (Follicle Stimulating
hormone)
Leydig cells secretes
secretes Sperm maturation
Androgens factor
(Testosterone)

152 Class-X BIOLOGY PW


Ovary and Oogenesis
‰ The process of synthesis of ovum/female gamete is called as oogenesis which starts during the embryonic development.
‰ The bulk of ovary consists of follicles. Follicles are bags made from cells inside which oogonium develops into Primary
Oocyte.
‰ Primary Oocyte undergoes meiosis I to form 1 secondary oocyte (the larger cell which retains most of cytoplasm) and
1st polar body. At this stage we call the follicle as primary follicle.
‰ Secondary oocyte undergoes meiosis II to form ovum and secondary polar body. By this time the follicle has matured
into Graafian follicle which is filled with fluid.
‰ In every month the most mature Graafian follicle releases its ova and a yellow mass of cells is left behind called Corpus
luteum. Cells of Corpus luteum release a hormone progesterone during the pregnancy which maintains the endometrium
lining.
‰ Corpus luteum is maintained by hormones secreted from placenta if fertilisation occurs. If fertilisation doesn’t occur
corpus luteum degrades to a white body (degenerating Corpus luteum) also known as corpus albicans.
Early Corpus
Corpus Luteum

R
Luteum
Corpus Albicans

A
Ovum

M
U
Ovary
Ruptured Primary
Follicle
K Follicle
Growing Follicle
T
Mature
Follicle
N
Developing Follicle
A

Fig. 30: Inner structure of ovary


SH

Oogenesis
A

Oogonium
PR

2n
R.

Primary 2n
oocyte
D

Secondary Meiosis I
Primary
oocyte n Polar body
Meiosis II n

Secondary
Polar body

n n n n

Ovum (n) 3 polar bodies (degenerate)


Fig. 31: Stages of Oogenesis

How Do Organisms Reproduce? 153


Hormonal control of Oogenesis
HYPOTHALAMUS

GnRH (Gonadotropin releasing hormone)

Anterior lobe of pituitary gland

LH FSH
(Luteinising hormone) (Follicle Stimulating
hormone)
‰ Rupture mature graafian follicle
and causes ovulation. ‰ Stimulates development
‰ Stimulates ruptured graafian follicle of ovarian follicles.
to develop into corpus luteum.

Menstrual Cycle

R
It is a repetitive series of hormonal changes in women's body every month. In every cycle body undergoes changes for

A
the pregnancy. The menstrual cycle starts at puberty and this is termed as menarche and ceases at 45-50 years of age,
termed as menopause. The Menstrual cycle is divided on 4 phases which are described as follows:

M
Hormone levels

U
Pituitary

FSH
LH

K
T
N
Ovarian events

Developing Regressing
Developing Mature follicle
corpus luteum corpus luteum
A

follicle
SH

Ovulation
Hormone levels

A
Ovarian

PR

Estrogen
Progesterone
R.

Menses
Uterine events
D

Days
1 3 5 7 9 11 13 15 17 19 21 23 25 27 29/1
Menstruation Follicular phase Luteal phase Next cycle
(Proliferative phase) (secretory phase) begins
Fig. 32: Representation of the hormonal regulation in female menstrual cycle
1. Menstrual Phase: It is the starting phase of menstrual cycle. In which the uterine wall called endometrium is shed along
with blood due to decreased levels of progesterone.
It lasts for 3 to 5 days
Common Symptoms
(a) Stomach cramps (b) Headache
(c) Back pain (d) Lassitude
(e) Mood Swings

154 Class-X BIOLOGY PW


2. Follicular Phase: This phase lasts till 13th-14th day.
 Hypothalamus releases gonadotropins which send signals to the pituitary gland to release FSH and LH i.e., follicular
stimulating hormone, luteinizing hormone. These stimulate the ovary to form sac like structures called follicles.
 FSH also stimulate follicular cells to release estrogen.
 The estrogens in the ovary increases which results in thickening of uterine walls and blood vessels.
 By the end of this phase the graafian follicle is almost mature. Higher Estrogen level however now boost the secretion
of FSH and LH.
3. Ovulatory Phase: The LH is at peak during this phase known as LH surge which causes ovulation i.e. release of ovum
around 13th and 14th day of the Menstrual cycle.
4. Luteal Phase: LH increases, stimulating the remaining graafian follicle to convert into corpus luteum which releases
the progesterone that helps in maintaining the thickness of endometrium.
If secondary oocyte gets fertilized, then the placental hormone hCG maintains corpus luteum which continues to release
progesterone. If no fertilization occurs it will stop releasing the progesterone and as a result the uterine wall sheds on the

R
28th day and again the cycle restarts.

A
Development of the Embryo

M
Cleavage
‰ The male and female gamete fuse to form a dip-

U
Zygote 2- Celled 4- Celled 8- Celled
loid zygote. This process is called as Fertilisation Morula
and the site of fertilisation is ampulla of the fal-
lopian tube. Fusion of egg
K
T
and sperm
‰ Zygote is a single cell. It undergoes divisions pronuclei
N

called cleavage. It is division not followed by Blastocyst


A

increase in size of daughter cells. Zygote divides


SH

to form 2-cell, 4-cell stage and so on.


‰ Morula Stage: This stage consists of 16 to 32
A

cells. Fertilization
PR

Implanted
‰ Blastocyst: It is a hollow ball of cells filled with blastocyst
fluid
Ovum
R.

‰ Implantation: It is the attachment of blastocyst


Fig. 33: Stages of human development from fertilization to implantation
to the uterine wall.
D

Pregnancy or Gestation
1. In human beings, after one month of pregnancy, the embryo’s heart is formed.
First trimester 2. By the end of the second month of pregnancy, the foetus develops limbs and digits.
(12 weeks ) 3. By the third month of pregnancy, most of the major organ system are formed. Example., the
limbs and external genital organs are well developed.
1. The initial foetal movements and the onset of head hair usually occur in the fifth month of
Second trimester pregnancy.
(24 weeks) 2. By the end of this trimester, the body is covered with fine hair, eye-lids separate, and eyelashes
are formed.
1. By the end of nine months of pregnancy, the foetus is fully developed and is ready for
Third trimester
delivery.

How Do Organisms Reproduce? 155


Knowledge Hub
Relaxin hormone is secreted by the ovary during later stages of pregnancy in preparation for childbirth. It relaxes
the ligaments in the pelvis and softens and widens the cervix.

Parturition
Parturition is the birth of a full grown young one from the mother’s uterus at the end of gestation period.
‰ The fully developed foctus and placenta send signals to the maternal pituitary to initiate the foetal ejection reflex.
‰ The maternal pituitary in turn releases oxytocin.
‰ Oxytocin stimulate the uterus and initiates mild uterine contractions.
‰ Oxytocin also stimulates the release of prostaglandins from the placenta.
‰ The forceful muscular contractions of the uterine wall leading to parturition is called labour that causes labour pain.

R
A
Estradiol Oxytocin
From

M
ovaries From mother's
posterior pituitary

U
Activates oxytocin

Positive feedback
receptors on uterus

K
Stimulates uterus
to contract
T
Stimulates placenta
N
to make
A

Prostaglandins
SH

Stimulates more
contractions
of uterus
A

The milk produced during the initial few day of lactation is called colostrum which contains several antibodies (like IgA)
PR

absolutely essential to develop resistance for the new-born babies.

Twins
R.

Twins: Natural selection and evolution favours, birth of one child as single baby tends to be healthier and also their
D

chance of survival is better. Mother can also take care of single child in better way. But some time mother female
produces two babies at a time. Sometime mother also produces triplets and quadruplets, but that is very rare.
Twins are produces in two different ways:
(i) From Single Egg: If the embryo splits early and divide further as an individual, it can develop as a complete young
one. Both the babies formed in this manner have same genetic component as they are formed from the union of same
egg and sperm. They always have same sexes and identical traits. They are therefore called identical twins or maternal
twins.
(ii) From Two Eggs: Some time two eggs are released at the same time from each ovary. When these eggs get fertilized
by two different sperms and each develops as young one separately such twins are genetically different as they are
developed from the union of two different eggs and sperms. They may have same or different sexes. They are known as
non-identical twins or fraternal twins.

156 Class-X BIOLOGY PW


(a) (b)
Fig. 34: (a) Identical twins (b) non-identical twins

Common STDs
Various STDs, the causative organism and symptoms

Name of STDs Causative Organism Symptoms


Syphilis Causes sores and lesions in the genital tract.
Treponema pallidum (a bacterium)

R
Burning sensation during urination.

A
AIDS (Acquired Immuno HIV (Human Immuno Deficiency Destroy the immune system of body.

M
Deficiency Syndrome) Virus) Persistent cough and fever. Body attacked
by other diseases like pneumonia, T.B. and

U
certain cancers.
Gonorrhoea
K
Neisseria gonorrhoeae (a bacterium) Infects mucous membranes of the urogenital
tract.
T
Genital warts Human papilloma virus Painful lumps on vagina, penis, anus or
N

upper thighs, itching or bleeding.


A
SH
A
PR
R.

1. Offspring formed by asexual method of reproduction 3. A Sporophyte of an angiosperm is meant for


have greater similarity among themselves because (a) Production of gamete
D

(i) Asexual reproduction involves only one parent (b) Production of gamete
(ii) Asexual reproduction occurs before sexual (c) Production of spores
reproduction (d) Asexual reproduction
(iii) Asexual reproduction does not involve gametes 4. Number of male gametes in a pollen grain about to
(iv) Asexual reproduction occurs involves two enter embryo sac is
parents. (a) 1 (b) 2
(a) (i) and (ii) (b) (i) and (iii) (c) 3 (d) 4
(c) (ii) and (iv) (d) (iii) and (iv) 5. If haploid DNA content of a plant cell is 12 nanograms,
pollen grain about to enter embryo sac must have
2. Root system in grafting is called ______ nanograms of DNA
(a) Scion (b) Stock (a) 12 (b) 24
(c) Radicle (d) Plumule (c) 36 (d) 48

How Do Organisms Reproduce? 157


6. If haploid DNA content of a plant cell is 12 nanograms, (c) To ensure the survival of the diploid zygote
embryo sac must have______ nanograms of DNA (d) To ensure that the ovary will not develop if double
(a) 24 (b) 48 fertilisation does not take place
(c) 12 (d) 108 11. Which one of the following is mismatched in
7. Development of an organism from an unfertilised ovum angiosperm?
is called: (a) Pollen grains – Triploid
(a) oogenesis (b) Megaspore – Haploid
(b) parthenogenesis (c) Synergid – Haploid
(c) vegetative propagation (d) Endosperm – Triploid
(d) asexual reproduction 12. Anther has Microspore mother cells that undergo
8. Which among the following statements are true for meiosis to form microspores. Pick the CORRECT
unisexual flowers? number of chromosomes in these plant parts or cells
(i) They possess both stamen and pistil if the haploid number of chromosomes in the plant
(ii) They possess either stamen or pistil species is 12

R
(iii) They exhibit cross pollination

male gamete
Pollen grain
Microspore

Microspore
mother cell
Filament
A
(iv) Unisexual flowers possessing only stamens cannot

Petal
produce fruits

M
(a) (i) and (iv) (b) (ii), (iii) and (iv)
(a) Cannot be Cannot be 24 12 24 12

U
(c) (iii) and (iv) (d) (i), (iii) and (iv)
known known
9. Which of the following processes represents P and Q?

Flowering plant
K (b) 24 24
(c) Cannot be Cannot be
24
24
12
24
12
12
12
12
T
known known
N
(d) Cannot be Cannot be 24 12 12 12
known known
A

Pollen Ovule 13. Testosterone is produced by


SH

(a) interstitial cells


(b) seminiferous tubules
A

P (c) the hypothalamus


PR

(d) the prostate


Seed
14. In the male mammals, excretory and reproductive
systems share
R.

Q
(a) the vas deferens (b) the urethra
D

(c) the seminal vesicle (d) the prostate


Seedling
15. During pregnancy:
P Q (a) the ovarian and uterine cycles occur more quickly
(a) Fertilization Pollination than before.
(b) Pollination Fertilization (b) GnRH is produced at a higher level than before.
(c) Dispersal Germination (c) the ovarian and uterine cycles do not occur.
(d) Fertilization Germination (d) the female secondary sex characteristics are not
10. Flowering plants show double fertilisation maintained.
(a) To produce the endosperm which have food 16. Which of the following hormones passes feedback to
reserves and ensures successful germination of cause the LH surge of the menstrual cycle?
seeds (a) Estrogen (b) Progesterone
(b) To protect the seeds from unfavourable conditions (c) FSH (d) GnRH

158 Class-X BIOLOGY PW


17. The function of the acrosome in the sperm head is to (c) Sperms must be released after ovulation for
(a) provide ATP for flagellar movements fertilisation to take place.
(b) control DNA replication in the sperm (d) The uterus lining is washed out of the female body
during menstruation.
(c) store enzymes used for penetrating the egg during
fertilization 21. Which among the following is not the function of testes
at puberty?
(d) enclose the genetic material
(i) Formation of germ cells.
18. Following ovulation, the corpus luteum develops under (ii) Secretion of testosterone.
the influence of (iii) Development of placenta.
(a) progesterone (b) FSH (iv) Secretion of estrogen.
(c) LH (d) estrogen (a) (i) and (ii)
19. (b) (ii) and (iii)
(c) (iii) and (iv)
Column-I Column-II
(d) (i) and (iv)
(P) Seminiferous (i) Manufactures egg

R
tubule 22. Read the following statements about reproductive

A
(Q) Fallopian tube (ii) Progesterone health and select the INCORRECT statements(s) from

M
(R) Ovary (iii) Sac enclosing embryo the given options.
(S) Corpus luteum (iv) Manufactures sperm (i) Diseases like gonorrhoea, syphilis and HIV- AIDS

U
(T) Amnion (v) Site of fertilization are sexually transmitted.

(a) (P)-(v); (Q)-(i); (R)-(iv); (S)-(ii); (T)-(iii) K(ii) Gonorrhoea and syphilis are caused by fungus,
transmitted sexually.
T
(b) (P)-(iv); (Q)-(v); (R)-(i); (S)-(ii); (T)-(iii)
(iii) Using condoms during sex helps to prevent
N
(c) (P)-(i); (Q)-(ii); (R)-(iii); (S)-(v); (T)-(iv)
transmission of many of the infections to some
A

(d) (P)-(i); (Q)-(ii); (R)-(iii); (S)-(iv); (T)-(v) extent.


SH

20. The diagram shows the menstrual cycle of a woman (iv) Examples of viral and bacterial infection STDs are
during the month of September. AIDS-HIV and warts respectively.
(a) (ii) and (iv)
A

Sunday 7 14 21 28
(b) (ii) and (iii)
PR

Monday 1 8 15 22 29
(c) (ii) Only
Tuesday 2 9 16 23 30
(d) (i) and (iv)
Wednesday 3 10 17 24
R.

Thursday 4 11 18 25 23. Read the following statements about reproductive


health and select the CORRECT statements(s) from
D

Friday 5 12 19 26
the given options.
Saturday 6 13 20 27
(i) AIDS is curable.
Key
(ii) AIDS could be managed with the help of medicines.
= ovulation
(iii) Oestrous cycle also includes shedding of
= menstruation endometrium.
Why can fertilisation not take place if sperms are (iv) Meiosis I and II during oogenesis produces 2
released into the vagina on 8th September? EQUAL cells both the times
(a) Sperms are washed out of the female uterus by the (a) (ii) and (iv)
menstrual flow. (b) (ii) and (iii)
(b) Sperms can survive in the female reproductive (c) (ii) Only
system for only 3 or 4 days. (d) (i) and (iv)

How Do Organisms Reproduce? 159


Competitive Corner

1. This diagram shows that process of __________ 7. 'AIDS' virus is called (Jharkhand 2018 stage 1)
 (Chandigarh 2017 SAT) (a) ARV (b) HTLV
(c) HIV (d) All of these
8. In meiosis each of the four daughter cells have one
set of chromosomes. Due to randomness of process
of chromosome separation in meiosis, larger number
of chromosome combinations can form gametes. How
many such chromosome combinations in the gametes
are possible in case of humans, assuming there is no
crossing-over taking place? (Jharkhand 2018)
(a) 222 (b) 223

R
(a) Binary Fission (c) 246 (d) 234

A
(b) Multiple fission
9. Find the odd one (Kerala 2017, SAT)

M
(c) Regeneration
(a) Fragmentation (b) Regeneration
(d) Budding

U
(c) Budding in Yeast (d) Budding in Hydra
2. In plants, the developing embryo is nourished by
endospermic tissues its cell consists of:
 (Jharkhand, 2017 SAT)
K
10. How many ova are produced from 50 primary oocyte?
 (Odisha, 2018, 2019 SAT)
T
(a) 50 (b) 100
(a) One genome (Haploid)
N
(c) 200 (d) 300
(b) Two genomes (Diploid)
A

(c) Three genomes (Triploid) 11. Parthenogenesis is (Andhra Pradesh, 2019, SAT)
SH

(d) Four genomes (Tetraploid) (a) Asexual Reproduction


(b) Sexual Reproduction
3. Which option indicates the parts of gynoecium?
(c) Artificial Propagation
A

 (Kerala 2017, SAT)


(d) Natural Propagation
PR

(a) Pollen grain, filament, ovary, tube nucleus


(b) Polar nuclei, stigma, ovule, style 12. The hormone, secreted by ovary during later phases of
pregnancy is (Rajasthan, 2019, stage 1 SAT)
(c) Style stamen, stigma, generative nucleus
R.

(a) Testosterone (b) Estrogen


(d) Ovary, anther, filament, polar nuclei
(c) Progesterone (d) Relaxin
D

4. The hormone that promotes lactation and simulates the


uterus to contract (Kerala 2017, SAT) 13. There is a very yellow dust which comes away on
fingers whenever we touch the middle of the flower.
(a) Calcitonin (b) Thyroxine
These tiny yellow grains are one of the most precious
(c) Oxytocin (d) Vasopressin substances in nature because they contain the secret of
5. How many secondary spermatocytes are required to plant life. What is the dust called?
form 400 spermatozoa? (Kerala 2017, SAT)  (Telangana, 2019, Stage 1 SAT)
(a) 100 (b) 200 (a) Pollen (b) Sperm
(c) 300 (d) 400 (c) Spore (d) Sporocyst
6. Which of the following hormone is the other name of 14. Read the given statements and select the correct option.
LH? (Odisha 2017 SAT) Statement - I: In humans, the gamete contributed
(a) TSH (b) ICSH by the male determines whether the child produced
(c) ACTH (d) FSH will be the male or female.

160 Class-X BIOLOGY PW


Statement-II: Sex in humans is dependent on the (c) Two fertilizations occur in a flower-one fertilization
X-chromosome or Y-chromosome of the father. results in the formation of a diploid zygote and the
 (Telangana, Stage 1 SAT) second fertilization results in the formation of a
(a) Both Statements-I and II are true and Statement - II triploid endosperm.
is the correct explanation of Statement-I. (d) Two pollen grains sending two pollen tubes inside
(b) Both Statements-I and II are true, Statement-II is the ovary, resulting in the formation of two seeds
not the correct explanation of Statement inside the fruit.
(c) Statement-I is true and Statement -II is false 19. The human embryo gets nutrition from the mother
(d) Both Statements-I and II are false blood with the help of a special organ called (2016)
15. Read the following statement and select the correct (a) Zygote (b) Ovary
option (Chandigarh, 2020, Stage 1 SAT) (c) Oviduct (d) Placenta
A: Wind pollinated flowers need to produce more 20. Which one of the following is correct route for passage
amount of pollen grain of sperms? (2014)
(a) Testes - scrotum - vasdeferens - urethra - penis
B: Seeds from cross pollinated flowers produce

R
(b) Scrotum - testes - urethra - vasdeferens - penis
weaker and less healthy plants.

A
(c) Testes vasdeferens - urethra - seminal vesicles
(a) A is false, B is true

M
(d) Testes - vasdeferens - urethra - penis
(b) A is true, B is false

U
(c) Both A and B are true 21. Embryonic development is a complex multi-step
process that involves transition from single cellularism
(d) Neither A nor B is true
16. Sperms are produced in the: Kto multicellularism. In animals, all of the following are
associated with embryonic development except
T
 (Delhi, 2020, Stage 1 SAT)  (2018)
N

(a) Seminiferous tubules (a) Migration of cells to specific areas


A

(b) Interstitial cell (b) Formation of germ layers


SH

(c) Vas deferens (c) Activation of all the genes in each cell
(d) Prostate gland (d) Inductive tissue interactions
A

17. Grafting is possible among dicot plants but not in 22. Sexual reproduction in plants and animals involve
PR

monocot plants. This is due to presence of one of the the union of two gametes to form a single cell called
following conditions in dicot plant (2021) zygote. Gametes include the egg and sperm cells.
(a) Presence of open vascular bundles Zygote is formed after the sperm fertilizes the egg,
R.

(b) Presence of collenchyrnas tissues resulting in diploid chromosome. The zygote develops
(2016)
D

(c) Presence of intercalary rneristern into ... . 


(d) Larger diameter of stern (Hint: The following figure indicates fertilized egg)
18. “Double fertilization” is a complex mechanism of
flowing plants that is also unique to angiosperms.
Choose the most appropriate statement from the Diploid (2n)
options listed below the explains this phenomenon.
 (2020)
(a) Fertilization in two flowers of the same plant
forming endosperms
(b) Two male gametes fertilize two eggs inside the
ovule as a result the ovary gives rise to bigger (a) Embryo (b) Endosperm
fruits (c) Carpel (d) Ovule

How Do Organisms Reproduce? 161


23. The figure below shows the process of egg production Which of the following statements is correct?
and early development of a healthy fertilized egg in (a) The sterilization and douche methods can be
the human reproductive system. (2015) considered good methods of avoiding pregnancy.
(b) The condom is completely effective in preventing
Y
pregnancy.
(c) Ninety eight percent of pregnancies can be
prevented by using hormone pills.
X (d) The contraceptive success rate of the rhythm
Sperm method, where sex is avoided during ovulation is
unfertilized about 35%.
Z egg
W 25. Human chorionic gonadotropin hormone (hCG) is
injected into the blood of a mouse. Antibodies are
Which of the following statements is NOT correct?
produced in the mouse blood in response to this
(a) Three polar bodies are attached to W. substance. What can the serum from this mouse blood
(b) X produces progesterone. be used for? (2009)

R
(c) The chromosome number for each cell in Y is 46. (a) Treatment of infertility

A
(d) Z is at the stage of blastocyst (b) Pregnancy test

M
24. Consider Figure 5, the graph comparing the effectiveness (c) Prevention of fertilization
of different methods of contraception. (2011) (d) Paternity test

U
Sterilization

Hermenal

K
Rhythm method

Pills

100 -
T
90 -
80 -
Condom

N
70 -
60 -
50 -
A

40 -
30 -
20 -
SH

10 -
0 -
A
PR
R.
D

162 Class-X BIOLOGY PW


2. Jasmine & Rose plants are grown using cutting
School Level 3. Spirogyra is a filamentous green alga. All cells are
similar and bear no specialization. Under favorable
conditions due to physical impact the filaments
Quick Recall may break. These broken pieces begin to grow and
divide, behaving like complete organisms.
Fill in the Blanks
4. Spore production is an asexual means where spores
1. Ameoba 2. Budding
are produced in huge quantities. These spores spread
3. Stamen 4. Embryosac quickly over a wide distance.(0.5 Mark) They are
5. Stigma, pistil 6. Plumule resistant to adverse conditions like high temperature

R
7. Testis 8. Oviduct and begin to produce the new organism as soon as
they find a nutritious environment. This ensures its

A
9. Vagina 10. Vasectomy
survival and success.

M
True and False Statements 5. Yes, shaded part in Figures D and E represent the

U
1. True 2. False 3. True regenerated halves.
4. False 5. True 6. True
7. False 8. False 9. False K
T
10. True
N
D E
Match the Following 6. Inside the anther there are pollen sacs. These pollen
A

1. (b) 2. (a) 3. (b) 4. (a) 5. (b) sacs produce pollen via meiosis. Pollen grain houses
SH

the cell which divides to produce the male gamete.


Multiple Choice Questions Ovule consist of cells which will produce the female
gamete eventually.
A

1. (d) 2. (a) 3. (b) 4. (c) 5. (b)


6. (a) 7. (b) 8. (c) 9. (c) 10. (c) 7. A unisexual flower will have only one of the sexual
PR

11. (a) 12. (d) whorl-stamen (the male) or pistil (the female).
Example. Eg. Mango. A bisexual flower has both
Assertion & Reason Type Questions the sexual whorls. Eg: Papaya.
R.

1. (a) 2. (a) 3. (a) 4. (b) 5. (b) 8. In a flower fertilisation requires both male and
D

female gametes. If pollination does not occur, male


Statement Type Questions gamete is not available hence fertilisation cannot
1. (d) 2. (b) 3. (d) 4. (d) 5. (a) take place.
9. Plant’s main body is a diploid structure. Gametes
are haploid structures. If the male gamete has 24
Subjective Questions
chromosomes, the female gamete will also have, 24
Very Short Answer Type Questions chromosomes.
A zygote is formed from fusion of male and a
1. The genetic material in organisms is DNA (in some
cases could be RNA). It bears the code of proteins. female gamete. It should have 48 chromosomes.
Proteins control metabolic reactions. These reactions 10. (a) Sexual reproduction involves two parents with
control body designs. The variations arising copying different sets of characters
of DNA give rise to different body designs. (b) The gene combinations are different in gametes.

How Do Organisms Reproduce? 163


11. (b) Fusion of nuclei of male and female gamete is the male gametes to the egg cell in the ovule for
known as fertilization. fertilisation. Pollen tube grows through style. Ovary
12. When the embryo gets implanted, a structure called is the bottom expanded part of the carpel carries
the placenta develops. This placenta bears villi rich ovule which houses the female gamete. The female
in blood vessels of mother and the child allowing gamete is fertilised to form zygote.
diffusion of substances between them- nutrition, 5. A cross pollination is the transfer of pollen from
gases and waste materials. anther of a flower to the stigma of another flower
on a different plant body belonging to the same
13. Oral contraceptives are formulations of estrogen species whereas self pollination is the transfer of
and progesterone mix. These hormones in high pollen grains from anther to stigma of the same
concentrations inhibit the release of FSH and LH flower. Site of fertilization is the ovule. The product
hormones. This inhibits the maturation and release is a zygote which will develop into a new plant
of ovum. eventually (if the seed is planted).
6. (a) (i) The uterus lining thickens. It develops more
Short Answer Type Questions blood vessels. This enables it to supply

R
1. In Binary fission, a single organism splits into two, nutrients to the growing embryo.

A
there is no parent left as the parent itself splits. At (ii) When no fertilization takes place the uterine
first, the nucleus divides and the cell elongates. lining that was prepared to house the embryo

M
Subsequently, the cytoplasm divides as well. This sheds. Females experience menstruation, the
discharge of blood and tissue, the thickened

U
mode is found in Amoeba.
lining that was prepared to support the
2. Amoeba reproduces via binary fission.
K(b) 
pregnancy.
Placenta has rich supply of capillaries which
T
help the exchange of nutrients between the
N
developing embryo and the mother.
A

7. (a) The act of sex is a very intimate act. Several


pathogens can be transferred during this and
SH

Amoeba Elongation Division of this can lead to diseases. Such diseases are
(parent cell) of nucleus nucleus and Two daughter
cytoplasm cells called as sexually transmitted diseases. AIDS
A

and Hepatitis are viral STDs. Gonorrhea and


3. Yeast, a unicellular fungus develops a bud on its Syphilis are bacterial STDs.
PR

surface, the outgrowth. This outgrowth develops (b) Contraception is the method of preventing
further and may remain attached to the parent. This pregnancy by use of different contraceptive
R.

set of events may continue and a chain of Yeast methods like condoms, hormonal pills, etc.
cells may develop. When very large, this may break 8. (a) Condom is a barrier contraceptive and prevents
D

apart. mixing of body fluids. Hence it prevents


Developing Daughter unwanted pregnancy and STDs.
buds Yeast
Parent (b) In some areas of India, fetus were aborted based
Yeast on their sex. This is sex selective abortion. It is
immoral. Besides, this skews the sex ratio in a
society and can disrupt population dynamics.
Long Answer Type Questions
4. A carpel is a unit of female whorl- the gynoecium.
1. (a) 
Vegetative reproduction is a means where
It has 3 parts- stigma, style and ovary. Stigma is vegetative parts of the parent organism, like stem,
the upper expanded part which receives the pollen. root and leaves, are used to produce offspring.
The pollen germinates to give rise to the pollen Vegetative propagation can happen via stem in
tube. The function of the pollen tube is to carry Sugarcane, and via leaves in Bryophyllum.

164 Class-X BIOLOGY PW


(b) 
This mode produces progeny which are During sexual reproduction two types of gametes
genetically similar to parents. This ensures that fuse. Although the gametes contain the same number
characteristics remain the same. of chromosomes, their DNA is not identical. This
(c) Hydra produces a bud, which is an outgrowth. This situation generates variations among the offsprings.
matures further and develops into a new hydra. It
can stay attached or detached after sometime. 4. (a) 
Tentacles
2nd Male Gamete
1st Male Gamete

Ultimately
Body Bud
develops bud

R
Fully mature gamete

A
bud Hydra

M
(d) Bryophyllum shows vegetative propagation by
leaves. Its leaves has buds. When these leaves (b) Pollen tube carries the male gamete to the ovule

U
fall they can give rise to new plants. for fertilisation.
2. Budding, fragmentation and regeneration are
considered as asexual types of reproduction because K
(c) Ovule develops into seed, Ovary develops into fruit.
T
all of them involve only one parent and gametes are 5.
N
not involved in reproduction Pollination Fertilisation
A

The process or The fusion of male and


SH

mechanism of transfer female gaemtes giving


of pollen grains from rise to zygote is termed
the anther to the stigma fertilisation.
A

is termed pollination.
PR

The site of fertilisation is ovule.


The product of fertilisation is zygote.
R.
D

2nd Male Gamete


1st Male Gamete
3.
Asexual reproduction Sexual reproduction
Involves only one parent Often involves two
parents
Gametes are not produced Gametes are produced
No fertilisation and Fertilisaton and zygote
zygote formation formation is observed.
Meiosis does not occur Meiosis occurs at gamete
at anytime during the time of gamete
reproduction formation

How Do Organisms Reproduce? 165


6. (a) Human sperms receive secretions from various 11. (a) (i) ovary (ii) Fallopian tube
glands. Seminal Vesicle secretions contain (ii)
fructose which is an energy source for the sperm
in the semen. Sperm has 23 chromosomes- 22 (i)
autosomes and X or Y sex chromosome.
(b) Sex chromosomes X and Y determine the sex of
the child. (i) Female has XX (ii) Males have XY
7. (a) 
A-Ureter, B-Seminal Vesicle, C-Urethra,
D-Epididymis
(b) Testis secretes testosterone. Testosterone is
responsible for production and development of
sperms and secondary male sexual characteristics. (b) Before the ban many would use the technique to
(c) Seminal Glands (B) adds source of energy for find out the sex of the baby and kill it incase it
the sperm. It also has enzymes that prepares the was a girl child. This is called female foeticide. To
sperm for its journey in the uterus. Urethra (C) is prevent this government banned it. In the long run
the common path of the sperm and urine and leads it ensures the sex ratio of birth remain good so that

R
them outside of the body. we have equal number of males and female in the

A
society and the population, social and economical
8. (a) Functions: dynamics remain stable.

M
™ Ovary: produces female gamete and secretes
female sex and pregnancy maintenance

U
hormones, i.e., estrogen and progesterone. Case-Based Type Questions
™ Oviduct: collects ovum and provides the path
for sperm journey to fertilise the female gamete K
Case Study-I
T
™ Uterus: site for growth of the embryo and path 1. (a) 2. (b) 3. (d) 4. (b)
N
for sperm towards ovum
Case Study-II
A

(b) The placenta facilitate the supply of oxygen and


nutrients to the embryo and also removal of carbon
SH

1. C,D,E are producing the maximum number of seeds


dioxide and excretory/waste materials produced by per flower and produce a very good amount of nectar
the embryo. also. Hence, we can conclude that C,D,E are witnessing
A

9. (a) Female Reproductive System sexual reproduction.


PR

(b) 1- ovary, 2- Isthmus of oviduct, 3-uterus, 4-vagina, 2. Species A and B will be least impacted as they have
5-Infundibulum of oviduct 6- cervix the least visit rate of humming birds.
(c) Estrogen and Progesterone 3. Sexual reproduction always leads to more variation
R.

(d) 2, Oviduct is involved in fertilisation. whereas asexual reproducers suffer from inbreeding
3→ Uterus, is the site of implantation. depression.
D

(e) 1. In Males- Vasectomy, Chances are selection of resistant species will take
2. In Females- Tubectomy. place in sexual reproducing varieties as they come
10. (a) D > C > B > A up with more variations thus, species E have a
(b) 
Fertilisation takes place in the oviduct. better chance of surviving.
Implantation is the act of attachment of the 4. Pollinator and plant species are often codependent.
embryo to the uterine wall. They, at many a times, co-evolve. A change or
(c) 
Egg has 23 chromosomes and Zygote has 46 phenomenon in one drives a change of phenomenon
chromosomes. in others. Red flower cost the plant energetically and
the output was successful and efficient reproduction,
(d) Gestation is the act of a mother bearing the child
in her uterus till birth. It lasts for 9 months. but, the pollinators became color blind so plants will
evolve in such a way that they produce black and white
(e) Copy diagram from text.
flower such that the color blind pollinators might still

166 Class-X BIOLOGY PW


respond to black and white flowers. Hence answer 4. (c) 
Oxytocin hormone secreted by the posterior
is (a). Increase in fragrance might add a recognition lobe of the pituitary gland is responsible for the
advantage. Root/ stem branching might allow asexual contraction of the uterus during parturition and
reproduction to compensate for loss of efficiency due expulsion of milk from the mammary glands.
to colour blindness. 5. (b) 
One secondary spermatocyte produces two
5. Before insects, wind or water were chief agents of spermatids by meiosis II. One spermatid
pollination. Both led to major loss of pollen and in produces one sperm. Thus, 200 spermatocytes are
efficient as pollen from any species could land on any required for the formation of 400 spermatozoa.
flower. Flowers produce rewards-nectar. Recognition 6. (b) 
LH is also known as ICSH (Interstitial cell
devices like colour, fragrance allowed the transmission stimulating hormone) as it stimulates leydig
of pollen in an efficient manner, making their cells or interstitial cells and stimulates synthesis
reproduction process a lot more efficient. Hence we and secretion of testosterone.
see a majority of vascular plants to be angiosperms-
7. (c) AIDS (Acquired Immunodeficiency syndrome)
flowering plants.
is caused by HIV (Human Immunodeficiency
Case Study-III virus).

R
1. (b) 2. (b) 3. (d) 4. (a) 8. (b) 
To compute the chromosome combinations,

A
during gametes formation = 2n where

M
Case Study-IV n = number of chromosomes.
1. (d) 2. (b) 3. (b) 4. (c) In the case of humans, One set of daughter cells

U
contain = 23 chromosomes.

Competitive Level K
Thus, the number of gametes produced = 223.
T
9. (b) Regeneration is the odd one.
N
10. (a) 
50, As 1 primary oocyte develops into one
ovum.
A

Multiple Choice Questions


11. (a) Parthenogenesis is the production of an animal
SH

1. (b) 2. (b) 3. (c) 4. (b) 5. (a) without fertilization thus, it is a type of asexual
6. (c) 7. (b) 8. (b) 9. (d) 10. (a) reproduction.
A

11. (a) 12. (a) 13. (a) 14. (b) 15. (c) 12. (d) 
Relaxin hormone is secreted by the ovary
PR

16. (a) 17. (c) 18. (c) 19. (b) 20. (c) during later stages of pregnancy in preparation
21. (c) 22. (c) 23. (c) for childbirth. It relaxes the ligaments in the
pelvis and softens and widens the cervix.
R.

13. (a) Pollen grains are the yellow dust which comes
Competitive Corner
D

away on fingers whenever we touch the middle


of the flower.
1. (c) The given diagram in the question illustrates the
regeneration mechanism in Planaria. 14. (a) In humans, the sex of the baby is determined by
the chromosome received from the father. Thus
2. (c) 
Endosperm is a triploid tissue formed by the
both statement I and II are correct and statement
fusion of the 2nd male gamete with the central
II is the correct explanation of statement I.
cell and provides nourishment to the developing
embryo. 15. (b) Wind pollinated flowers need to produce a large
amount of pollen as a large amount of pollen
3. (b) The gynoecium is the female reproductive whorl of grains are lost during pollination.
the flower and consists of pistil. The Pistil consists
16. (a) Each testis has about 250 compartments called
of three parts namely stigma, style and ovary.
testicular lobules. Each lobule contains one
Ovary consists of ovule which in turn consists of
to three highly coiled seminiferous tubules in
the female gametophyte having polar nuclei.

How Do Organisms Reproduce? 167


which sperms are produced. Each seminiferous 20. (d) S
 eminiferous tubules → Rete testis → Vasa
tubule is lined on its inside by two types of efferentia → Epididymis → Vas deferens →
cells called male germ cells (spermatogonia) Ejaculatory duct → Urethra.
and Sertoli cells. The male germ cells undergo
21. (c) 
As a result of differentiation different cells
meiotic divisions finally leading to sperm
formation, while Sertoli cells provide nutrition begin to act differently. They have different
to the germ cells. set of capacities as different set of genes are
active in them each producing respective set of
17. (a) Grafting is possible in dicot plants because of
the presence of a cambium ring due to which proteins (enzymes)
the vascular bundles are called open vascular 22. (a) 
After fertilization, the following changes are
bundles. observed in a flower: There is the formation
18. (c) of a diploid zygote and it develops into an
DOUBLE embryo, which forms the future plant. The
FERTILIZATION
endosperm cells serve as a source of nutrition
for the developing embryo. The ovule becomes

R
Syngamy/ Triple the seed.

A
Fertilisation Fusion
23. (a) Meiotic divisions of the Oocyte are asymmetrical.

M
One male gamete The 2nd male gamete The first division produces only 1 polar body. It

U
fuses with the egg cell fuses with the 2 polar is only after fertilisation that the second meiotic
and forms of diploid nuclear called as triple division takes place. Hence three polar bodies
zygote fusion which results
in the formation of
K will be with the Egg and not attached to W.
T
triploid forms called as 24. (c) Oral pills are 98% effective. That means that
N
endosperm. if 100 people take the medication perfectly,
A

two or fewer people would become pregnant.


19. (d) 
The placenta facilitate the supply of oxygen Taking a pill perfectly can be difficult, though.
SH

and nutrients to the embryo and also removal


Typical use averages how well a method works
of carbon dioxide and excretory/waste materials
to prevent pregnancy when real people use it in
A

produced by the embryo. The placenta is


connected to the embryo through an umbilical real life.
PR

cord which helps in the transport of substances 25. (b) hCG hormone is present in the urine of a pregnant
to and from the embryo. female which is detected in a pregnancy test.
R.
D

168 Class-X BIOLOGY PW

You might also like